Download as pdf or txt
Download as pdf or txt
You are on page 1of 152

Vector &

Three-Dimensional
Geometry
DISCLAIMER
“The content provided herein are created and owned by various authors and licensed
to Sorting Hat Technologies Private Limited (“Company”). The Company disclaims all
rights and liabilities in relation to the content. The author of the content shall be solely
responsible towards, without limitation, any claims, liabilities, damages or suits which
may arise with respect to the same.”
Vector & Three-Dimensional
Geometry
VECTORS AND SCALARS :
The physical quantities (we deal with) are
generally of two types:

Scalar Quantity:
A quantity which has magnitude but no sense
of direction is called scalar quantity (or scalar),
e.g., mass, volume, density, speed etc.

Vector Quantity:
A quantity which has magnitude as well as a
sense of direction in space and obey the laws
of vector algebra is called a vector quantity,
e.g., velocity, force, displacement etc.

Notation and Representation of Vectors :


  
Vectors are represented by a,b, c and their
magnitude (modulus) are represented by a, b,
  
c or a , b , c . The vectors are represented by

directed line segments.



For example, line segment OP represents a
vector with magnitude OP (length of line
segment), arrow denotes its direction. O is
initial point and P is terminal point, also called
as head and tail of vector respectively.

KINDS OF VECTORS :
1. Zero or null vector:
A vector whose magnitude is zero is called 
zero or null vector and it is denoted by 0 or 0
Vector & Three-Dimensional Geometry
Point to Remember!!!
. The initial and terminal points of the directed
line segment representing zero vector are
(i) â = 1
coincident and its direction is arbitrary.
(ii) U
 nit vectors parallel to x - axis, y
2. Unit vector:
- axis and z - axis are denoted by
A vector of unit magnitude is called a unit ˆi, ˆj and k̂ respectively.

vector. A unit vector in the direction of a is
denoted by â. Thus (iii) T
wo unit vectors may not be
 equal unless they have the
 a  Vector a
a =
=  same direction.
a  Magnitude of a

1.
3.
Equal Vectors: 

Two vectors a and b are said to be equal, if
 
(a)  a = b

(b) they have the same sense of direction


4. Co-initial vectors:
Vectors having same initial point.
5. Free vectors:
All such vectors are those which when
transformed into space from one point
to another point without affecting their Know the facts
magnitude and direction, can be considered
as equal. i.e. the physical effects produced In mathematics we mainly deal
by them remains unaltered. with free vectors.
e.g. displacement, velocity.
6. Localised vectors:
e.g. force, different physical effect if line of
application is changed.

ADDITION OF VECTORS :
Triangle law of addition :
If two vectors are represented by two
consecutive sides of a triangle, then their
sum is represented by the third side of the
triangle but in opposite direction. This is
known as triangle law of addition of vectors.
     
Thus, if = = b and AC = c then
AB a,BC
     
AB + BC = AC i.e. a + b =
c
Vector & Three-Dimensional Geometry

Converse of triangle law is also true.

Parallelogram Law of Addition:


If two vectors are represented by two
adjacent sides of a parallelogram, then their
sum is represented by the diagonal of the
parallelogram whose initial point is the same
as the initial point of the given vectors. This
is known as parallelogram law of addition of
vectors.

2.
     
Thus if = OA a,= OB b and OC = c then
  
OA + OB = OC
  
i.e. a + b =c

POSITION VECTORS:
Let
 O be fixed point in space, then vector
OP (P is any point in space) is called position
vector of point P w.r.t. O. If A and B are any
two point in space then
  
AB = p.v of B – p.v of=A OB − OA
  
i.e.  AB= b − a

MULTIPLICATION OF A VECTOR BY SCALAR:


 
If a is a vector and m is a scalar, then ma is

a vector parallel to a whose modulus is m

times that of a . This multiplication is called
 
SCALAR MULTIPLICATION. If a and b are
vectors and m, n are scalars, then :
  
m= ( ) ( )
a =a m ma
Point to Remember!!!
  
m= ( ) ( ) ( )
na n=ma mn a
   (
Position vector of a point P x, y, z )
( )
m + n a = ma + na
in terms of its cartesian coordinate
   
( )

m a + b = ma + mb is OP = x ˆi + y ˆj + z kˆ

DISTANCE BETWEEN TWO POINTS :


Let A and B be two given points whose
( )
Vector & Three-Dimensional Geometry
coordinate are respectively x 1 , y 1 , z1 and

( x2 , y2 , z2 )
 
If a and b are p.v. of A and B relative to
point O, then

a = x 1ˆi + y 1ˆj + z1kˆ

b = x2ˆi + y 2ˆj + z 2kˆ
Now,
   
( ) ( ) ( )
AB = OB − OA = b − a = x2 − x 1 ˆi + y 2 − y 1 ˆj + z2 − z1 kˆ

3.
 
Distance between the points A and B =
magnitude of

( ) + ( y2 − y 1 ) + (z2 − z1 )
2 2 2
AB = x2 − x 1

COLLINEAR VECTORS OR PARALLEL VECTORS :


Vectors which are parallel to the same
line are called collinear vectors or parallel
vectors. Such vectors have either same
direction or opposite direction. If they have
the same direction they are said to be like
vectors, and if they have opposite directions,
they are called unlike vectors.
 
In the diagram a and c are like vectors
 
whereas a and b are unlike vectors.
   
( ) (
i.e.  a = k 1c k 1 > 0 ,  a = k2b  k2 < 0 )
COPLANAR VECTORS :
If the directed line segments of some given
vectors are parallel to the same plane then
they are called coplanar vectors. It should be
noted that two vectors are always coplanar
but three or more vectors may or may not
be coplanar.

SECTION FORMULAE
 :

If a and b are the position vectors of two

points A and B, then the position vector c of
Point to Remember!!!
Vector & Three-Dimensional Geometry

a point P dividing AB in the ratio m : n is


given by
 
 mb + na Particular Case:
c= 1. Position vector of the mid
m+n  
a +b
point of AB is .
2
2. I f the point P divides AB in the
ratio m : n externally, then p.v.

 mb − na
of P is given by c =
m−n

4.
Note:
  
α+β+γ
1. P.V. of the centroid of a triangle ABC =
3
(Concurrency of medians) 
 
aα + bβ + cγ
2. P.V. of incentre of the A =
a +b+c
(Concurrency of internal angle bisectors)
     
−aα + bβ + cγ aα − bβ + cγ
Excentres of the A are ;
−a + b + c a −b+c
  
aα + bβ − cγ
and .
a +b−c
3. P.V. of circumcentre

of the
 
αsin2A + βsin2B + γsin2C
A=
∑ sin2A
(Concurrency of perpendicular bisectors of sides)
4. P.V. of orthocenter
 of the
 
atanA + btanB + ctanC
A=
∑ tanA
(Concurrency of altitudes)

Q. Through the middle point M of the side AD of ||gm ABCD, a straight line BM is
drawn intersecting AC at R and CD produced at Q. use vectors to prove that
QR = 2RB.

DM QD  
Sol. MA AB
= ⇒ QD = AB
     
d − Q =b ⇒ Q =d − b
Let R divides AC in ratio λ : 1 and QB in
µ : 1 ratio by section formula
       

Vector & Three-Dimensional Geometry


 λ(b + d) µb + Q µb + (d − b) (µ − 1)b + d
=R = = =
λ+1 µ+1 µ+1 µ+1
   
λ(µ + 1)b + λ(µ + 1)d = (µ − 1)(λ + 1)b + (λ + 1)d
λ(µ + 1) = (µ − 1)(λ + 1) & λ(µ + 1) = λ + 1
λµ + λ = µλ − λ + µ − 1 & λµ + λ = λ + 1
1
2λ = µ – 1 λ=
µ
2
=µ−1
µ
2 = µ2− µ ⇒ µ2 – µ – 2 = 0 ⇒ µ = –1, 2
∴ µ = 2 ⇒ QR = 2RB
5.
Q. In ∆AOB, E is the mid point of OB and F divides BA in the ratio 1 : 2 use vectors
OP 3
to prove that = .
PF 2

Sol. Let P divides of in ratio λ : 1 & EA in ratio


  
 2b + a 
λ   b
  3  µ(a) +
µ:1–p =
=   2
λ+1 µ+1

λ     b
(µ + 1)(2b + a) = (λ + 1)  µa + 
3  2 

2λ(µ + 1)  λ(µ + 1)   (λ + 1) 
b+ a = (λ + 1)µa + b
3 3 2
2λ(µ + 1) λ + 1 λ(µ + 1)
= & = (λ + 1)µ
3 2 3
4λ(µ + 1) = 3λ + 3 λµ + λ = 3µ(λ + 1)
4λµ + 4λ = 3λ + 3 λµ + λ = 3µλ + 3µ
4λµ = 3 – λ λ = 2µλ+ 3µ
3−λ (3 − λ ) (3 − λ )
µ= λ = 2λ +3
4λ 4λ 4λ
6λ − 2λ2 + 9 − 3λ
λ =

4λ2 + 2λ2 = 3λ + 9
6λ2 = 3λ + 9
2λ2 =λ+3
3
λ= , −1
2
3 0P 3
∴ λ= ⇒ =
2 PF 2
Vector & Three-Dimensional Geometry

Q. Let OACB be parallelogram with O at the origin & OC a diagonal. Let D be the
mid point of OA. Using vector method prove that BD & CO intersect in the
same ratio. Determine this ratio.

Sol. OACB is parallelogram with O as origin.


        a
Let OA= a, OB= b, OC= a + b and OD =
2

CO and BD meets at P .

6.
   
λ·0 + 1(a + b)
∴ OP. = [ along OC]
λ+1

 a + b
OP = …(i)
λ+1
a
 µ  2  + b

  µa + 2b
Again,
= OP   [along BD ⇒ OP= … (ii) ]
µ+1 2(µ + 1)
   
a + b µa + 2a
From Eqs. (i) and(ii) , =
λ + 1 2(µ + 1)
1 µ 1 1

= = and
λ + 1 2(µ + 1) λ+1 µ+1
On solving, we get µ = λ = 2 Thus, required ratio is 2 : 1

  
Q.
 = −i + 3j + 4k & c = 4i − 2j − 6k constitute the sides
If the vectors a = 3i + j − 2k,b

of a DABC, then the length of the median bisecting the vector c is
(A) 2 (B) 14 (C) 74 (D) 6

Sol. (D)
= a 14 b
= 26 c
= 56
1
2b2 + 2a2 − c2
Length of Median =
2
1
=
= 2(26) + 2(14) − 56
2
1 1
= 52 + 28 − 56 = 24
2 2

Vector & Three-Dimensional Geometry


1
= × 2 6 = 6
2

Q. Consider the points A, B and C with position vectors ( −2i + 3j + 5k)
 , (iˆ + 2ˆj + 3k)
ˆ

and 7i − k respectively.


  
Statement-1 : The vector sum AB + BC + CA = 0
Statement-2 : A, B and C form the vertices of a triangle.
(A) Statement-1 is true, statement-2 is true and statement-2 is correct
explanation for statement-1.

7.
(B) Statement-1 is true, statement-2 is true and statement-2 is NOT the
correct explanation for statement-1.
(C) Statement-1 is true, statement-2 is false.
(D) Statement-1 is false, statement-2 is true.

Sol. (C)

AB =(I + 2j + 3k)  − ( −2I + 3j + 5k)


= 3I − j − 2k
 ^ ^ ^ ^ ^ ^ ^ ^
BC = (7 I − K) − (I + 2 j+ 3K) = 6 I − 4K − 2 j

CA =( −2I + 3j + 5k)  − (7i − k)

=−9I + 3j + 6k


   
 + (6I − 4k − 2j)
∴ AB + BC + CA= (3i − j − 2k)  = 0
 + ( −9i + 3j + 6k)

So, A, B, C are collinear. So they will not form vertices of a triangle

Q.
ABCD is a parallelogram whose diagonals meet at P. If O is a fixed point, then
   
OA + OB + OC + OD equals
   
(A) OP (B) 2OP (C)  3OP (D)  4OP

Sol. (D)
Since, P bisects both the diagonal
AC and BD, so
 
∴ OA + OC = 2OP and
       
OB + OD = 2OP ⇒ OA + OB + OC + OD = 4OP

 
Q.
Vector & Three-Dimensional Geometry

If a,b are represented by the sides AB and BC of a regular hexagon ABCDEF,


then vector represented by FA will be
       
(A) a + b (B) b − a (C) a − b (D) 2b − a

Sol. (C)
 
 AC= a + b
 
 
  
=AD 2BC (
= 2b   By property of hexagon AD
= 2BC )
  
∴D   C = DA + AC
    
= −2b + a + b = (
a −b )
    
But F   A =DC ⇒ FA =a − b  
8.
Q. If the mid-points of the consecutive sides of a quadrilateral are joined, prove
that the resulting quadrilateral is a parallelogram.

Sol. Let ABCD be the given quadrilateral and P, Q, R, S


be the mid-points of sides AB, BC, CD and AD
respectively. Let O be origin of reference and let
   
a,b, c, d be the position vectors of A, B, C and D
respectively.
 1    1  
Then, OP = a + b , OQ = b + c
2 2
( ) ( )
 1    1  
(
OR = c + d ,  OS = d + a
2 2
) ( )
   1     1  
Now, P   Q= OQ − OP=
2
b+c −a −b =
2
(
c−a ) ( )
   1     1  
and  SR= OR − OS=
2
c + d − d − a=
2
( c−a ) ( )
 
∴P  Q= SR
Thus, PQ = SR and PQ  SR i.e., two opposite sides of PQRS are equal and
parallel. Hence PQRS is a parallelogram.

Q. A rigid body rotates about an axis through the origin with an angular velocity

10 3 m/s. If ω points in the direction with position vector (iˆ + ˆj + k)
ˆ , then find

ω.

ˆi + ˆj + kˆ  
Sol. ˆ=
n
3
ˆ
⇒ ω = | ω| n

Vector & Three-Dimensional Geometry


 ˆi + ˆj + kˆ 
= (10 3 )  
 3 

= 10( ˆi + ˆj + kˆ )

9.
Q. Find the number of distinct unit vectors in space perpendicular to a given
plane.

Sol. There will be two distinct unit

vectors in space perpendicular

to a given plane

Q. How many unit vectors to a given line in space are possible ?

Sol. There will be infinite directions

perpendicular to a given line.

So infinite unit vectors are possible

Q. ABCD is parallelogram whose diagonals meet at P. If O is a fixed point and


    
OA + OB + OC + OD = µOP find m.

Sol.
Since, P bisects both the diagonals AC and BD, so
     
∴ OA= + OC 2OP and OB = + OD 2OP
(P is the mid point of AC & BD)
Vector & Three-Dimensional Geometry

    


OA + OC + OB + OD = 4OP

Q. If mid points of BC, CA, AB are D, E, F then find the position vector of centroid
of triangle DEF. Given that when position vector of A, B, C are
ˆi + ˆj, ˆj + kˆ and kˆ + ˆi respectively.

10.
Sol. centroid of ∆ABC & ∆DEF are same
∴ position vector of centroid of
1
 + (k + i)]
∆DEF = [(I + j) + (j + k)
3
2   
= (I + j + k)
3

Q. Three co-initial vectors of magnitade a, 2a, 3a meet at a point and their direc-
tions are along the diagonals of 3 adjacent faces of a cube. Determine their
resultant.

Sol. Consider a unit cube whose one vertex at the origin and three coterminous
edges OA, OB and OC along the coordinate exes OX, OY and OZ respectively.
  
Then,= OB ˆj and OC = kˆ .
OA ˆi,=
Let OD, OE and OF be the diagonals of three adjacent faces of the cube passing
through O along which act the vectors of magnitude a, 2a and 3a respectively.
We have
    
OD = OB + BD = OB + OA = ˆj + ˆi

∴ | OD |= 1+ 1 = 2
Thus, the unit vector
 1 ˆ ˆ
along OD is (i + j)
2
Similarly, unit vectors
  1 ˆ ˆ
along OE and OF are ( j + k) and
2
(iˆ + k)
ˆ
Vector & Three-Dimensional Geometry
respectively.
2
A vector of magnitude ‘a’ along OD is given by
 1 ˆ ˆ a ˆ ˆ
r1 = a × (i + j) = (i + j)
2 2
Similarly, a vector of magnitude 2a and 3a along OE and OF are given by
 1 ˆ ˆ 2a ˆ ˆ
r2 =2a × ( j +=
k) ( j + k)
2 2
 3a ˆ ˆ
=r3 (i + k) respectively.
2

11.
   
Let r be the resultant of r1 ,r2 ,r3 .
Then,
   
r = r1 + r2 + r3
 a ˆ ˆ 2a ˆ ˆ 3a ˆ ˆ a
r
⇒ = (i + j) + ( j + k) + (k +=
i) (4iˆ + 3ˆj + 5k)
ˆ ,
2 2 2 2
 a
|r|
∴= 16 + 9 +=
25 5a
2

Q. Centre of a Room(cube) is joined to 8 corners of cube Room(cube). 3 vectors


out of thus formed 8 vectors are selected at random. Find probability that
selected vectors are coplanar.

Sol. Total ways to select 3 vectors out of 8 = 8C3. Favourable number of ways

= (4C3) × 2
4
C3 × 2 4 × 2 1
=p = =
8
C3 8×7 7

Q. ABCDE is regular pentagon.


   
AB + AE + BC + CD + DE =
λAE
Then find the value of l ?

   


Sol. AB + AE + BC + CD + DE =
    
λAE

(AB + BC) + CD + DE + AE = λAE
    
(AC + CD) + DE + AE = λAE
   
Vector & Three-Dimensional Geometry

(AD + DE) + AE = λAE


  
AE + AE = λAE
λ=2

 
Q. In triangle ABC, AB =ˆi + 2ˆj + 3kˆ and BC = 4i + 5j + 6k then find the length of
3rd side and median?

   


Sol. In ∆ABC AB + BC + CA = 0

12.
 
 + (4I + 5j + 6k)
(I + 2j + 3k)  + CA = 0
 
(5I + 7j + 9k)  + CA = 0

CA = − (5iˆ + 7ˆj + 9k)ˆ

| CA |= 25 + 49 + 81 = 155
  
BA + BC = 2BD
 1
BD=  = 1 (3iˆ + 3ˆj + 3k)
( −I − 2j − 3k + 4i + 5j + 6k) ˆ
2 2
 3 2 3 3
| BD=| 1 + 12 + 1= 2
2 2

Q. In quadrilateral ABCD prove that figure formed by joining the mid-points of


consecutive sides is a parallelogram.

Sol.
Let ABCD be a quadrilateral & p, q, r, s are
midpoints of sides as shown-
 
 c + b − (a + b) c − a
=PQ =
2 2
   
 d + c − (d + a) c − a

=SR =
2 2
 
∴ PQ = SR ⇒ PQ = SR and PQ || SR
⇒ PQRS is a parallelogram

     
Q. In triangle ABC, AB = x and BC = y then find AM in terms of x and y , where

M divides BC in 1 : 3.

 y Vector & Three-Dimensional Geometry


Sol. BM =
4
   
AB + BM + MA = 0
  
 y
x + = –MA = AM
4
  y
∴ AM = x+
4

13.
RELATION BETWEEN TWO PARALLEL VECTORS :
 
1. I f a and b be two parallel vectors, then there Point to Remember!!!
 
exists a non-zero scalar k such that a = kb i.e.

there exist two non-zero scalar quantities x If a = a 1i + a2 j + a3 k and
  
and y so that xa + yb =
0 b = b1i + b2 j + b3 k then from the
2. I f a and be two non-zero non-parallel vectors property of parallel vector, we
 
then xa + yb = 0 ⇒ x = 0 and y = 0 have a  b
 
=a 0,b
= 0 a1 a2 a3
 ⇒ = =
  or   b1  b2  b3
  
3. If xa + yb =0 ⇒ = x 0,= y 0
  or 
  
 a  b

 
Q. The value of λ when a = 2i − 3j + k and b= 8i + λj + 4k are parallel is
(A) 4 (B) – 6 (C) – 12 (D) 1

Sol. (C)
  a a a
Since a  b ⇒ 1 = 2 = 3
bl b2 b3
2 3 1
∴ = − = ⇒ λ = −12
8 λ 4
 
Q. Find λ if a = 2i − 3j + k and b= 8i + λj + 4k are parallel.

 
Sol. a = µb
Vector & Three-Dimensional Geometry

(2I − 3j + k)
 = µ(8I + λj + 4k)

2I − 3j + k = 8µI + µλj + 4k)


Comparing, 2 = 8µ, –3 = µλ, 1 = 4µ


1 1
µ= , −3
= ( λ)
4 4
⇒ λ = – 12

14.
B ≡ (pi + qj) and C ≡ (i − j) are collinear, find p and q.
Q. If A ≡ (2i + 3 j),


AB = (p − 2)iˆ + (q − 3) j
Sol. 
AC =−i − 4 j
 
For collinearity, AB = λ(AC)
(p–2)i + (q − 3) j =λ( −i − 4 j)
p–2=–λ q – 3 = – 4λ
p = 2 – λ, q = 3 – 4λ (Infinite values of p & q)

Linear combination :

A vector r is said to be a linear Point to Remember!!!
  
combination of vectors a 1 , a2 … , an if there
(i) A pair of non-collinear vector
exist scalars m1 ,m
  2 , … ,mn such that
    is linearly independent.
 
=r m1 a 1 + m2 a2 + … + mn an . If a,b non zero non-collinear
  
Linearly Independent : vectors such that xa + yb =0
   ⇒ x = y = 0
A system of vectors a 1 , a2 , … , an is said to be 
Vector a and b are called as
linearly
 independent
 if   base vector.
m1 a 1 + m2 a2 + …… + mn an = 0
(ii) A triad (set of three) of non-
⇒ m1 = m2 = …… = mn = 0 coplanar vector is linearly
independent.

Q. Can ˆi − ˆj and 2iˆ − 2ˆj be treated as the base vectors in xy-plane ?

Vector & Three-Dimensional Geometry


Sol. ( ˆi − ˆj ) and ( 2iˆ − 2ˆj ) are collinear vectors, so they cannot be treated as base
vectors.

Q. Can ˆi − ˆj and i + ˆj be treated as the base vectors in xy-plane ?


Sol. x(I − j) + y(i + j) =
0

(x + y)I + ( −x + y)j =0

15.
⇒ x+y = 0 &
+ –x + y =0

=y 0=
&x 0
As x = 0 = y so given vectors can be treated as base vector

Result - 1: 
If a and b are two non-collinear vectors,
  
then every vector r coplanar with a and b
Know the facts
can be expressed in one and only one way as
a linear combination xa + yb ; x and y being 
  Any vector r in xy plane can be
scalars. Here a and b are termed as base 
r xi + yj .
taken as =
vectors.

GEOMETRICAL RESULTS WITH VECTORS:


(i) 
Straight line joining the mid points of two
non parallel sides of a trapezium is parallel
to the parallel sides and half of their sum.
Point to Remember!!!
(ii) Four diagonals of any parallelopiped (A prism
whose base is a||gm) and the join of the mid
Tetrahedron (a pyramid on a
point of each pair of opposite edges are
triangular base)
concurrent and are bisected at the point
(a) 
Lines joining the vertices of a
concurrency. This point is called the centre
tetrahedron to the centroids of
of the parallelopiped with position vector
      the opposite faces are
a +b+c OA + OB + OC concurrent and this point (P) of
i.e.
2 2 concurrency
   with
 position vector
Vector & Three-Dimensional Geometry

 a +b+c+d
g= is called the
4
centre of the tetrahedron (say
G).
(b) 
In a tetrahedron straight lines
joining the mid points of each
pair of opposite edges are also
concurrent at the centre of the
tetrahedron.

16.
 
Q. Centre of the parallelopiped formed by PA =+ 
i 2 j + 2k,PB =(4ˆi , −3ˆj, 1kˆ ) ,

PC (3iˆ, 5ˆj, −1kˆ ) is (7, 6, 2). Find position vector of ‘P’.
=

Sol.
P(α, β, γ)
A(1 + α, 2 + β, 2 + γ)
B(4 + α, –3 + β, 1 + γ)
C(3 + α, 5 + β, –1 + γ)
D(7 + α, 2 + β, γ)
F(5 + α, –1 + β, 3 + γ)
G(8 + α, 4 + β, 2 + γ)
E(4 + α, 7 + β, 1 + γ)
 32 + 8α 16 + 8β 8 + 8γ 
Average  , , = (7, 6, 2) ⇒ α= 3, β= 4, γ= 1
 8 8 8 

Q. The diagonals of the three faces of the parallelopiped drawn from the same
vertex are prolonged half their lengths. Show that the three points thus ob-
tained are coplanar with the opposite vertex.

   
Sol. Diagonal vector is (a + b + c) =
   c a
d

d−P = b − −
2 2

   a b
d − Q =c − −
2 2

   b c
d−R = a − −
2 2

 c a     
 a b  b c
b − − = x c − −  + y a − − 
2 2  2 2   2 2 
 

Vector & Three-Dimensional Geometry


x y −1 y −1 x
1= − − x−
= = +y
2 2 2 2 2 −2
y−1 x 1
x + y =−2 − 0 ...(i) x = ...(ii) y = − ...(iii)
2 2 2
y−1 −1 − 1
+ y =−2 x=
2 2
y – 1 + 2y = – 4 x=–1
3y = – 3
y=–1
Values of x & y from (i) & (ii) satisfy (iii)

17.
Hence three points thus obtained are co-
planar with the opposite vertex 1

VECTOR EQUATION OF A STRAIGHT LINE:


Vector equation of a straight line passing through

( )
a given point A a and parallel to a given vector

b:
Let O be the origin. Let the line pass through

a given point A whose position vector is a ,
 
then OA = a .

Let the given line be parallel to vector b

Let r be the position vector of any point P
on the
line, then

OP = r
   
Since AP is parallel to b  ∴  AP = tb , where
t is a scalar.
     
Now  O =P OA + AP  ∴  r =a + tb … (i)
Since for different values of t, we get different
positions of point P on the line, hence
(i) is the vector equation of the required
straight line.

Vector equation of straight line


 passing through

( )
two given point A a and B b :() Point to Remember!!!
Let O be the origin. Let the line pass through
two given point A and B whose  position (i) T
 wo lines in a plane are

vectors referred to O be a and b respectively, either intersecting or parallel
Vector & Three-Dimensional Geometry

   
then OA = a and OB = b conversely two intersecting or
     parallel lines must be in the
∴  AB =OB − OA =b − a same plane
Clearly, the required line passes through (ii) 
However in space we can
 

( ) (
A a and is parallel to the vector b − a . ) have two neither parallel nor
intersecting lines. Such non
Hence the vector equation of the required
coplanar lines are known as
line is,
skew lines. If two lines are
  
 
r = ( ) 
(

)
a + t b − a   or  r =1 − t a + tb parallel and have a common
point then they are coincident.

18.
Q. Find the p.v. of the point of intersection of the lines

r = i − j − 10k + λ(2i − 3j + 8k) 
(i) and  
r= 4i − 3j − k + µ(i − 4j + 7k)  


r = −3i + 6j + λ( −4i − 3j + 2k)  
(ii) and  
r = −2i − 7k + µ( −4i − j + k) 


r= t(3i − j + k)
 
(iii) and  
r =−2i − s( −6i − 2j − 2k) 


r 2k + λ(3i + 2j + k)
=  
(iv) and  
r = −3i − 2j + 3k + µ(6i + 4j + 2k)  


Sol. (i) L 1 : r = (2λ + 1)iˆ − (1 + 3λ )ˆj + (8λ − 10)k

L2 : r= (4 + µ )iˆ − (4µ + 3)ˆj + (7µ − 1)kˆ
The given lines are not parallel. For coplanarity, the lines must intersect.
∴ (2λ + 1)iˆ − (1 + 3λ )ˆj + (8λ − 10)kˆ= (4 + µ )iˆ − (4µ + 3)ˆj + (7µ − 1)kˆ
2λ + 1 = 4 + µ …(i)
1 + 3λ = 4µ + 3 …(ii)
8λ – 10 = 7µ – 1 …(iii)
Solving (i) & (ii), λ = 2, µ = 1 and λ = 2, µ = 1 satisfies equation (iii) Given lines
are intersecting & hence coplanar.

put µ = 1 in L2 : r = 5iˆ − 7ˆj + 6kˆ

(ii) −3i + 6 j − 4 λi − 3λ j + 2λk =−2i − 7k + ( −4µ )i − µ j + µk


– 3 – 4λ = – 2 – 4µ
4(µ – λ) = 1
1
µ= + λ …(i)

Vector & Three-Dimensional Geometry


4
6 – 3λ = – µ …(ii)
1
6 – 3λ = – –λ
4
1

6 + =2λ
4
25 1 25 27
λ= µ= + =
8 4 8 8
Also
2λ = – 7 + µ …(iii)
values of λ & µ from (i) & (ii) does not satisfy (iii)
Hence they are not intersecting

19.
(iii) 3ti − t j + tk = −2i + 6si + 2sjˆ + 2sk
ˆ

3t = (–2 + 6s)…(i) – t = 2s …(ii) t = 2s …(iii)

3(–2s) = – 2 + 6s

2 = 12s
1  1

s = ∴ t = −2  
6 6
1
t= −
3
value of s & t from (i) & (ii) does not satisfy (iii)
So lines are not coplanar
(iv) lines are parallel

Q. Find the vector equation of the line through the point 2i + j − 3k and parallel to
the vector ˆi + 2ˆj + kˆ.

 
Sol. Let the given point be A ( a )
and given vector be b and O be the origin.
  
Then,  a = OA = 2iˆ + ˆj − 3k  ˆ and  b =ˆi + 2ˆj + kˆ

Now vector equation of the line through A and parallel to b is
  
r= a + tb , where t is a scalar.


or ; r = 2iˆ + ˆj − 3kˆ + t ˆi + 2ˆj + kˆ ( )
Q. In a triangle ABC, D divides BC in the ratio 3 : 2 and E divides CA in the ratio
1 : 3. The lines AD and BE meet at H and CH meets AB in F.
Find the ratio in which F divides AB.
Vector & Three-Dimensional Geometry

Sol. Take C as the origin of reference.


 
Let- a, b be the position vectors of the points A
and B respectively. The equation of AD and BE
are
  2      1  
r = a + k  b − a  and r =
b + t  a − b
5  4 
respectively. For the point of intersection H, we
have
 2    1  
a + k  b − a  =b + t  a − b  …(i)
5  4 

20.
t 2 5 2
⇒ 1 − k = , k = 1− t ⇒ k = t =
4 5 6; 3
On putting these values of k and t in the equations of the lines AD and BE, we
 
1 1  a b
see that the point H is a + b. The equations of CH and AB being =r t + 
6 3 6 3
 
  
and r = a + k(b − a) respectively.
 
a b    
For the point of intersection F, t  +  = a + k(b - a)
6 3
 
t  t 
⇒  − 1 + k
=  0,  − k
=  0
 6   3 
2
These give t = 2; k=
3
 
a + 2b
Thus, the point F is
3
So, that F divides AB in the ratio 2 : 1

Vector equation of bisectors of angle between


two straight lines :
Let OA and OB be the given straight line
parallel to unit vectors â and b̂ respectively.
Take the point O as origin, and let Q be a
point on the internal bisector of the angle
AOB. From Q draw QR parallel to OA cutting t
OB at R.
Now  ∴ ∠AOQ = ∠BOQ
(as OQ is the bisector)
and ∠BOQ =
∠OQP (alternative angles)
∴  ∠AOQ =∠OQP ∴ OP = t (s ay) ,
= QP Vector & Three-Dimensional Geometry
where t is a scalar.
 
∴  OP = ˆ and P
ta  ˆ
  Q = tb
 
OQ = r
  
Let  O= Q OP + PQ

or  r
= ta ˆ + tb

r t a
= (ˆ
ˆ +b )

21.
 
 a b
=r t + 
a b
  Point to Remember!!!
 
where= a a,=b b
Equation of external Bisector:
This is the equation of internal bisector of If OP’ be the external bisector of
∠AOB . ∠OAB , then OP’ may be regarded
as the internal bisector of the angle
Corollary: between the lines which are parallel
If the lines intersect at E having position to â and −b̂ Hence its equation is
  
vector α , then the above equations becomes a b
 
r = α+t a (
ˆ +bˆ
) and
 
r = α+t a ˆ
ˆ −b ( )

r=t a ( ˆ
) 
ˆ − b   or,  r =−
t
 a b 
 
respectively.

Q. Use vectors to prove that the internal (external) bisectors of a triangle divide
the opposite base internally (externally )in the ratio of the side containing the
angle.

Sol. Let’s suppose ∠BAD = θ , where θ is acute,


∠CAD =
and
 
| BD |:| CD | = r : s . Using the section formula,
 r  s 
=AD AC + AB.
r+s r+s
Then by dot product,
    r  s  
=AD·AC | AD || = AC | cos θ | AC |2 + AB·AC
r+s r+s
 r  s   
⇒| AD = | cos θ | AC | + | AB | cos 2θ Similarly, by considering AD · AB , we
r+s r+s
Vector & Three-Dimensional Geometry

have
 r  s 
=| AD | cos θ | AC | cos 2θ + | AB |
r+s r+s
Combining 1 and 2 gives
r  s  r  s 
| AC | + = | AB | cos 2θ | AC | cos 2θ + | AB |
r+s r+s r+s r+s
r  s 
| AC | (1 − cos = 2θ) | AB | (1 − cos 2θ)
r+s r+s
 
AB r | BD |
 = =  .
AC s | CD |

22.
Q. Prove, by vector method that the internal bisectors of the angles of a triangle
are concurrent.

  
Sol. Let α, β, γ be the position vectors of
vertices A, B, C respectively of ∆ABC.
Let AL be the bisectors of ∠BAC
BL BA c
Then = =
LC AC b
Thus L divides BC internally in the
ratio c : b 

bβ + cγ
P.V. of L =
b+c
∴   Now P.V. of the point which divide AL internally in the ratio b + c : a will be
  
aα + bβ + cγ

a +b+c
Here we have divided AL in the ratio b + c : a because b + c occurs in the

denominator of P.V. of L and there is bβ and cγ therefore, there should also be


Similarly, we can show that the position vectors of the points which divide
bisectors BM of ∠ABC and CN of ∠ACB in the ratio c + a : b and a + b : c
  
aα + bβ + cγ
respectively will be each . Thus the point having position vector
a +b+c
  
aα + bβ + cγ
lies on the three internal bisectors of the angles of the triangle
a +b+c
ABC and hence internal bisectors of angles of a triangle are concurrent and the
  
aα + bβ + cγ
position vector of incentre of ∆ABC will be .
a +b+c
Vector & Three-Dimensional Geometry
COLLINEARITY OF THREE POINTS :
  
1. If a,b, c be position vectors of three points
A, B and C respectively and x, y, z be three
scalars so that all are not zero, then the
necessary and sufficient conditions for three
points to be collinear is that
  
xa + yb + zc =0  where x + y + z =0
2. 
Three
 points

 A, B and C are collinear, if
AB = λBC .

23.
    
Q. If 2a − 3b,b and a − b are position vectors of three points A, B and C then they
are
(A) Collinear (B) Non- collinear
(C) Can’t say anything (D) None of these

Sol. (A)    


( ) () (
 1 2a − 3b + 1 b − 2 a − b = )
0 and 1 + 1 − 2 =0

So the given vectors are collinear.

Q. ( ) ( ) ( )
If A ≡ 2iˆ + 3ˆj ,B ≡ piˆ + 9ˆj and C ≡ ˆi − ˆj are collinear, then the value

of p is
(A) 1 / 2 (B) 3 / 2 (C) 7 / 2 (D) 5 / 2

Sol. (C)
 
( )
 AB = p − 2 ˆi + 6ˆj, AC =−ˆi − 4ˆj
  p−2 6
Now A,B,C are collinear ⇔ AB  AC ⇔ = ⇔ p= 7 / 2
−1 −4

Q. Find whether the following points are collinear or not


(i) 2i + 5j − 4k;i
  + 4j − 3k;
 4i + 7j − 6k

(ii) 3iˆ − 4ˆj + 3k;
ˆ −4iˆ + 5ˆj − 6k;
ˆ 4iˆ − 7ˆj + 6kˆ

(i) (2iˆ + 5 j − 4k ) − ( ˆi + 4 j − 3k ) = λ(4ˆi + 7 j − 6k − ( ˆi + 4 j − 3k ))


   
Sol.
ˆi + ˆj − kˆ =λ(3iˆ + 3ˆj − 3kˆ ) 1
1 = 3λ ⇒ λ = ⇒ collinear
3
(ii) [(3iˆ − 4 j + 3k ) − ( −4ˆi + 5 j − 6k )] =λ(4ˆi − 7 j + 6k − ( −4ˆi + 5 j − 6k )]
Vector & Three-Dimensional Geometry


(7 ˆi − 9 j + 9k ) =
λ(8iˆ − 12 j + 12k )

Value of λ can’t be calculated uniquely ⇒ not colinear

      
Q. ( )
Given sin2 θ a + (2 sin θ)b + c =0 if A(a),B(b), c(c) are collinear. Find q.

Sol.
sin2θ + 2sinθ + 1 = 0
(sinθ + 1)2 = 0 ⇒ sinθ = – 1
 π
θ = nπ + ( −1)n  −  , n ∈ z
 2

24.
  
Q.
Vectors P, Q act at ‘O’ (origin) have a resultant R . If any transversal cuts their
P Q R
line of action at A, B, C respectively, then show that + =.
OA OB OC
  
Sol.
∴ P+Q= R …(1)
    
Again let =P OL= mOA,= Q OM= nOB
 
and= R ON = t · OC
P Q R
∴ m= ;n= ; t= …(2)
OA OB OC
Hence from (1), we get
  
m·OA + n·OB = t·OC
  
or m · OA + n · OB − t · OC = 0 …(3)
But A,B,C are collinear and we know that if there exists a relation of the form
xa + yb + zc = 0 between the P.V’s of three collinear points then x + y + z = 0.
Hence from (3) we must have
m + n – t = 0 ∴ m + n = t.
P Q R
Now putting the value of m, n and t from (2) in m + n = t we get + =.
OA QB OC
PRODUCT OF VECTORS:
Product of two vectors is done by two
methods when the product of two vectors
results in a scalar quantity then it is called
scalar product. It is also called as dot
product because this product is represented
by putting a dot.
When the product of two vectors results in
a vector quantity then this product is called
Vector Product. This product is represented Vector & Three-Dimensional Geometry
by (x) sign so that it is also called as cross
product.

Scalar or dot product of two vectors:


Definition :

If a and b are two vectors and θ be the
angle between their tails or heads, then their
scalar product (or dot product) is defined
 as
  
the number a b cosθ where a and b are

25.
   
moduli of a and b respectively and 0 ≤ θ ≤ π. It is denoted by a ⋅ b .
   
Thus =
a ⋅ b a b cosθ

Note: 

(i)  a ⋅ b ∈ R
   
(ii)  a ⋅ b ≤ a b
     π
(iii)  a ⋅ b > 0 ⇒ angle between a and b say θ ∈ 0,  .
 2
    π 
a ⋅ b < 0 ⇒ angle between a and b say θ ∈  , π 
2 
    π 
 a ⋅ b = 0 ⇒ angle between a and b say θ = or atleast one of a and
2

b is zero vector.
(iv) The dot product of a zero and non-zero vector is a scalar zero i.e.

0⋅a = 0.
 
(v) If θ be angle between any two non zero vector a and b then
 
a ⋅b
cosθ =   .
a ⋅b

Geometrical Interpretation:
Geometrically, the scalar product of two
vectors is equal to the product of the
magnitude of one and the projection of
second in the direction of first vector i.e.
    
= ( ) 
 a ⋅ b a b cosθ = a (projection of b in

the direction of a)
   
( )
Vector & Three-Dimensional Geometry

Similarly = a ⋅ b b   a cosθ
  
= b (projection of a in the direction of b )
  
 a ⋅b
Here projection of b on a = 
a

  a ⋅ b
Projection of a on b = 
b

26.
 
Q. If angle between a and b is 120 and their magnitudes are respectively 2 and
 
3 , then a ⋅ b equals
(A) 3 (B) − 3 (C) 3 (D) – 3

Sol. (B)    
We know that =
a ⋅b a b cosθ

= 2 3cos120

(
2 3 −1 / 2 =
= − 3 )

Q. The projection of vector ˆi + ˆj + kˆ on the vector ˆi − ˆj + kˆ is


(A) 3 (B) 1 / 3 (C) 2 / 3 (D) 2 3

Sol. (B)

Projection
=
(ˆi + ˆj + kˆ ) ⋅ (ˆ=
i − ˆj + kˆ ) 1− 1+ 1
=
1
ˆi − ˆj + kˆ 1+ 1+ 1 3

Scalar product in particular cases :


   
1.  a ⋅ b = b ⋅ a
      
( )
2.  a ⋅ b + c = a ⋅ b + a ⋅ c
   
( ) ( 
3. ma ⋅ b = m a ⋅ b = a ⋅ mb) ( )
   
4. If θ= 0 ⇒ a ⋅ b= a b (like vectors)
   
5. If θ = π ⇒ a ⋅ b = − a b (unlike vectors)
ˆ= cos θ
Vector & Three-Dimensional Geometry
6. If â and b̂ are unit vectors then â ⋅ b
(where θ is angle between them).
     
7. a ⋅ a = | a |2 ⇒ a = a ⋅a
       
8. If a ^ b ⇒ a ⋅ b = 0 but a ⋅ b = 0 ⇒ a = 0 or
   
b = 0 or a ^ b .
9. If î , ĵ and k̂ are unit vectors along the
rectangular coordinate OX, OY and OZ then
ˆi ⋅ ˆi = ˆj ⋅ ˆj = kˆ ⋅ kˆ = 1, ˆi ⋅ ˆj = ˆj ⋅ kˆ = kˆ ⋅ ˆi = 0

27.
     
10. a ⋅ b = a ⋅ c ⇒ b = c Point to Remember!!!
      
 ( 
)
Infact a ⋅ b = a ⋅ c ⇒ a ⋅ b − c = 0 ⇒ a = 0 or
  
    
b = c or a ^ b − c( ) ( )
(a) a ⋅ b ⋅ b is not defined

     2   2
( )
11.   a ⋅ b ⋅ c is meaningless. (b) (a + b)2= a + 2a ⋅ b + b
  2 2   2
(c) (a =
− b) a − 2a ⋅ b + b
    2 2
( )(
(d) a + b ⋅ a − b = )
a −b
     
(e) a + b = a + b ⇒ a  b
 2 2 2  
(f) a + b = a + b ⇒ a ^ b
     
(g) a + b = a − b ⇒ a ^ b

Q. If the sum of two unit vectors is a unit vector then find the magnitude of their
difference and the angle between â and b̂ .

Sol. | aˆ + bˆ | =1 ⇒ | aˆ + bˆ | =1
2

|a ˆ |2 + 2a·b
ˆ |2 + | b ˆˆ =1

1+ 1+ 2|a ˆ | cos θ =1
ˆ || b

= (θ Angle between a ˆ
ˆ + b)
1 + 2(1) (1) cosθ = 0
1 2π
cos θ = − ⇒ θ = .
2 3
Vector & Three-Dimensional Geometry

ˆ|
ˆ −=
|a b |a ˆ |2 −2a·b
ˆ |2 + | b ˆˆ

= (1)2 + (1)2 − 2 | a ˆ | cos θ


ˆ || b

= 1 + 1 − (1)(1)( −1)

= 1+ 1+ 1= 3 units.

Q. Determine the values of c such that for all x (real) the vectors cxˆi − 6ˆj + 3kˆ
and xˆi + 2ˆj + 2cxkˆ make an obtuse angle with each other.

28.
Sol. If θ is the angle between the given vectors then
cx2 − 12 + 6cx
cosθ =
c2 x2 + 45 x2 + 4c2 x2 + 4
If θ is obtuse then cosθ < 0 ⇒  cx2 + 6cx − 12 < 0∀x ∈ R
Which is possible if c < 0 and 36c2 + 48c < 0 ⇒ c < 0 and 12c 3c + 4 < 0 ( )
⇒ c < 0 and −
4
3
< c < 0 (but for=c 0, cx2 + 6cx − 12 < 0 ∀x ∈ R )
4
Hence − <c≤0
3

     
Q. If a,b, c are unit vectors such that a is perpendicular to the plane of b and c
    
then find a + b + c when the angle between b and c is π / 3 .

         π 1
Sol.
We have a = b = c = 1, a ⋅ b = a ⋅ c = 0 and  b=
⋅ c cos=
3 2
               
Now | a + b + c |2 = ( )( )  
(
a + b + c ⋅ a + b + c = | a |2 + | b |2 + | c |2 +2b ⋅ c  a ⋅ c = a ⋅ b = 0 )
1   
= 1+ 1+ 1+ 2⋅ = 4 ⇒ a +b+c = 2
2

Scalar product in terms of components:



Let a and b be two vectors such that
 
a = a 1ˆi + a2ˆj + a3kˆ and b = b1ˆi + b2ˆj + b3kˆ
 
Then  a ⋅ b= a 1b1 + a2b2 + a3b3
In particular
  
a ⋅ a = | a |2 = a21 + a22 + a23

For any vector a
( ) ( ) ( ) Vector & Three-Dimensional Geometry
   
a = a ⋅ ˆi ˆi + a ⋅ ˆj ˆj + a ⋅ kˆ kˆ

  
ˆ then find a ⋅ b .
If a = 3iˆ + 2ˆj + kˆ and b =ˆi − 2ˆj + 5k 
Q.
 
Sol. ( )( ) ( )( ) ( )( )
 a ⋅ b = 3 1 + 2 −2 + 1 5 = 3 − 4 + 5 = 4

29.
Q. Prove that the angle in a semi circle is a right angle.

Sol. Let O be the centre of the semi circle with AOB


as its diameter. Let P be a point on the semi-
circle, so that ∠APB is an angle in the semi
circle. Join OP. Let O be taken as origin. Let the
  
position vectors of A, B and P be α, −α and r
respectively.
Clearly, OA = OB = OP
     
(
Now  AP= r − α and BP= r + α ) ( )
     
( ) ( )
∴ AP ⋅ BP= r − α ⋅ r + α = r2 − α2= OP2 − OA2= 0

 OP = OA 

∴  AP ^ BP i.e. ∠APB =


90.

Q. Prove by vector method the following formula of plane trigonometry


(
cos α − β )
= cosαcosβ + sinαsinβ

Sol. Let the unit vectors along OX and OY be î and


ĵ respectively. If OA and OB be any two lines in
the same plane making angles α and β re-
spectively, with OX then, ∠AOB = α − β
 
Again, let OP and OQ represent unit vectors
along OA and OB respectively, so that their dot
product is the cosine of the angle between
Vector & Three-Dimensional Geometry

their directions.
 
Now, OP ⋅ OQ = 1 ⋅ 1cos(α − β) = cos(α − β) …(1)

Since OP makes an angle α with x-axis.

∴ OP = cos αˆi + sin αˆj

Similarly, OQ = cos βˆi + sin βˆj
 
∴  OP ⋅ OQ = cosαˆi + sinαˆj ⋅ cosβˆi + sinβˆ=
  ( )
j cosα ⋅ cosβ + sinα ⋅ sinβ … 2

( )
From (1) and (2), we get cos α − β= cosα ⋅ cosβ + sinα ⋅ sinβ

30.
a2 + b2 − c2
Q. In any ∆ABC , prove that cosC =
2ab
with the help of vectors.

   


Sol. In ∆ABC, BC + CA + AB =
  
0
⇒  AB = (
− BC + CA )
     
(
⇒  AB ⋅ AB = BC + CA ⋅ BC + CA)( )
   

 
( )(
⇒  c ⋅ c = a + b ⋅ a + b )
 
⇒  c2 = a2 + b2 + 2a ⋅ b
2
⇒ c= a2 + b2 + 2ab cos( π − C)
⇒  c2 = a2 + b2 − 2abcosC B
C

Q. (
Prove by vector method that a 1b1 + a2b2 + a3b3 )
2
( )(
≤ aI2 + a22 + a23 b21 + b22 + b23 )
 
Let a = a 1i + a2 j + a3 k and b = b1 i + b2 j + b3 k
Sol.
 
Then  a ⋅ b= a 1b1 + a2b2 + a3b3 ...(1)
 
Also a = a21 + a22 + a23 and b = b21 + b22 + b23   …(2)
 
If θ be the angle between the vectors a and b , then
2
   
a ⋅b 2  a ⋅ b 
=cosθ    cos θ ≤ 1 ∴     ≤ 1
a b a b
 
  2  2  2
or (a ⋅ b) ≤| a | | b |

∴ (a1b1 + a2b2 + a3b3 )


2
( )(
≤ aI2 + a22 + a23 b21 + b22 + b23 )
Vector & Three-Dimensional Geometry
Angle between two vectors in terms of
components :
 
If a and b be two vectors such that Point to Remember!!!

a = a ˆi + a ˆj + a kˆ and b = b ˆi + b
1 2 3 1 2
ˆj + b kˆ and
3
 
θ be the angle between them, then If a and b are perpendicular to
a 1b1 + a2b2 + a3b3 each other then
cosθ = a 1b1 + a2b2 + a3b3 =
0
a21 + a22 + a23 b21 + b22 + b23

31.
Q. Find the angle between the vectors 4iˆ + ˆj + 3kˆ and 2iˆ + 2ˆj − kˆ .

Sol. Let the required angle is θ .


= ∴ θ cos
=
 4.2 + 1.2 + 3 −1
−1 
 
 cos−1  7 
 ( )
   3 26 
 16 + 1 + 9 4 + 4 + 1   

 
Components of b along and perpendicular to a :
  
1. Component of b along a = OM

= 
OMa
= bcosθ a ( )
 
=
(
abcosθ 
= a
) (a ⋅ b) a = ( a ⋅ b ) a
a a2 | a |2
 
2. Component perpendicular to a = ON
  
= b ON + OM
  
∴ ON = b − OM
 

 
ON= b −
(
a ⋅b 
a
)
a2

Q. Find the vector components of a vector 2iˆ + 3ˆj + 6kˆ along and perpendicular to
non-zero vector 2iˆ + ˆj + 2kˆ .

 
Sol. Let a = 2iˆ + 3ˆj + 6kˆ and b = 2iˆ + ˆj + 2kˆ
 
Now vector component of a along b
 
a · b  4 + 3 + 12 ˆ ˆ ˆ 19 ˆ ˆ ˆ
=  = b (2i + j + 2k)
= (2i + j + 2k)
Vector & Three-Dimensional Geometry

|b| 2 9 9
 
and vector component of a perpendicular to b
 
 a ·b  ˆ − 19 (2iˆ + ˆj + 2k)
ˆ = 1 ( −20iˆ + 8ˆj + 16k)
= a −  2 b = (2iˆ + 3ˆj + 6k) ˆ
|b| 9 9

Q. ( )
Find the perpendicular distance of the point A 1, 0, 1 to the line through the

( )
points B 2, 3, 4 and C −1, 1, −2 . ( )

32.
   
r =BA =−ˆi − 3ˆj − 3kˆ and a =
Sol. BC = −3iˆ − 2ˆj − 6kˆ
  
Now BL = Projection vector of r on a

r·a  3 + 6 + 18
= = a ( −3iˆ − 2ˆj − 6k)
ˆ
| a |2 49
−27 ˆ
= (3i + 2ˆj + 6k)
ˆ
49
    
LA = LB + BA = BA − BL

 r·a  ˆ + 27 (3iˆ + 2ˆj + 6k)
ˆ = 1 (32iˆ − 93ˆj + 15k)
= r −  2 ·a = ( −ˆi − 3ˆj − 3k) ˆ
|a| 49 49
 9898
∴  LA = LA =
49

More Illustrations on Dot Product


 
If a =2i − 3j + 6k and b =−2i − 5j − 14k
Q.  
Projection of a on b
If λ =   . Find l.
Projection of b on a
ˆ  
a·b (a·b) |a| 4 + 9 + 36 7
Sol. =

ˆ
b·a
=
 (b·a) =
|b|
=
4 + 25 + 196 15
|b| 
(| a |)


Q. Express the vector a = 5i − 2j + 5k as the sum of two vectors such that one is
 
parallel to b= 3i + k and the other perpendicular to b .

Vector & Three-Dimensional Geometry



a = 5iˆ − 2ˆj + 5kˆ
Sol.

b 3iˆ + kˆ
=
  
Consider, a = β1 + β2
   
Where β1 is parallel to b and β2 is perpendicular to b
 
Since, β1 is parallel to b -
 
Then, β1 = λb = λ(3iˆ + k)
ˆ = 3λˆi + λkˆ …(1)
  
Now, a = β1 + β2

33.
  
β2 = a − β1

β=2
(5iˆ − 2ˆj + 5k)
ˆ − (3λˆi + λk)
ˆ

(5 − 3λ )iˆ − 2ˆj + (5 − λ )kˆ …(2)
β2=
 
Since, β2 is perpendicular to b
 
β2 .b = 0
Then, 3(5 – 3λ) + 0(–2) + (5 – λ) = 0
λ = 2
Therefore,

β1 = 6iˆ + 2kˆ from (1)

β2 =−ˆi − 2ˆj + 3kˆ …(3)

Hence, a= (6iˆ + 2k) ˆ + ( −ˆi − 2ˆj + 3k)
ˆ

  
 = i + 3j − 5k , find the angle between a and a + 2b
If a = 5i − j − 3k,b
Q. .

 
a + 2b= (5iˆ − j − 3k ) + 2( ˆi + 3 j − 5k ) = 7 ˆi + 5 j − 13k
Sol.   
(a + 2b)·a (7iˆ + 5ˆj − 13k)·(5i
ˆ ˆ − ˆj − 3k) 35 − 5 + 39 69 13
cos θ
= =
   = = =
| a || a + 2b | 25 + 1 + 9 49 + 25 + 169 35 243 359 3 3 105
 13 
θ =cos−1 
 3 105 
 

        π
Q. Find the angle between a + b and a − b if| a=| 2,| b=| 1 & a ∧ =
b
3
.

           
Vector & Three-Dimensional Geometry

Sol. If angle between (a + b) and (a − b) īs θ Then (a + b)·(a − b) = | (a + b) || (a − b) | cos θ


  
(a + b)·(a − b)
⇒ cos θ =    
| (a + b) || a − b |
  
a 2 − a · b + b · a − b2
⇒ cos θ =
a 2 + b2 + 2ab cos φ a 2 + b2 − 2ab cos φ

a2 − b2
⇒ cos θ =
a2 + b2 + 2ab cos φ a2 + b2 − 2ab cos φ

34.
22 − 12
⇒ cos θ =
π 2 π
22 + 12 + 2 × 2 × 1cos 2 + 12 − 2 × 2 × 1cos
3 3
3
⇒ cos θ =
7× 3
3
⇒ cos θ =
7
3
⇒ θ =cos −1
7
⇒ θ = 49.11°

     
Q. Find p ∧ q if 2p + q =i + j , p + 2q =−
i j .

   
Sol. (2p + q) + (p + 2q) = ˆi + ˆj + ˆi − ˆj
    2ˆ
⇒ 3(p + q) = 2iˆ ⇒ p + q = i …(1)
3
   
Also, (2p + q) − (p + 2q) = ˆi + ˆj − (iˆ − ˆj)
 
∴p − q = 2ˆj …(2)
From (1) and (2)
 2ˆ
2p
= i + 2ˆj …(3)
3
 2ˆ
2q
= i − 2ˆj …(4)
3
 22 4 32
∴ 4p·q = · + 2 · ( −2) = − 4 =−
33 9 9

Vector & Three-Dimensional Geometry


  8
∴ p · q = −
9
2
 2 2 2
Also from (3), 2 | p=|   + (2) = 10
3
  3
 1
∴| p | = 10, and from (4)
3
2
 2 2  1
2=
|q| 2)2
  + ( −= 10, ∴
=|q| 10
3 3 3

35.
 8

p·q 9 4
∴ cos θ =   = =−
| p || q | 1 1 5
10· 10
3 3
 4
∴ θ cos−1  − 
=
 5
   4
Hence p ∧
= q cos−1  − 
 5
     
Q. |=
a | 11,| =
b | 23 and | a − =
b | 30 find | a + b |

   
Sol. Given, | a=| 11,| b=| 23,| a − b=| 30
     
We know that, | a − b |2= | a |2 + | b |2 − 2 | a || b | cos θ
302 = 112 + 232 – 2 × 11 × 23 · cosθ
2 × 11 × 23 · cosθ = 112 + 232 – 302 = – 250
     
Now, | a + b |Ố= | a | + | b | +2 | a || b | cos θ = 112 + 232 – 250 = 400
 
| a + b=
| 400 = 20

      
Q. If a + b +=
c 0,| =
a | 3;| =
b | 1 and | c |= 4 . Find ∑ (ab)
        
Sol. We know, (a + b + c)2 = (a + b + c)·(a + b + c)
     
⇒ 0= | a |2 + | b |2 + | c |2 +2(a·b + b·c + c·a)
  
(Given a + b + c = 0)
  
⇒ 0= (3)2 + (1)2 + (4)2 + 2(a·b + b·c + c·a)
Vector & Three-Dimensional Geometry

  
(Given = | a | 3,|
= b | 1,|
= c | 4)
  
⇒ 0 = 9 + 1 + 16 + 2(a·b + b·c + c·a)
  
⇒ 0 = 26 + 2(a·b + b·c + c·a)
   26
⇒ a·b + b·c + c·a = − = −13
2
Hence the value asked in the question is
  
a·b + b·c + c·a = −13

36.

Q. A vector of magnitude 25 is collinear with a = 2i − 3j + 4k and makes obtuse
angle with negative z-axis find the vector.
  
Sol. | b | = 25 and b = λa

…(i)

b= λ(2I − 3j + 4k)




| b |=λ 4 + 9 + 16
25  25
⇒λ= So, b = (2I - 3 j + 4k )
29 29

ˆ=| θ ˆ=| θ


Q. ˆ +b
Prove that | a ˆ −b
2 + 2 cos θ= 2 cos   and | a
2
 
2 − 2 cos θ= 2 sin   ,
2
 
where θ is the angle between the vectors a & b .

ˆ |=
Sol. ˆ +b
Let | a k

ˆ |2 =
ˆ +b
⇒ |a k2
ˆ ˆ + b)
ˆ + b)·(a
⇒ (a ˆ =k2
⇒ 1 + 1 + 2cosθ = k2
θ
k
⇒ = 2 + 2 cos θ θ 2 cos2
 1 + cos =
2
ˆ=| θ
ˆ −b
Similarly | a 2 − 2 cos θ= 2 sin .
2


Q. Let u be a vector on rectangular coordinate system with sloping angle 60°.
  
Suppose that |u - ˆi| is geometric mean of u and |u – 2i|
ˆ where î is the unit

Vector & Three-Dimensional Geometry


vector along x-axis then | u | has the value equal to a − b where a, b ∈ N,
find the value (a + b)3 + (a – b)3.


Sol. Let use consider | u | = λ
 λ ˆ
⇒ u= (i + 3 ˆj)
2
 ˆ  
Also | u =
−i| | u || u − 2iˆ |
2
λ λ
⇒ (iˆ + 3 ˆj) − ˆi =
λ (iˆ + 3ˆj) − 2iˆ
2 2

37.
2
 λ 2
3λ2   2
3λ2 
  2  λ − 4 
⇒  − 1 + =
λ   +
 2  4   2  4 
   
⇒ (4λ2 – 4λ + 4)2 = 16λ2(λ2 – 2λ + 4)

( ) ( )
2
⇒ λ2 − λ + 1 = λ2 λ2 − 2λ + 4

−2 ± 4 + 4
⇒ λ= = −1 ± 2
2
λ>0⇒ = λ 2−1
∴ a = 2 and b = 1
(a + b)3 + (a – b)3 = (3)3 + (1)3 = 27 + 1 = 28
Hence, the value of (a + b)3 + (a – b)3 is 28.

  2   2
 a b   a −b 
Q. Prove that   2 −  2  =
| a |
 | b | 
   
 | a || b | 
 
Sol. Given | a | a=
= and | b | b
  2   2
 a b  a −b
We need to prove that  2 − 2  =  
a b 
  ab 
  2
 a b 
LHS
=  2 − 2
a b 

   
 a b  a b 
= 2 − 2   2 − 2 
a b a b 
 2   
|a| a·b b·a | b |2
= 2 2 − 2 2 − 2 2 + 2 2
a ·a a ·b b ·a b ·b
 
Vector & Three-Dimensional Geometry

2
a a·b a·b b2
= 2 2 − 2 2 − 2 2 + 2 2
a ·a a ·b a ·b b ·b

1 2a·b 1
= 2 − 2 2 + 2… ...(i)
(i)
a a ·b b
  2    
a −b a −ba −b
RHS =  =
 ab   ab   ab 
    
1       
= 2 2
(a·a − a·b − b·a + b·b)
ab
1  2      2
=
a 2 b2
(
| a | − a·b − a·b + | b )|

38.

=
1
2 2
ab
(a 2
− 2a·b + b2 )

1 2a·b 1
=2 − 2 2 + 2
b ab a

1 2a·b 1
= 2 − 2 2 + 2… ...(ii)
(ii)
a ab b
From (i) and (ii), we get LHS = RHS Hence proved.

 
Q. A point P moves in space such that P(A) · P(B) < 0 then the locus of the point P
is the interior of the sphere with AB as diameter.


Sol. Let origin is A .

Let P = x ˆi + y j + zk

B = aiˆ + b j + ck
[fixed point]

PA = − ( x ˆi + yjˆ�
+ zkˆ )

PB = (a − x )iˆ + (b − y ) j + (c − z)k
 
Now, PA · PB < 0
⇒ x2 + y2 + z2 – ax – by – cz < 0
Which represents interior portion of sphere.

Q. Find vector equation of an ellipse/a Hyperbola whose foci are the position

vectors and − c and length of major/Transversal Axis is 2a.

 Vector & Three-Dimensional Geometry


Sol. If r be any point on Ellipse/Hyperbola
   
then | r + c | + r − c = 2a for Ellipse …(i)
   
and r + c − | r − c |=2a for Hyperbola …(ii)
   
| r + c |2 = (2a − | r − c |)2 [from (i)]
     
r2 + c2 + 2r·c = 4a2 + r2 + c2 − 2r·c − 4a | r − c |
  
a2 − r.c = a | r − c |
 2  
Similarly for Hyperbola r·c − a = a | r − c |

39.
Q. Arc AC of the quadrant of a circle with centre as origin and radius unity sub-
tends a right angle at the origin. Point B divides
 the arc AC in the ratio 1 : 2.
 
Express the vector c in terms of a and b .

Sol. Let the radius of the arc be r. Now, B divided the arc in ratio 1 : 2, so AB would
subtend angle of 30° at the origin O and similarly, BC would subtend an angled

 ˆ c rjˆ=
of 60° origin. Let a = ri,
= ( ) )
ˆj r ˆj + r 3 ˆi
,b r cos 30° ˆi + r(sin 30° =
2 2
 1 3    
∴ b= c+ a ⇒ c = 2b − 3a
2 2

Q. Two adjacent sides of a parallelogram ABCD are given by AB =2i + 10j + 11k

and AD = −i + 2j + 2k . The side AD is rotated by an actue angle a in the plane of
the parallelogram so that AD becomes AD’, If AD’ makes a right angle with the
side AB, then the cosine of the angle a is given by [JEE 2010]
8 17 1 4 5
(A) (B) (C) (D)
9 9 9 9

Sol. We have
−2 + 20 + 22 40 8
cos
= φ = =
2 2
2 + 10 + 11 2 2 2
1 +2 +22 15 × 3 9

17 17
We have = sin φ then cos =
α sin=
φ
9 9

i − j and b =
i + 2j are two vectors. Find a unit vector coplanar
Q. Given that a =
  
with a and b and perpendicular to a .
Vector & Three-Dimensional Geometry

 
Sol.
Let r is co-planar vector with a & b
     
r = a + λb and r ^ a ⇒ r·a = 0
  
So, r·a= a·a + λa·b
⇒ 0 = 2 + λ(–1)
⇒λ=2
  
r = a + 2b = ˆi − j + 2iˆ + 4 j = 3iˆ + 3 j

î + j 1 ˆ 1 
=r = i+ j
2 2 2

40.
Q.   

a

b

c
  
If a,b, c are coplanar vectors, prove that a·a a·b a·c = 0 .
  
b·a b·b b·c

  
Sol. As a,b and c are coplanar, they are linearly dependent.
  
So, λa + µb + nc = 0 …(1)
for some scalars λ, µ, n.
 
Taking dot product with a and b respectively,
  
λa·a + µa·b + va·c = 0
  
and λb·a + µb·b + vb·c = 0
λ
∴    
(a·b)(b·c) − (b·b)(a·c)
µ
=    
(a·c)(b·a) − (a·a)(b·c)
v
=    
(a·a)(b·b) − (a·b)(b·a)
∴ from (1),
                  
a{(a·b)(b·c) − (b·b)(a·c)} +b[(a·c)(b·a) − (a·a)(b·c)} +c[(a·a)(b·b) − (a·b)(b·a)] =
0
  
a b c
  
∴ a·a a·b a·c = 0
  
b·a b·b b·c

Q. If two pairs of opposite edges of a tetrahedron are at right angles, then


show that the third pair is also at right angle. Further show that for such a

Vector & Three-Dimensional Geometry


tetrahedron, the sum of the squares of each pair of opposite edges is the
same.

Sol.
Let ABCD be the tetrahedron.
Take D as the origin.
  
Let the position vectors of A, B, C be a,b, c respectively.
        
Then AB = b − a,BC = c − b, CA =a−c
Let DA ^ BC and DB ^ CA.
(i) To prove: DC ^ AB.
Since DA ^ BC,

41.
     
∴ DA= · BC 0 i.e. (a − 0)·(c = − b) 0
  
⇒ a·c − a·b = 0 …(1)
Since DB ^ CA.
     
∴ DB = · CA 0 i.e. (b − 0)·(a = − c) 0
 
⇒ b·a − b·c = 0 …(2)
   
Adding (1) and (2), a·c − b·c = 0 [ a·b = b·a]
  
⇒ (a − b)·c = 0
  
⇒ c·(a − b) = 0
   
⇒ (c − 0)·(a − b) = 0
   
⇒ DC · BA =0 ⇒ DC · AB =0 ∴ DC ^ AB
Hence, the first part.
   
(ii) Now (DA)2 + (BC)2 = (a − 0)2 + (c − b)2
   
= a2 + c2 + b2 − 2c·b
   
(DB)2 + (CA)2 = (b − 0)2 + (a − c)2
   
= b2 + a2 + c2 − 2a·c
   
and (DC)2 + (AB)2 = (c − 0)2 + (b − a)2
    
= c2 + b + a 2 − 2a·b
  
But a·b = b·c = c·a [From (1) and (2)]
∴ (DA) + (BC) = (DB) + (CA)2 = (DC)2 + (AB)2
2 2 2

Hence, the second part.

        
Q. If a,b and c are unit vectors, then | a − b |2 + | b − c |2 + | c − a |2 does NOT
exceed
Vector & Three-Dimensional Geometry

(A) 4 (B) 9 (C) 8 (D) 6

Sol. (B)

Given that,
     
| a − b |2 + | b − c |2 + | c − a |2
        
=| a |2 + | b |2 −2(a·b)+ | b |2 + | c |2 − 2(b·c)+ | c |2 + | a |2 −2(c·a)
     
= 2 | a |2 + | b |2 + | c |2 −(a·b + b·c + c·a)
 
  
= 2[3 − (a·b + b·c + c·a)]

42.
  
=6 − 2(a·b + b·c + c·a)
  
So, | a + b + c |2 ≥ 0
     
⇒| a |2 + | b |2 + | c |2 +2(a·b + b·c + c·a) ≥ 0
  
⇒ 3 + 2(a·b + b·c + c·a) ≥ 0
   3
⇒ a·b + b·c + c·a ≥ −
2
  
⇒ −2(a·b + b·c + c·a) ≤ 3
By using (i) and (ii), it will get
     
| a − b |2 + | b − c |2 + | c − a |2 ≤ 6 + 3
     
⇒| a − b |2 + | b − c |2 + | c − a |2 ≤ 9
Therefore, the correct option is (B).

Q. Use vectors to prove that in a DABC cos 2A + cos 2B + cos 2C > – 3/2. Also
prove that the distance between the circumcentre and the centroid is

R2 −
9
(
1 2
a + b2 + c2 . )

Sol. Consider a circle with centre O and a ∆ABC inscribed in it.


  
⇒ | OA=| | = OB | | OC
=| R (circum-radius)
  
 | OA + OB + OC |2 ≥ 0
     
⇒ (OA + OB + OC)·(OA + OB + OC) ≥ 0
  
⇒ | OA |2 + | OB |2 + | OC |2 + 2
     
(OA · OB + OB · OC + OC · OA) ≥ 0

Vector & Three-Dimensional Geometry


⇒ 3R2 + 2R2 (cos 2A + cos 2B + cos 2C) ≥ 0
⇒ 2(cos 2A + cos 2B + cos 2C) + 3 ≥ 0
3
∴ cos 2A + cos 2B + cos 2C ≥ −
2
and
     
Take “O” as the origin. Let OA = α, OB = β and OC = γ . Therefore
     
( β, =
γ ) 2A, ( γ, α
= ) 2B and ( α, β) =2C
  
 α +β+γ
It is known that OG = (see Corollary)
3

43.
Therefore
   
9 | OG |2 = | α + β + γ |2
   
= | α |2 + | β |2 + | γ |2 +2 ∑ (β·γ )
3R2 + 2R2 (cos 2A + cos 2B + cos 2C)
=
= 3R2 + 2R2  1 − 2 sin2 A + 1 − 2 sin2 R + 1 − 2 sin2 C 
 
9R2 − 4R2 sin2 A − 4R2 sin2 B − 4R2 sin2 C
=
 a b c 
= 9R2 − a2 − b2 − c2  = = = 2R 
 sin A sinB sinC 

Hence | OG |2 = R2 −
9
1 2
a + b2 + c2 ( )
Linear Combination of Vectors :

A vector r is said to be a linear
  
combination of the vectors a,b, c……
if ∃ scalars x, y, z,… such that
   
=r xa + yb + zc + …

Theorem in plane :
 
If a and b are two non zero non collinear

vectors then any vector r coplanar with
them can
be expressed as a linear
  
combination r = xa + yb ⋅ (Explain using a
sketch)
Vector & Three-Dimensional Geometry

Q. Arc AC of the quadrant of a circle with centre as origin and radius unity sub-
tends a right angle at the origin. Point B divides the arc AC in the ratio 1 : 2.
  
Express the vector c in terms of a and b .

  
Sol. c xa + yb …(i)
 =

Taking dot product with a in (i)
     
c ⋅ a = xa ⋅ a + ya ⋅ b

3
0= x + y …(ii)
2

Taking dot product with c in (i)

44.
     
c ⋅ c = xa ⋅ c + yb ⋅ c
y
1=0+  ⇒ y = 2
2
  
from (ii), x =− 3  ∴  c =− 3a + 2b

VECTOR OR CROSS PRODUCT OF TWO VECTORS :


Definition :

If a and b be two vectors and θ 0 ≤ θ ≤ π be ( )
the angle between them, then their vector (or
cross) product is defined to be a vector whose
magnitude is absinθ and whose direction is
 
perpendicular to the plane of a and b such that
   
a,b and a × b form a right handed system.
   
∴= a × b | a || b | sin θn
Where n is a unit vector perpendicular to the
   
plane of a and b such that a,b and n̂ form a
right handed system.

Properties of Vector Product :


       
1. In general, a × b ≠ b × a. In fact a × b =−b × a .
    
(
2. For scalar m,ma × b = m a × b = a × mb . )
      
( )
3.  a × b ± c = a × b ± a × c
    
4. If a  b then θ =0 or π ⇒ a × b =0 (but
        
a × b = 0 ⇒ a = 0 or b = 0 or a  b). In particular
  
Vector & Three-Dimensional Geometry
a×a = 0
         
5. If a ^ b= ˆ=
then a × b | a || b | n (or | a × b | | a || b |)

6. i × i = j × j = k × k = 0 and i × j = k,
 j × k = i and

kˆ × ˆi =ˆj (use cyclic system)


 
7. Unit vector perpendicular to a and b is given by
 
a ×b
±  
a ×b

45.
 
8. If θ is angle between a and b then
 
a ×b
sinθ =  
a b
 
9. If a = a 1ˆi + a2ˆj + a3kˆ and b = b1ˆi + b2ˆj + b3kˆ then
ˆi ˆj kˆ
 
a ×b =a1 a2 a3
b1 b2 b3

= ( a 2b3 − a 3b2 ) ˆi + ( a 3b1 − a 1 b3 ) ˆj + ( a 1 b2 − a 2b1 ) kˆ

   
Q. If a = 2iˆ + 2ˆj − kˆ and b = 6iˆ − 3ˆj + 2kˆ then a × b equals
(A) 2iˆ − 2ˆj − kˆ (B) ˆi − 10ˆj − 18kˆ (C) ˆi + ˆj + kˆ (D) 6iˆ − 3ˆj + 2kˆ

Sol. (B)
ˆi ˆj kˆ
 
a × b = 2 2 −1 = ˆi(4 − 3) − ˆj(4 + 6) + k(
ˆ −6 − 12) = ˆi − 10ˆj − 18kˆ
6 −3 2

Q. If angle between ˆi − 2ˆj + 3kˆ and 2iˆ + ˆj + kˆ is θ then sinθ equals


(A) 5 / 7 (B) 5/21 (C) 5 / 2 7 (D) 3 / 14

Sol. (C)
 
a ×b

We know that sinθ =  
a b
 
Now a × b =−5iˆ + 5ˆj + 5kˆ
Vector & Three-Dimensional Geometry

 
∴ a × b= (5)2 + (5)2 + (5)2= 75= 5 3
 

a= 1 + 4 + 9, b = 4 + 1+ 1

5 3 5 3 5 5
∴ sin
= θ = = =
1+ 4 +9 4 + 1+ 1 14 6 28 2 7

46.

Q. Find equation of a line which passes through the point with p.v. a and
     
perpendicular to the lines r = b + λp and r = c + µq .

     
Sol. Equation r= a + λd for d= p × q
   
So required equation of line is r = a + λ(p × q)

Q. Prove that for any vector


   2
| a × i |2 + | a × j |2 + | a × k |2 =
2a


Sol. Let a = a 1ˆi + a2ˆj + a3kˆ



∴ a × ˆ=
i ( )
a 1ˆi + a2ˆj + a3kˆ × ˆ=
i a 1 (iˆ × ˆi) + a2 (ˆj × ˆi) + a3 (kˆ × ˆi)

= a3ˆj − a2kˆ
 
∴ | a × i =| a23 + a22

Similarly, | a × ˆj =| a21 + a32 and | a ˆ × kˆ =| a21 + a22
   
L.H.S. = | a × ˆi |2 + | a × ˆj |2 + | a × k |2

= a22 + a23 + a21 + a23 + a21 + a22

( )
 2
= 2 a21 + a22 + a23 = 2 | a |= R.H.S.

Q. Find the equation of the line through the point with p.v. 2i + 3j and
 
perpendicular to the vectors A =i + 2j + 3k and B =3i + 4j + 53k


a= 2I + 3j Vector & Three-Dimensional Geometry
Sol.   
Equation of line r = a + λb
  
b= A × B
ˆi ˆj kˆ

b 1 2 3= ˆi(106 − 12) − ˆj(53 − 9) + k(4
= ˆ − 6) = 94iˆ − 44ˆj − 2kˆ
3 4 53



r= (2I + 3j) (
 + λ 94I − 44j − 2k .
)

47.
Q. If (2i + 6j + 27k)
 × (i + λj + µk)
 = 0 , find l and m.

i j k

Sol. 2 6 27= i(6µ − 27λ ) − j(2µ − 27) + k̂(2λ − 6) =0


1 λ µ

So, 2l – 6 = 0 ⇒ λ = 3
and 2µ – 27 = 0
27
⇒ µ=
2
Geometrical interpretation of vector product :
 
The vector product of the vectors a and b
represents a vector whose modulus is equal
to the area of the parallelogram whose two


adjacent sides are represented by a and b .
Area of parallelogram = base × height
 
= absinθ= a × b

Area of quadraliteral if its diagonal vectors


  1  
are d1 & d2 is given=
by d × d2
2 1

Q. Find the area of a parallelogram whose two adjacent sides are represented by

a = 3iˆ + ˆj + 2kˆ and b = 2iˆ − 2ˆj + 4kˆ

 
Sol. Area of parallelogram= a ×b

ˆi ˆj kˆ
 
2 = 8iˆ − 8ˆj − 8kˆ
Vector & Three-Dimensional Geometry

Now a × b = 3 1
2 −2 4

∴ Area = | 8iˆ − 8ˆj − 8kˆ |= 8 3 units

Area of a triangle :
1 1   1  
1. A
 rea of triangle ABC
= absin=
θ a ×=
b AB × AC
2 2 2
  
2. If a,b, c are position vectors of vertices of a ∆ABC then its
1    
Area=
2
( ) (
a ×b + b×c + c×a  ) (
 
)

48.
Q. Find the area of ∆ABC if position vectors of its vertices A,B, C are ˆi + ˆj, ˆj + kˆ and
kˆ + ˆi respectively.

Sol. AB = (ˆj + k)
ˆ − (iˆ + ˆj) = kˆ − ˆi

AC = (kˆ + ˆi) − (iˆ + ˆj) = kˆ − ˆj
i j k
 
∴ AB × AC =−1 0 1 =ˆi + ˆj + kˆ
0 −1 1

1   1
∴ Area of ∆ABC
= | AB × AC
=| 1 + 1 +=
1 3 /2
2 2
INTERPRETATION OF VECTOR PRODUCT AS
VECTOR AREA :
1. Vector area of plane figures:
With every closed bound surface which has
been described in a certain specific manner
and whose boundaries do not cross, it is

possible to associate a directed c such that

(i) 
 c = no. of units of area enclosed by the
plane figure

(ii)  The support of c is perpendicular to the
area and
(iii) The sense of description of the boundaries

and the direction of c is in accordance with
the R.H. screw rule.
2.
Vector area of a plane ∆ (Triangle) :

Vector & Three-Dimensional Geometry


1  
Vector area of ∆OAB is= ∆ (a × b)
2
  
If a,b, c are the position vectors then the
vector area of ∆ABC is
1    
=
2
( ) (
c−b × a −b 
 )
 
=
2
((
1 
) (   
)
a × b + b × c × (c × a ) )

49.
Note :
  
(i) If 3 points with position vectors a,b and c
     
are collinear then a × b + b × c + c × a =0
(ii) Unit vector perpendicular to the plane of the
  
∆ABC when a,b, c are the p.v. of its angular
     
a ×b + b×c + c×a   
point is n̂ = ± , where a,b, c
2∆
are the position vectors of the angular points
of the triangle ABC.
(iii) Vector Area of a quadrilateral ABCD = Vector
area of
∆ABC + vector area of ∆ACD
1   1  
= (
2
AB × AC +) (
2
AC × AD )
1     1   
= (
2
AB × AC − AD × AC= ) 2
(
AB − AD × AC)
1    1  
= (
2
)
AB + DA × AC =
2
DB × AC

1   1  


∴  Area of  Quadrilateral ABCD= DB × AC= AC × BD
2 2

Q. ( ) ( ) (
Using vector method, show that the points A 2, −1, 3 ,B 4, 3, 1 and C 3, 1, 2 are )
collinear

Sol. Let O be the origin


Given

A ≡ (2, −1, 3) ∴ OA = 2iˆ − ˆj + 3kˆ

B ≡ (4, 3, 1) ∴ OB = 4ˆi + 3ˆj + kˆ
Vector & Three-Dimensional Geometry



C ≡ (3, 1, 2) ∴ OC = 3iˆ + ˆj + 2kˆ

Now AB= (P.V. of B) − (P.V. of A)= (4iˆ + 3ˆj + k) ˆ − (2iˆ − ˆj + 3k)
ˆ = (2iˆ + 4ˆj − 2k)
ˆ

And AC = (P.V. of C) − (P.V. of A) = (3iˆ + ˆj + 2k)
ˆ − (2iˆ − ˆj + 3k)
ˆ = (iˆ + 2ˆj − k)
ˆ

ˆi ˆj kˆ ˆi ˆj kˆ
  
∴ AB ×=
2 4 = AC
−2 1 2= −1 0  R2 and R3 are identical 
1 2 −1 1 2 −1
 
Thus, AB and AC are parallel vectors, having a common point A.
Hence, the points A, B, C are collinear.

50.
Q. AD, BE and CF are the medians of a triangle ABC intersecting in G . Show that
1
∆AGB = ∆BGC = ∆CGA = ∆ABC
3
 
Sol. Let b, c be the position vectors of B and C with respect to A as the origin of
reference.
1   1 1
Therefore, the position vectors of D, E, F are
2
( )
b + c , c, b respectively. Also
2 2
the position vector of the point G , the centroid, is
1   1  

3
(0 + b + c=) 3
(
b+c )
1  
Therefore, area of ∆AGB =
2
(
AB × AG )
1  1   1    
=
2 3
(
b× b + c = ) 6
b×b +b×c

1   1
= b× c=  area of ∆ABC
6 3
1
Similarly, we can show that area of ∆BGC = area of ∆ABC and area of
3
1
∆CGA = area of ∆ABC
3

        
Q. For a non zero vector a , if a·b = a·c and a × b = a × c . Prove that b = c .
 
Sol. a·b = a·c
         
a·(b − c) = 0 ⇒ a ^ b − c and a × b = a × c
    
a × (b − c) = 0 ⇒ a || (b − c)
As a is not zero
 
Vector & Three-Dimensional Geometry
⇒ (b − c) = 0
 
b=c

Q. Find : (i) A vector of magnitude 6 perpendicular to the plane ABC


(ii) Area of triangle ABC
(iii) Length of altitude from A
(iv) Perpendicular distance of a point P from a
line using cross product.
  
(a − b) × c
d=  .
|c|

51.

AB = 2ˆi − k − ˆi + j − 2k
Sol.
= î + j − 3k

BC = 2 j + k − 2iˆ + k
=−2iˆ + 2ˆj + 2kˆ
î ˆj kˆ
 
(i) AB × =
BC 1 1 −3
−2 2 2

= (2 + 6)iˆ − ˆj(2 − 6) + k(2


ˆ + 2)

= 8iˆ + 4ˆj + 4kˆ


8iˆ + 4ˆj + 4kˆ
Required vector = 6
4 6
= 2iˆ + ˆj + kˆ
1  
(ii) Area = | AB × BC |
2
1
= 4 6 = 2 6
2
1
(iii) AD × BC =Area of ∆ABC, where AD is altitude
2
2×2 6

⇒ AD= = 2 2
2 3
  
( )
(iv) | a − b × c |

  
= | c| |
a −b
| sinθ
(Perpendicular dis tance d)
  

| a −b ×c |
d= 
( )
Vector & Three-Dimensional Geometry

| c|

   
Q. b =3i − 2j + 7k and c = 2i − j + 4k . Find the vector d which
Let a =i + 4j + 2k;

   
is perpendicular to both a and b and satisfy c . d = 15 .

Sol. We have,
ˆi ˆj kˆ
 
a × b= 1 4 2= 32iˆ − ˆj − 14kˆ
3 −2 7

52.
    
Since d -is perpendicular to both a and b and a × b is also perpendicular to both
 
a and b .
  
⇒ d is parallel or collinear to a × b
  

⇒d= (
λ a × b for some scalar λ)

⇒ d= 32λˆi − λˆj − 14λkˆ

Given c·d = 15
⇒ 2.32λ + ( −1)( −λ ) + 4( −14λ ) =15
5
⇒ 9λ =15 ⇒ λ =
3
 5
⇒=d (32iˆ − ˆj − 14k)
ˆ
3
 
Q. Four vertices O, A, B, C of a tetrahedron satisfy OA × OB = i − j + k ;
     
OB × OC = i; OC × OA = −i + j find CA × CB .

Sol. The concept here is the sum of all normal vectors in a tetrahedron is zero.
       
OA × OB + OB × OC + OC × OA + CB × CA = 0

(( )
 
) ()
CB × CA =− i − j + k + i + ( −i + j)

 

CA × CB = −i − k

   
Q. If a,b, c, d are position vectors of the vertices of a cyclic quadrilateral ABCD
           
| axb + b × d + dxa | | bxc + cxd + dxb |
prove that :     +     0
=
(b − a)·(d − a) (b − c)·(d − c)

Vector & Three-Dimensional Geometry


         

Sol.
a ×b + b× d + d× a a−d × b−a
=  
( ) ( ) =
sinA
=tanA
     
(
b−a ⋅ d−a )( )
b−a ⋅ d−a ( )( ) cosA
     
b× c + c × d + d×b
   
(
b−c ⋅ d−c)( )
   
(
b−c × c−d) ( sinC )
=     = = tanC

(
b−c ⋅ d−c)( cosC )

53.
As it is a cyclic quadrilateral, we have
A = 180° – C or tanA = tan(180°–C) or tanA + tanC = 0
           
a ×b + b× d + d× a b× c + c × d + d×b
    +     =0
( )(
b−a ⋅ d−a )
b−c ⋅ d−c ( )( )
      
Q. Find the unknown vector R satisfying R × b = c × b and R.a = 0 where
 
 b = i + j + k and c = 4i − 3 j + 7k .
a = 2i + k,
          
Sol. ( )
R × b = c × b ⇒ R − c × b = 0 ⇒ R = c + tb
     
  c⋅a
Since R ⋅ a =0 ∴ c ⋅ a + t b ⋅ a =0 ⇒ t =−  
b⋅a
  c ⋅ a  15
∴ R = c −   b = 4, −3, 7 −
b⋅a
( 3
1, 1, 1 ) ( )

= (4,–3, 7)–(5, 5, 5)
( )
= −1, −8, 2 = −i − 8j + 2k

     
Q. Prove that 3 points with position vectors a + b, a − b and a + λb are collinear
for ∀ l ∈ R.

     
Sol. Let the position vectors of points A, B and C be a + b, a − b, a + λb respectively.

Then, AB = P.V. of B - P.V. of A.
   
= a −b−a −b

= −2b

BC = P.V. of C - P.V. of B.
   
= a + λb − a + b
Vector & Three-Dimensional Geometry


= b λ+1 ( )

CA = P.V. of A - P.V. of C.
   
= a + b − a − λb

= b 1− λ ( )
  
Now, the points A,B,C are collinear if and only if, AB and CA are multiples of BC
 
Let, AB = βBC
 
⇒ −2b = βb λ + 1 ( )

(
⇒ −2 = β λ + 1 )

54.
2
⇒β= −
λ+1
Hence, for real values of λ, the given position vectors are parallel.

     
Q. Prove the identity (a − b) × (a + b) = 2(a × b) and give its geometrical
interpretation.

   
Sol. (
L.H.S = a − b × a + b ) ( )
       
= a × a + a × b − b × a − b × b [By distributive law]
   
= a ×b −b×a
   
[ a × a = 0 and b × b = 0]
   
= a ×b + a ×b
 
= 2 a ×b ( )
   
If a and b are the adjacent sides of a parallelogram, then its diagonal are a + b
 
and a − b
     
(
Now, a − b × a + b = 2 a × b) ( ) ( )
  1    
⇒ a × b=
2
( a − b) × (a + b)

  1    
⇒ a ×b
=
2
( ) (
a −b × a +b )

⇒ Area of the parallelogram
1
= ( Area of the parallelogram whose adjacent side are diagonal of the given
2
parallelogram)

Previous Year’s Question Vector & Three-Dimensional Geometry

         1
If a,b, c and d are unit vectors such that (a × b)·(c
= × d) 1 and
= a·c , then
2
     
(A) a,b, c are non-coplanar (B) b, c, d are non-coplanar [JEE-2009]
     
(C) b, d are non-parallel (D) a, d are parallel and b, c -are parallel

55.
      
Q. Let OA = a, OB =10a + 2b and OC = b where O, A & C are non-collinear points.
Let ‘p’ denote the area of the quadrilateral OABC, and let ‘q’ denote the area
of the parallelogram with OA and OC as adjacent sides. If p = kq. Find k.
      
Sol. Here OA = a, OB =10a + 2b, OC =b
q = Area of parallelogram with OA and OC as adjacent sides.
 
∴  q = a × b …(i)

p = Area of quadrilateral OABC
= Area of △OAB+ Area of △OBC
1   1  
=
2
( 
)
a × 10a + 2b +
2
( 
)
10a + 2b × b

   
= a × b +5 a × b
 
∴  p = 6 a × b

or p = 6q [ From Eq. (i)]
∴ k=6

Shortest distance between 2 skew lines :


Note:
(i) 2 lines in a plane if not  must intersect and
2 lines in a plane if not intersecting must be
parallel. Convertely 2 intersecting or parallel
lines must be coplanar.
(ii) In space, however we come across situation
when two lines neither intersect nor  , Two
such lines (like the flight paths of two planes)
in space are known as skew lines or non
Vector & Three-Dimensional Geometry

coplanar lines.
(iii) 
S.D. between two such skew lines is the
segment intercepted between the two lines
and perpendicular to both.
Method I: Two ways to determine the S.D.
  
L 1 : r = a + λp
 
L2 : r = b + µq
  
n= p × q
  
AB= (b − a)

56.
     
  AB·n (b − a)·(p × q)
S.D.= Projection of AB = on n =   
n p×q

If S.D.= 0 ⇒ lines are intersecting and hence coplanar.


   
Method II: p.v. of N1 = a + λp ; p.v. of N2 = b + µq
     
N2 − N=
1 (b − a) + ( µq − λp)
     
Now (N 1 )
− N2 = (
.p 0 and N1 − N2 =)
.q 0 (two linear equations to get

the unique values of λ and µ.) One p.v’s of N1 and N2 are known
we can also determine the equation to the line of shortest
distance and the S.D.

Shortest Distance between two parallel lines :


  
 
d= a − b sinθ=
a −b ×c ( )

c

Q. Find the shortest distance between the two lines whose vector equations are
 ˆ and r =2iˆ + 4ˆj + 5kˆ + µ(3iˆ + 4ˆj + 5k)
given by : r =ˆi + 2ˆj + 3kˆ + λ(2iˆ + 3ˆj + 4k) ˆ

     
Sol. If the equations of the lines are r= a 1 + λb1 and r= a2 + λb2 , then shortest
distance ‘d’ between them is given by
 
) ( )
 
(
a2 − a 1 ⋅ b1 × b2
d=   …(i)
Vector & Three-Dimensional Geometry
b1 × b2
 
Here a 1 =ˆi + 2ˆj + 3k,
ˆ b =2iˆ + 3ˆj + 4kˆ
1
 
a2 = 2i + 4 j + 5k, b2 = 3iˆ + 4ˆj + 5kˆ
ˆ ˆ ˆ
 
Now, a 2 − a 1 = (2iˆ + 4ˆj + 5kˆ ) − ( ˆi + 2ˆj + 3kˆ )
=ˆi + 2ˆj + 2kˆ …(ii)
ˆi ˆj kˆ
 
b1 × b2 =
2 3 4
3 4 5

57.
= (15 − 16)i − (10 − 12)j + (8 − 9)k
=−ˆi + 2ˆj − kˆ …(iii)
 
b1 × b2 = ( −1)2 + 22 + ( −1)2 = 6 …(iv)
  
)( )

and (
a2 − al · b2 × b1 = (iˆ + 2ˆj + 2k)·(
ˆ −ˆi + 2ˆj − k)
ˆ

= 1 × ( −1) + 2 × 2 + 2 × ( −1) = 1 …(v)


1 1
Substituting the values from (iv) and (v) in (i), we get
= d =
6 6

Q. Determine whether the following pair of lines intersect r = ˆi − ˆj + λ(2iˆ + k)
ˆ and

r= 2iˆ − ˆj + µ(iˆ + ˆj − k)
ˆ .

Sol. The shortest


  
distance between the given pair of lines is given by
(a2 − a1 )·(b1 × b2 )

d=  
b1 × b2

The two lines will intersect if and only if d = 0.


  
)( )

(
i.e. a2 − a 1 · b1 × b2 =
0
 
ˆ = a + λb
Here, equation of first line is r = ˆi − ˆj + λ(2iˆ + k) 1 1
 
where a 1 = ˆ ˆ
i − j and b1 = ˆ ˆ
2i + k
 
Also equation of second line is r= 2iˆ − ˆj + µ(iˆ + ˆj − k)ˆ= a + µb
2 2
 
where a2 = 2iˆ − ˆj and b2 = ˆi + ˆj − kˆ
ˆi ˆj kˆ
   
Now, b1 × b2 = 2 0 1 =−ˆi + 3ˆj + 2kˆ and a2 − a 1 =2iˆ − ˆj − (iˆ − ˆj) =ˆi
Vector & Three-Dimensional Geometry

1 1 −1
  
)( )

(
Since a2 − a 1 · b1 × b2 =ˆi·( −ˆi + 3ˆj + 2k)
ˆ =( −1)(1) + 3(0) + 2(0) =−1 ≠ 0

Hence the given lines do not intersect.

58.
Q. Find shortest distance between


r = i + j + λ(2i − j + k),

r= 2i + j − k + µ(3i − 5j + 2k)

 
Sol. Here a 1 = i + j, b1 = 2i − j + k
 
a2 = 2iˆ + ˆj − k,b
ˆ
2
= 3iˆ − 5ˆj + 2kˆ
 
∴ a2 − a 1 =ˆi − kˆ
ˆi ˆj kˆ
 
b1 × b2 = 2 −1 1 = 3iˆ − ˆj − 7kˆ
3 −5 2
 
∴ b1 × b2 = 9 + 1 + 49= 59

∴ The shortest distance between the given lines is


 
d=
( )(
 
b1 × b2 ⋅ a2 − a 1
 
)
b1 × b2

3−0+7 10

= =
59 59

Q. Vector equation of two lines are



r = (1 − t)i + (t − 2)j + (3 − 2t)k,

r = (s + 1)i + (2 s − 1)j − (2 s + 1)k
Find shortest distance.

Vector & Three-Dimensional Geometry
 ˆ = a + tb
r = (1 − t)iˆ + (t − 2)ˆj + (3 − 2t)kˆ = (iˆ − 2ˆj + 3k)
Sol. ˆ + t( −ˆi + ˆj − 2k)
1 1
 
ˆ ˆ ˆ ˆ
where a 1 =i − 2 j + 3k and b1 =− i + j − 2k ˆ ˆ

Again, r = (s + 1)iˆ + (2s − 1)ˆj − (2s + 1)kˆ = (iˆ − ˆj − k) ˆ + s(iˆ + 2ˆj − 2k)
ˆ
  
=r a2 + sb2
 
where a2 = ˆi − ˆj − kˆ and b2 =ˆi + 2ˆj − 2kˆ
ˆi ˆj kˆ
 
Now, b1 × b2 =−1 1 −2
1 2 −2

59.
= ˆi( −2 + 4) − ˆj(2 + 2) + k(
ˆ −2 − 1) = 2iˆ − 4ˆj − 3kˆ
 

∴ b1 × b2= (2)2 + ( −4)2 + ( −3)2= 4 + 16 + 9= 29
 
a2 − a 1 = ˆi − ˆj − kˆ − (iˆ − 2ˆj + 3k)
ˆ = ˆj − 4kˆ
 
(  
)(
Now b1 × b2 ⋅ a2 − a 1 = (2iˆ − 4ˆj − 3k) )ˆ ⋅ (ˆj − 4k)
ˆ

= 2(0) + (-4) (1) + (–3) (–4) = 0–4 + 12 = 8


 


=
∴ d
 
(
b1 × b2 ⋅ a2 − a 1
= = =
|8| 8 )(8 29 )
units
 
b1 × b2 29 29 29

Q. Find the distance between the lines given by



 + λ(2i + 3j + 6k)
r = (iˆ + 2j − 4k) 

 + µ(2i + 3j + 6k)
r = (3i + 3j − 5k) 

 ˆ a =3iˆ + 3ˆj − 5kˆ


Here a 1 =ˆi + 2ˆj − 4k,
Sol. 2

and b = 2iˆ + 3ˆj + 6kˆ [∵ both lines are parallel]
 
∴ a2 − a 1 = (3iˆ + 3ˆj − 5k)
ˆ − (iˆ + 2ˆj − 4k)
ˆ

= 2iˆ + ˆj − kˆ

| b |= (2)2 + (3)2 + (6)2

= 4 + 9 + 36 =7
ˆi ˆj kˆ
  
and a2 − a 1 =
×b ( ) 2 1 −1
2 3 6
Vector & Three-Dimensional Geometry

= ˆi(6 + 3) − ˆj(12 + 2) + k(6


ˆ − 2)

= 9iˆ − 14ˆj + 4kˆ



∴ (a2 − a 1 ) =
×b (9)2 + ( −14)2 + (4)2 = = 81 + 196 + 16 = 293

Shortest distance between the lines is



  
S.D. d = = =
(
a2 − a 1 × b 293 293 )
|b| 7 7

60.
PRODUCT OF THREE OR MORE VECTORS :
Scalar triple product : Know the facts
Definition :
  
If a,b, c are three vectors, then their scalar
Scalar triple product always
triple product is defined
  as the dot product of results in a scalar quantity

two vectors a and b × c . It is generally denoted (number).
     
( )
by a ⋅ b × c or a b c  . It is read as box
 
  
product of a,b, c. Similarly other scalar triple
     
( ) (
products can be defined as b × c ⋅ a, c × a ⋅ b )
Geometrical Interpretation :
       
(
a × b= )
⋅ c a b sinθ=

n ⋅ c a b c sinθcosφ
   
where = θ a ^= b; φ n ^ c
But
  
| a || b | sin θ area
= of ||gm OACB and | c | cos φ h
There absolute value of scalar triple
product of three vectors is equal to the
volume of the parallelopiped whose three
coterminous edges are represented by the
given vectors.
      Point to Remember!!!
Therefore (a × b)·c =a b c  = Volume of
 
the parallelopiped whose coterminous Formula for scalar Triple Product :
   
edges are a,b and c . If a = a 1i + a2 j + a3 k,

 
b = b1i + b2 j + b3 k and c
 = c1i + c2 j + c3 k,

Properties of Scalar Triple product :
then
1. The position of . ()and x ( )
can be
a 1 a2 a3
Vector & Three-Dimensional Geometry
        
( ) (
interchanged i.e. a ⋅ b × c = a × b ⋅ c ) a b c  = b b b
  1 2 3

         c 1 c2 c3
2.  = b c a   c a b
a b c   =
 
Therefore if we don’t change the cyclic order
of a, b and c then the value of scalar triple
product is not changed.
3. If the cyclic order of vectors is changed, then
sign of scalar triple product is changed i.e.
           

  ( )
a ⋅ b × c  =−a ⋅ c × b  or a b c  =−  a c b

From (ii) and (iii) we have

61.
                 
a b c  = b c a c a b a c b b a c 


   
=
  
 =
− 
 
 =
− 
 
 =
−  b a
 c

4. The scalar triple product of three vectors when two of them are equal
     
or parallel, is zero i.e. = a b a  0 
a b b  =
 
5. The scalar triple product of three mutually perpendicular unit vectors
is ±1 . Thus
ˆi ˆj kˆ  = 1, ˆi kˆ ˆj = −1
   
     
6. If two of the three vectors a,b, c are parallel then a b c = 0
 
     
7. a,b, c are three coplanar vectors if a b c  = 0 i.e. the necessary and
 
sufficient condition for three non-zero non-collinear vectors to be
  
coplanar is a b c  = 0
 
   
8. For any vectors a,b, c, d
         
a + b c d= a c d + b c d
     
  
9.  For right handed system, a b c  > 0 and for left handed system,
 
     
a b c  < 0; where a,b, c  are non-coplanar.
 
        
10.  a + b, b + c, c + a  = 2  a b c 
 
     
11.  a − b b − c c − a  is always zero.
 
     
l ⋅a l ⋅b l ⋅c
                 
12.   l m n a b c  = m ⋅ a m ⋅ b m ⋅ c , where l ,m,n & a,b, c are non
        
n⋅a n⋅b n⋅c

coplanar vectors.
Vector & Three-Dimensional Geometry

    
p⋅a q⋅a a
         
  ( )
13. a b c  p × q = p ⋅ b
 
q⋅b b
  
p⋅c q⋅c c
           
14. If a =a 1 l + a2 m + a3 n, b =b1 l + b2 m + b3 n and c =c1 l + c2 m + c3 n , then

a 1 a2 a3   
  
a b c  = b b b l m n
  1 2 3  
c 1 c2 c3

62.
   = i + j − k and c= 3i − k represent three coterminous edges of a
If a = 2i − 3j,b
Q.
parallelopiped, then the volume of that parallelopiped is-
(A) 2 (B) 4 (C) 6 (D) 10

Sol. (B)   
Volume = a b

c

2 −3 0
=| 1 1 −1 |= −2 + 9 − 3 =4
3 0 −1

        
Q.
For any three vectors a,b, c a + b, b + c, c + a  equals-
 
         2
(A) a b c  (B) 2 a b c  (C) a b c  (D) 0
     

Sol. (B)          
For any three vectors ( ) ( ) (  
)
 a + b b + c c + a  = a + b ⋅  b + c × c + a 
   
           
= ( ) (
a +b ⋅ b×c +b×a + c×c + c×a
 ) 
       

= ( )(
a +b ⋅ b×c +b×a + c×a   )  
 c × c = 0
     
= a b c  +  a b a  +  a c a  +  bb c  +  bb a  +  b c a 
           
  
= a b c  +  b c a  = 2 a b c 
     

        
Q.
 If a,b, c be three non- zero vectors, then  a = (
× b ⋅ c a b c ,  if − )
       
(A) a ⋅ b = b ⋅ c = 0 (B) b ⋅ c = c ⋅ a = 0
         

Vector & Three-Dimensional Geometry


(C) c ⋅ a = a ⋅ b = 0 (D) a ⋅ b = b ⋅ c = c ⋅ a = 0

Sol. (D) 
       
( a × b )=
⋅c
a b c ⇔ a × b c cosθ a b c ( )
        
(where θ is the angle between a × b=
and c) a b c sin
= φcosθ a b c
 
(where φ is the angle between a and b) ⇔ sinφ = 1, cosθ = 1 ⇔ φ = 90 , θ = 0
     
⇔ a ⋅ b= 0, a ⋅ c= 0,b ⋅ c= 0

63.
Q. If vectors aiˆ + ˆj + kˆ , ˆi + bjˆ + kˆ and ˆi + ˆj + ckˆ ( a ≠ b ≠ c ≠ 1 ) are coplanar, then
1 1 1
+ + equals
1−a 1−b 1−c
(A) 1 (B) 0 (C) −1 (D) None of these

Sol.
(A)
Since vectors are coplanar,
a 1 1 a 1 1
∴ 1 b 1 = 0 ⇒ 1−a b− 1 0 = 0  UsingR2 − R1 ,R3 − R2 
1 1 c 0 1−b c − 1

( )( ) ( ) ( ) ( )
⇒ a b− 1 c − 1 − 1−a  c − 1 − 1−b  =
 
0

⇒ a ( 1 − b )( 1 − c ) + ( 1 − a )( 1 − c ) + ( 1 − a )( 1 − b ) = 0

⇒ ( a − 1 + 1)( 1 − b )( 1 − c ) + ( 1 − a )( 1 − c ) + ( 1 − a )( 1 − b ) = 0

⇒ ( 1 − b )( 1 − c ) + ( 1 − a )( 1 − c ) + ( 1 − a )( 1 − b ) = ( 1 − a )( 1 − b)( 1 − c )
1 1 1
⇒ + + 1
=
1−a 1−b 1−c
  
Q.

If u = 2i − j + k, v = i + j + k and w is unit vector then find the maximum value
  
of [u v w] .

    
Sol. [u v w] = (u × v ) ⋅ w
ˆi j kˆ

= 2 −1 1 ⋅ w
1 1 1
Vector & Three-Dimensional Geometry


= [ˆi( −2) − j(1) + k(3)]
ˆ ⋅w≤ 4 +1 +9
 
∴ Maximum value of u v w  = 14

Q. Find the value of l for which three points with p.v.’s A (1, 0, 3), B (–1, 3, 4),
C (1, 2, 1) and D (l, 2, 5) are in the same plane. Also find the equations of the
plane through ABC.

64.
Sol. Equation of plane is
x−1 y−0 z−3
−1 − 1 3 − 0 4 − 3 =0
1− 1 2−0 1−3

( x − 1)( −6 − 2) − y ( 4 − 0) + (z − 3)( −4 − 0) =0
–8x + 8–4y–4z+12 = 0
2x + y + z = 5
∴ 2(λ) + 2 + 5 = 5 ⇒ λ = –1

Q. Find the value of p for which the vectors (p + 1)i − 3j + pk,pi
  + (p + 1)j − 3k and

−3iˆ + pjˆ + (p + 1)kˆ are coplanar.

Sol. The vectors are linearly dependent


p + 1 −3 p
⇒ p p + 1 −3 = 0 c1 → c1 + c2 + c3 and take (2p–2) common
−3 p p+1

1 −3 p
⇒ (2p-2) 1 p + 1 −3 =0
1 p p+1

1 −3 p
⇒ 2(p-1) 0 p + 4 −3 − p =0
0 p+3 1
⇒ 2(p-1)(p+4+(p+3)2) = 0
⇒ (p-1) (p2+7p + 13) = 0
Roots of p2 + 7p + 13 = 0 are (imaginary)
∴p=1

Q. Vector & Three-Dimensional Geometry


Find the altitude of the parallelopiped whose coterminous edges are vectors,
    
 b = 2i + 4 j − k,
a = i + j + k,  c = i + j + 3k
where a & b are base vectors.

 Volume of parallelepiped 
Sol. h=
 Area of base 
1 1 1
2 4 −1

[ABC] 1 1 3 4 2 38
= =   = =
| A ×B | ˆi ˆj kˆ | −5iˆ + 3ˆj + 2kˆ | 19

1 1 1
2 4 −1

65.

Q. The vector OP =i + 2j + 2k turns through a right angle, passing through the
positive xaxis on the way. Find the vector in its new position.

  


Given, OP =i + 2j + 2k after rotation of OP , let new vector is OP .
Sol.
 
Now, OP, i, OP′ will be coplanar
   
′ (OP × i) × OP    ′ 
So, OP | OP
= = | 
   | OP | OP 
| (OP × i) × OP |  
 
But, (OP × i) × OP = 8i − 2j − 2k
 3(8i − 2j − 2k)

⇒ OP' =
2×3 2
 4  1  1 
⇒ OP' = i− j− k
2 2 2
  
Q. 2v + v × (i + 2 j) = 2iˆ + k then find 3 | v | .

 
2V + V × (iˆ + 2ˆj) = (2iˆ + k)
Sol. ˆ (i) or

2V ⋅ (iˆ + 2ˆj) + 0 = (2iˆ + k)
ˆ ⋅ (iˆ + 2ˆj)

or 2V ⋅ (iˆ + 2ˆj) = 2 or

| V ⋅ (iˆ + 2ˆj) | = 1 or
 2
| V | ⋅ | ˆi + 2ˆj |2 cos2 θ =1

(θ is the angle between V and ˆi + 2ˆj )

(
or | V |2 5 1 − sin2 θ =1 or )
Vector & Three-Dimensional Geometry

 
V |2 5sin
= 2
θ 5 V |2 −1  (ii)From

Eq. (i), we have


 
| 2V + V × (iˆ + 2ˆj) |2 = | 2iˆ + kˆ |2
 
or 4 | V |2 + | V × (iˆ + 2ˆj) |2 = 5
 2  2
or 4 | V | + | V | ⋅ | ˆi + 2ˆj |2 sin2 θ =5
 
or 4 | V |2 +5 | V |2 sin2 θ =5
   
or 4 V |2 +5 V |2 −1 =5 or 9 | V |2 = 6 or 3 V = 6

66.
    

  + [jab]j + [kab]k
Q. Show that : a × =
b [iab]i

 
ˆ  ˆ ˆ ˆ
Sol. Let a = a 1 i + a 2 j + a 3k and b = b1 i + b2 ĵ + b3k

1 0 0
ˆ  ˆ
[ i a b ]i = a 1 a2 a 3 ˆi
b1 b2 b3

= ( a 2b3 − b2 a 3 ) ˆi
 
Similarly, [j a= b] j (
a3b1 − a 1b3 j )
 ˆ
[kˆ a=b]k (
a 1b2 − a2b1 kˆ )
   
∴ [ˆi a b ]iˆ + [ j a b] j + [kˆ a b]kˆ = ( a 2b3 − b2 a 3 ) î + ( a 3b1 − a 1 b3 ) j + ( a 1 b2 − a 2b1 ) kˆ

ˆi ˆj kˆ
 
= a1 a2 a3 = a × b
b1 b2 b3

Previous Year’s Question

  
If u, v, w are three non-coplanar unit vectors and a, b, g are the angles between
        
u and v, v and w, w and u respectively and x, y, z are unit vectors along the

bisectors of the angles a, b, g respectively. Prove that

     
x × y y × z z × x  1      2 α β γ [JEE 2003, 4 out of 60]
= u v w sec2 sec2 sec2
Vector & Three-Dimensional Geometry
  16   2 2 2

       
Q. [a + b b + c c + a] =
2[abc] . This identity can be geometrically interpreted as :

(Volume of a cuboid whose three coterminous edges are the face diagonals of
the cuboid is twice the volume of the cuboid, whose three coterminous edges
     
are the vectors a,b, c ) . This is also conclusive that if a,b, c are coplanar then
     
a + b,b + c, c + a , are also coplanar.

67.
         
Sol. (
a + b b + c c + a  = a + b ⋅  b + c × c + a 
   ) (
 
 ) ( )
         
( )
= a + b ⋅ b × c + c × c + b × a + c × a 
   
     

( )
= a + b ⋅ b × c + 0 + b × a + c × a 
           

 
( )  
( ) ( ) (
=a ⋅ b × c + a ⋅ b × a + a ⋅ c × a + b ⋅ b × c + b ⋅ b × a − b ⋅ (c × a )) ( )
  

 
( )
= a ⋅ b×c + 0 + 0 + 0 + 0 +b⋅ c×a ( )
        

( ) (
= a ⋅ b × c + a ⋅ b × c = 2 a ⋅ b × c 
 )
 ( )
 
= 2 abc 
 

     
Q. Prove that : [a − b b − c c − a] =
0

     
Sol.
To Prove : a − b b − c c − a  =
 
0
     
Solution: a − b b − c c − a 
 
    

(
a −b ⋅ b−c × c −a
 ) (   
) ( )
         
( )
a − b ⋅ b × c − b × a − c × c + c × a  By distributive law

       
(a − b). b × c − b × a + c × a 
 
                 
( ) ( ) ( ) ( )
a ⋅ b × c − b ⋅ b × c − a ⋅ b × a + b ⋅ b × a + a ⋅ ( c × a ) − b.(c × a)
     
abc  −  bbc  −  aba  +  bba  +  aca  −  bca 
           
 
abc  −  bca 
   
Vector & Three-Dimensional Geometry

⇒0 L.H.S = R.H.S Hence proved.

Previous Year’s Question


  
Let ( )
 = i + λj + 4k and c = 2i + 4j + λ2 − 1 k be coplanar vectors. Then the
a = i + 2j + 4k,b
 
non- zero vector a × c is: [JEE (Main) 2019]
(A) −10i − 5j (B) −14iˆ − 5ˆj (C) − 14iˆ + 5ˆj (D) −10i + 5j

68.
     
Q. l ·a l ·b l ·c
        
Prove that [lmn][abc] = m·a m·b m·c .
     
n·a n·b n·c


ˆ  = m ˆi + m ˆj + m kˆ and n
Let l = l 1ˆi + l 2ˆj + l 3k,m

= n1ˆi + n2ˆj + n3kˆ
Sol. 
1 2 3
 ˆ = b ˆi + b ˆj + b kˆ and c = c ˆi + c ˆj + c kˆ .
a = a 1ˆi + a2ˆj + a3k,b 1 2 3 1 2 3

Therefore, l ·a = l 1a 1 + l 2a2 + l 3a3 = Σl 1a 1

Similarly, l ·b = Σl 1b1 , etc.
l1 l2 l3 a1 a2 a3
  
Now [lmn][abc] = m1 m2 m3 b1 b2 b3
n1 n2 n3 c1 c2 c3

Σl 1 a 1 Σl 1 b1 Σl 1 c1
Σm1 a 1
= Σm1 b1 Σm1 c1
Σn1 a 1 Σn1 b1 Σn1 c1
  
l ·a l ·b l ·c
     
= m·a m·b m·c
  
n·a n·b n·c

Q.   
a·a a·b a·c
   
Prove that [abc]2 = b·a b·b b·c ,
  
c·a c·b c·c
  

Vector & Three-Dimensional Geometry


where a,b, c are non–coplanar vectors.

  
a = a 1 i + a2 j + a3 k, c = c1 ˆi + c2 ˆj + c3 kˆ
 etc.
Sol. b = b1 ˆi + b2 ˆj + b3 kˆ
   
∴ a . a =
= ∑ a21 , a .b ∑a b 1 1

a 1 a2 a3

∴  a b c  =
b1 b2 b3 =∆
 
c 1 c2 c3

69.
a 1 a2 a3 a 1 a2 a3
2
∆ =b1 b2 b3 b1 b2 b3

c 1 c2 c3 c 1 c2 c3

Σa21 Σa 1b1 Σa 1c1


Σb1a 1
= Σb21 Σb1C1
Σc1a 1 Σc1b1 Σc21
     
a ⋅a a ⋅b a ⋅c
     
b a b ⋅b b ⋅c
=⋅
     
c ⋅a c ⋅b c ⋅c

Q.     
l ·a l ·b l
         
Prove that l m n (a × b) =
  m·a m·b m .
 
    
n·a n·b n


Sol. Let l = l 1 i + l 2 j + l 3 k etc.
l1 l2 l3
  

∴  l m n =  m1 m2 m3
 
n1 n2 n3

i j k

( 
)
a ×b =a1 a2 a3
b1 b2 b3

l1 l2 l3 i j k
    
( )
Vector & Three-Dimensional Geometry

 l m= n a × b m1 m2 m3 ⋅ a1 a2 a3
 
n1 n2 n3 b1 b2 b3

l 1i + l 2 j + l 3 k ∑ l 1a 1 ∑ l 1b1
= m1i + m2 j + m3 k ∑ m1a 1 ∑ m1b1
n1i + n2 j + n3 k ∑ n1a 1 ∑ n1b1

70.
    
l l ·a l ·b
    
= m m·a m·b
    
n n·a n·b
    
l ·a l ·b l
    
= m·a m·b m
    
n·a n·b n

VOLUME OF TETRAHEDRON :
  
1. If a,b, c are position vectors of vertices A,B
and C with respect to O, then volume of
tetrahedron OABC represented by V is given by
1
V= Base area × height
3
1      
Base Area= a ×b +b×c + c×a
2
      
Let  a × b + b × c + c × a = n
1 
∴   Base area = n
2
 
Height = projection of a on n
        
| a·n | | a·(a × b + b × c + c × a) | | [a b c] |

= =   = 
|n| |n| |n|

 
1 1  a b c  1   
∴ V = ⋅ n  = abc
3 2 n 6  
   
2. If a,b, c, d are position vectors of vertices
A,B, C,D of a tetrahedron ABCD , then Vector & Three-Dimensional Geometry
 1   
 | [AB AC AD ] |
6
Its volume =  or
1      
 | [b − a c − a d − a] |
6

71.
  
Q. If the vertices of any tetrahedron be a =ˆj + 2k,
ˆ b =3iˆ + k,
ˆ c =4iˆ + 3ˆj + 6kˆ

and d = 2iˆ + 3ˆj + 2kˆ then find its volume.

   
Sol. Let the p.v. of the vertices A,B, C,D with respect to 0 are a,b, c and d
  
respectively then AB = b − a = 3iˆ − ˆj − kˆ
 
AC = 4iˆ + 2ˆj + 4kˆ and AD = 2iˆ + 2ˆj
3 −1 −1
1      1
Now volume of tetrahedron = AB AC AD = 4 2 4 = −6
6  6
2 2 0
∴ Required volume = 6 units

Previous Year’s Question

Let the volume of a parallelopiped whose coterminous edges are given by


  
 v = i + j + 3k and w = 2i + j + k be 1 cu. unit. If q be the angle between the
u = i + j + λk,
 
edges u and w , then cosq can be : [JEE Main 2020]
7 5 7 5
(A) (B) (C) (D)
6 6 7 6 3 3 3

VECTOR TRIPLE PRODUCT:


Definition:
The vector triple product of three vectors
  
a,b, c is defined as the vector product of
Vector & Three-Dimensional Geometry

  
two vectors a and b × c . It is denoted by
  
a× b×c ( )
  
( )
a × b × c is a vector which is coplanar with
  
a and b and perpendicular to c .
   
( 
)
Hence a × b × c = xa + yb …(1)
 
[linear combination of a and b ]
     
( )  
(
c ⋅ a × b × c= x a ⋅ c + y b ⋅ c ) ( )
72.
 
 
(
0= x a ⋅ c + y b ⋅ c ) ( ) …(2)

x y
∴  = −  = λ
b⋅c a ⋅c
 
(  
∴ x =λ b ⋅ c and y = −λ a ⋅ c) ( )
        
Substituting the values of x and y in a × b × c = λ b ⋅ c a − λ a ⋅ c b ( ) ( ) ( )
  
This is an identity and must be true for all values of a,b, c
  
Put=a ˆi;b
= ˆj and c= ˆi

(ˆi × ˆj) × ˆi =λ (ˆj ⋅ ˆi ) ˆi − λ (ˆi ⋅ ˆi ) ˆj


ˆj = −λˆj ⇒  λ = −1
        
hence a × b × c =( ) ( a ⋅ c ) b − (b ⋅ c ) a
Properties:
1. Expansion formula for vector triple product is given by
        

( ) (
a × b×c = a ⋅c b − a ⋅b c
     
) ( )
  
( ) (
b×c ×a = b⋅a c − c⋅a b ) ( )
     
a ⋅a a ⋅b a ⋅c
       2      
2. a × b b × c c × a  = a b c  = b ⋅ a b ⋅ b b ⋅ c
         
c ⋅a c ⋅b c ⋅c
        
Note that if a,b, c are non coplanar vectors then a × b,b × c and c × a
will also be non coplanar vectors.
3. Vector triple product is a vector quantity.
     
4. (
 a × b × c ≠ a × b × c ) ( )
Vector & Three-Dimensional Geometry
  
5.
   a ×b ×c
Unit vector coplanar with a & b and perpendicular to c is ±   
( )
a ×b ×c ( )

Q. ( ) ( )
ˆi × ˆj × kˆ + ˆj × kˆ × ˆi + kˆ × ˆi × ˆj  equals
( )
(A) î (B) ĵ (C) k̂ (D) 0

Sol. (D)
( ) ( ) ( )
 ˆi × ˆj × kˆ + ˆj × kˆ × ˆi + kˆ × ˆi × ˆj ⇒ ˆi × ˆi + ˆj × ˆj + kˆ × kˆ = 0 + 0 + 0 = 0 

73.
        
Q.  If a,b, c are coplanar, then show that a × b,b × c and c × a are also coplanar. 

  


Sol. a b c  are coplanar ⇒ a b c  =
    0 ⇒  a b c  =

0
     
⇒  a × b b × c c × a  =
0
     
⇒ a × b,b × c and c × a are coplanar.

         
Q. Let a,b, c such that= b 1 and c = 2 and if a × a × c + b =
a 1,= (
0 then acute )
 
angle between a and c is
π π π
(A) (B) (C) (D) None of these
3 4 6

Sol. (C)  
If angle between a and c is θ then
   
a ⋅ c a c cosθ
=
= 1.2cos = θ 2cosθ
   
But a × a × c + b =
0 ( )
      
⇒ (a ⋅ c) a − (a ⋅ a ) c + b = 0
  
⇒ ( 2cosθ ) a − 1 ⋅ c =−b
  
⇒ [ ( 2cosθ ) a − c]2 =− [ b]2
    
⇒ 4cos2 θ | a | 2 −2 ⋅ ( 2cosθ ) a ⋅ c + | c | 2 =
| b |2
⇒ 4cos2 θ − 4cosθ ( 2cosθ ) + 4 = 1 
 
( 2
⇒  4 1 − cos= )
θ 1 |= a | 1, =| b | 1

Vector & Three-Dimensional Geometry

⇒  sinθ =1 / 2
π
⇒  θ =   
6

       
Q. Let a = 2ˆi + ˆj − 2kˆ and b= ˆi + ˆj if vector c is such that a ⋅=
c c , c−=
a 2 2
   
( ) 
( 
and angle between a × b and c is the 30 then a × b × c is equal to )
2 3
(A) (B) (C) 2 (D) 3
3 2

74.
Sol. (B)     
( ) ( )
 | c − a |2 = c − a ⋅ c − a = (2 2)2
2 2  
⇒  c + a – 2c · a = 8
2  
⇒ c + ( 4 + 1 + 4 ) – 2c · a = 8
2    
⇒  c + 9 – 2 c = 8   a · c =| c | 
2  2 
⇒  c – 2 c + 1 = 0 ⇒ c – 2c + 1 = 0
 
⇒  (| c | –1)2 = 0 ⇒ c = 1
      1   1  
| (a × b) × c |= a × b | | c | sin30o = 1 × | a × b |= | a × b |
2 2
ˆi ˆj kˆ
 
 But  a × b = 2 1 −2 = 2ˆi − 2ˆj + kˆ
1 1 0
 
∴ a ×b = 4 + 4 + 1= 3

   3
( )
∴ a ×b ×c =
2

Q. Prove that [a × b b × c c × a] = [a b c]2

Sol. We have, [a × b b × c c × a]
= {(a × b) × (b × c)}·(c × a)
= {d × (b × c)}·(c × a) [ where, d = (a × b)]
=[(d·c)b − (d·b)c]·(c × a)
= [{(a × b)·c)b − (a × b)·b}c]·(c × a)

= {[a b c]b − 0}·(c × a) [ [ a b =


b ] 0] Vector & Three-Dimensional Geometry


= {a b c  {b·(c × a)}

= [a b c] [b c a] { [a b c] = [b c a]}
= [a b c]2

75.
Q.    
V 1 =a × (b × c) 
    
If V 2 =b × (c × a)  then prove that
    
V 3 =c × (a × b) 
  
(i) V 1 , V2 , V3 are coplanar
  
(ii) V 1 , V2 , V3 from the sides of a triangle
  
(iii) V 1 + V2 + V3 is a null vectors
  
(iv) V 1 + V2 + V3 are linearly dependent.

        
Sol. a × (b × c) = ( )
(a ⋅ c)b − a ⋅ b c
        

b × (c × a) = ( )
(b ⋅ a) c − b ⋅ c a
        
c × (a × b) = (c ⋅ b) a − (c ⋅ a)b
   
Adding, we get the result. V1 + v 2 + v 3 =
0 which proves (iii) and (ii)
Now
           
( ) (
Let p = a × b × c , q =b × c × a and r =c × a × b . ) ( )
           
( )
Then, p + q + r = a × b × c + b × c × a + c × a × b = 0 ( ) ( )
   
But, p + q + r =0 means that one of the vectors is a linear combination of other
two.
  
This shows that p, q, r are coplanar.
        
( ) ( ) ( )
Hence, a × b × c ,b × c × a and c × a × b are coplanar. hence linearly dependent

which Proves (i) & (iv).


    
Q.
 a·b = 1 and a × b = j − k , then find b .
If a = i + j + k,
Vector & Three-Dimensional Geometry

        
Sol. ( )
As a × b × a = (a ⋅ a ) b − (a ⋅ b) a
   
a ⋅ a =| a |2 = 1 + 1 + 1 = 3 and a ⋅ b = 1

∴ (ˆj − k)
ˆ × (iˆ + ˆj + k)
ˆ= 3(b) − (iˆ + ˆj + k)
ˆ

ˆi ˆj kˆ

i.e. 0 1 −= 1 3(b) − (iˆ + ˆj + k)
ˆ
1 1 1

(2iˆ − ˆj − k)
ˆ + (iˆ + ˆj + k)
ˆ =3b
i.e. î = b

76.

Q. Find a vector v which is coplanar with the vectors i + j − 2k & i − 2j + k and is

orthogonal to the vector −2iˆ + ˆj + kˆ It is given that the projection of v along
the vector ˆi − ˆj + kˆ is equal to 6 3 .


Sol. v = α(–2î + j + k)ˆ × [(î + j – 2k)
ˆ × (î – 2jˆ + k)]
ˆ

v = α( −2iˆ + ˆj + k)
ˆ × [−3î − 3ˆj − 3k]
ˆ

v = α[−9ˆj + 9k] ˆ …(1)

v ⋅ (î − ˆj + k)
ˆ
=6 3
3
⇒ α( −9ˆj + 9k) ˆ ⋅ (î − ˆj + k)
ˆ =18
⇒ α (9 + 9) = 18
α =1

From (1), v = 9( −j + k)

   ˆi − 2ˆj  2iˆ + ˆj + 3kˆ


Q. If a and b are vectors in space= given by a =
5
and b
14
, then
     
find the value of (2a + b)·[(a × b) × (a − 2b)] .

Sol. From the given information, it is clear that


 î − 2ˆj    
a= ⇒  a= 1, b= 1, a ⋅ b= 0
5
     
( ) ( ) (
Now, 2a + b ⋅  a × b × a − 2b=


) (
 2a + b
 )
  


( ) 
( 
a2b − a ⋅ b ⋅ a + 2b2 ⋅ a − 2 b ⋅ a ⋅ b)
   2 2
Vector & Three-Dimensional Geometry

= 2a + b ⋅ b + 2a  = 4a + b [as a . b = 0]
 
= 4.1 + 1 = 5
   
Q. Prove that : i × (a × i) + j × (a × j) + k × (a × k)
 =2a .

   
Sol.
Let a = a 1 i + a2 j + a3k
          
( ) ( ) ( )
i × a × i   = i ⋅ i a − i ⋅ a i = a − a 1 i …(1)
   

( )
Similarly j × a × j   = a − a2 j …(2)

77.
    
k × (a × k) = a − a3k …(3)
  

   ˆ
(1) + (2) + (3) ⇒ ˆi × (a × ˆi) + ˆj × (a × ˆj) + kˆ × (a × k)

( )   
= 3a − a 1 i + a2 j + a3k = 3a − a = 2a

Previous Year’s Question


    ˆ 2
If a = 2i + j + 2k , then the value of | ˆi × (a × ˆi) |2 + | ˆj × (a × ˆj) |2 + | kˆ × (a × k) | is equal to____
[JEE Main 2020]

         
Q. Given that a,b,p, q are four vectors such that a + b =
   
µp, b·q =

0 & (b)2 =1,

where µ is a scalar then prove that | (a·q)p − (p·q)a | =


| p·q |

Sol.
Given,
  
a +b = µp ...(i)
     
⇒ a ⋅ q + b ⋅ q =µ p ⋅ q ( )
   
⇒ a ⋅ q =µ p ⋅ q ...(ii)( )
         
Now., | (a ⋅ q)p − (p ⋅ q)a | = | (p ⋅ q) || µp − a | from (i)
  
= | (p ⋅ q) || b | from (ii)
 
= p⋅q ( )
  
Q. If the vectors b, c, d are not coplanar, then prove that the vector
           
(a × b) × (c × d) + (a × c) × (d × b) + (a × d) × (b × c) is parallel to .

Sol.
We know that
Vector & Three-Dimensional Geometry

        
(
A × B× C = B A ⋅ C − C A ⋅B ) ( ) ( )
       

a × b × c= (
×d
 ) (
  )
a c d b − bcd a

…(i)
       
) ( )
 
Similarly,for a × c × d = ×b
 (
adb c − cdb a
  
         
Similarly, for a × d × b = ×c
 (  ) (
adc  b − adb c
 )
So adding all the three equations, we get
           
( ) (  
) ( ) (
a × b × c × d + a × c × d × b + a × d × b × c = −2 bcd a
  ) ( ) ( )

Thus parallel to a .

78.
Scalar Product of four Vector :
   
            a ⋅c a ⋅d
( )( ) ( )( ) (
(I)   a × b ⋅ c × d = a ⋅ c b ⋅ d − a ⋅ d b ⋅ c =    
b⋅c b⋅d
)( )
         
( )( ) ( ) ( )
Proof: : a × b ⋅ c × d =u ⋅ c × d = u × c ⋅ d (Dot & cross are interchangeable in STP)
 

u
   
                  
(( ) )
a × b × c ⋅ d= (( ) ( ) )
a ⋅ c b − b ⋅ c a ⋅ d= (( )( ) ( )( ))
a ⋅c b⋅d − b⋅c a ⋅d =
a ⋅c
 
b⋅c
a ⋅d
 
b⋅d
   
      2 a ⋅a a ⋅b    
( )( )
a × b ⋅ a × b =(a × b) =  
a ⋅b
  =(a)2 (b)2 − (a ⋅ b)2 which is lagrange’s identity.
b ⋅b

Q. Prove that acute angle between the two plane faces of a regular tetrahedron
1
is cos−1
3

Sol. Let edge length of regular tetrahedron = 1


  
n1 = normal vector to plane OAB= a × b
  
n2 = normal vector to plane OBC= b × c
∴ acute angle between plane focus OAB & OBC is given as
     
n1 n2 | (a × b) ⋅ (b × c) |
cos θ =
=     
n1 n2 | (a × b) || (b × c) |
   
a ⋅b a ⋅ c
   
b ⋅b b ⋅ c
=
sin60° ⋅ sin60°
cos 60° cos 60°
cos 0° cos 60°
=
3/4
1 1 Vector & Three-Dimensional Geometry

4 2 1
= =
3 3
4

1
⇒ θ =cos −1  
3

79.
Q. Prove that the angle between any edge and a face of a regular tetrahedral is
1
cos−1
3

Sol. Let us find angle between edge OA a face OBC, since tetrahedral is regular so

each face is equilateral triangle required angle is angle between vector a and

 
vector along angle bisector let a,b, c be unit vector Angle bisector of ∠BOC in
  
  (
a⋅ b+c
△0BC is b + c ∴ cosθ =   
)
a b+c
   
a ⋅b + a ⋅c
⇒ cosθ =   
|a| b+c
   
a ⋅b + a ⋅c
⇒ cosθ =  
|b+c|
  
Now | b+ c |2 = b2 + c2 + 2b.c = 1 + 1 + 2cos 60°
 
| b+ c |=3
1.1 × cos60° + 1.1 .cos60o
⇒ cosθ =
3
1
⇒ θ =cos−1
3
Hence the statement is proved.

       
Q. Prove that : {(a × b) × (a × c)}·d =
(a·d)[abc]

  
Vector & Three-Dimensional Geometry

Sol. a × b =λ
               
∴ [(a × b) × (a × c)]·d = [ λ × (a × c)]·d = [( λ.c)a – (–λ.a)c]·d
           
= [{(a × b) ⋅ c}a − {(a × b) ⋅ a} c] ⋅ d = (a ⋅ d)[abc]

       
Q. Prove that : d·[a × {b × (c × d)}] =
(b·d)[acd] .

     
Sol. ( ) ( )( ) ( )( )
b × c × d  = b ⋅ d c − b ⋅ c d
 
  

 
Now crossing with a from left , we have a × b × c × d 
 ( )

80.
     


( ) ( ) ( )( )
=a ×  b ⋅ d c − b ⋅ c d 
 
    


( )( )  
( )( )
=a ×  b ⋅ d c  − a ×  b ⋅ c d 
   
       

( ) ( ) ( )( )
= b⋅d a×c − b⋅c a×d
                 
∴ d ⋅ a × b × ( c × d )   = ( b ⋅ d ) d ⋅ ( a × c )  − (b ⋅ c ) ( d ⋅ a × d ) = (b ⋅ d) dac 
         
  
= (b ⋅ d) acd

Vector Product of Four Vector :
    
V = (a × b) × (c × d)
         
=u × (c × d) =[a b d]c − [a b c]d …(1) (where u= a × b)
              
Again V = (a × b) × (c ×
d) = (a ⋅ v)b − (b ⋅ v)a = [a c d]b − [b c d]a …(2)

v
       
From (1) and (2), [a b d]c − [a b c]d = [a c d]b − [b c d]a …(3)
   
 
( ) ( )
Note that a × b × c × d = 0 ⇒ planes containing the vectors a & b
 
and c & d are parallel.
   
( )( )
Similarly, a × b ⋅ c × d = 0 ⇒ the two planes are perpendicular.
   
(i) Equation (3) is suggestive that if a,b, c, d are four vectors, no three of
them are coplanar then each one of them can be expressed as a linear
combination of other.
   
(ii) If a,b, c, d are p.v.’s of four points then these four points are in the
same plane if
   
a b d  −  a b c  =  a c d  −  b c d
       

         Vector & Three-Dimensional Geometry


Q.
If a,b, c are three vectors such that a × b= c,b × c= a , then
           
(A)= b 1,= c a (B)= c 1,=a b (C) = b 2, = c 2 a (D)=
a 1,=
b c

Sol. (A)
  
Given a × b = c ...(i)
  
b×c = a …(ii)
   
From (i), c ^ a and c ^ b …(iii)
   
From (ii), a ^ b and a ^ c …(iv)

81.
  
From (iii) and (iv), a,b, c are mutually perpedicular.
Taking cross product of (i) with (ii), we get,
         
( ) ( )
 
a × b × b × c =c × a ⇒  a b c  b − a bb c =
   
c×a
   
⇒ a b c  b= c × a
 
       
⇒   a b c  b =  c × a  ⇒ [a b c] b = | c × a
 
       2 
⇒  a b c b = c a  ⇒ | b |= 1 ⇒ b = 1
  
From (i), a × b = c
       
⇒ a × b = c ⇒ a b = c   [ a and b are mutually perpendicular]
   

=  a c   =  b 1
 
  
 b = 1 and a = c

Condition for coplanarity of four points :


  
4 points with pv’s a,b, c, d are coplanar if ∃ scalars x, y,
z and t not all simultaneously zero and satisfying
 
xa + yb + zc + td =
0 where x + y + z + t =0

Case I:
Let the four points A, B, C, D are in the same plane
     
⇒ the vectors b − a, c − a and d − a are in the
same plane.
     
hence d − a = l(b − a) + m(c − a)
Vector & Three-Dimensional Geometry

or
       
(
l + m − 1) a − l b − mc

1 d = 0 ⇒ xa + yb + zc + td = 0
 +
y z
x i

where, x + y + z + t =0 and
x, y, z, t not all simultaneous zero.

Case II:  
 
Let xa + yb + zc + td =
0 where x + y + z + t =0 and not
all simultaneously zero

82.
Let t ≠ 0
   
( −y − z − t ) a + yb + zc + td =0 
[putting x =− y − z − t ]
     
( ) (
d−a t + y b−a +z c−a = 0 ) ( )
    
⇒ d − a,b − a and c − a are coplanar points
A, B, C, D are coplanar.

Theorem in space:


 
If a,b, c are 3 non zero non coplanar vectors

then any vector r can be
expressed as a linearcombination :
    
r = xa + yb + zc of a,b, c
Example:
  
Express the non coplanar vectors a,b, c in
     
terms of b × c, c × a, a × b
      
( )(
Since [a b c]2 = a × b b × c c × a 
  )( )
  
∴   If a,b, c are non-coplanar
     
⇒  a × b,b × c, c × a are also non coplanar.
   

( ) (
a= x a × b + y b × c + z c × a ) (
 
)

Taking dot product with a

2  (a)2
= a 
y abc =  ⇒y 
  a b c 
 

Taking dot product with b

Vector & Three-Dimensional Geometry


 
   a ⋅b
=a ⋅ b z b c a  ⇒=  z  
  a b c 
 

Similarly taking dot product with c
 
   a ⋅c
=a ⋅ c x a b c = ⇒x 
  a b c 
 
     2      
 ( )( ) (
a ⋅ c a × b + (a) b × c + a ⋅ b c × a ) ( )( )
a= 
a b c 
 

83.
  
Q. If a,b and c are three non-coplanar vectors, then prove that
  
 a·d   b·d   c·d  
=d    (b × c) +    (c × a) +    (a × b)
[abc] [abc] [abc]

        
Sol. Since a,b and c are non-coplanar, vectors a × b,b × c and c × a are also non-
coplanar.
        
d = l(b × c) + m(c × a) + n(a × b) ...(i)

Now multiplying both sides of (i) scalarly by a , we have
            
a·d = la·(b × c) + ma·(c × a) + na·(a × b) = l[abc]  [aca] = 0 = [aab]
 
⇒l = (a·d) / [abc]
 
Similarly, multiplying (i) scalarly by b and c successively, we get
   

= m (b·d)
= / [abc] and n (c·d) / [abc]
Putting these values of l, m and n in (i), we get the required relation.

Q. Show that the p.v. of circumcentre of a tetrahedron OABC is (where ‘O’ is the
origin)
     
a 2 (b × c) + b2 (c × a) + c2 (a × b)
  
2[a b c]

Sol. If the centre ‘P’ has position vector, r then
       
a −=r PA,b −= r PB, c −=
r PC
    
where PA = PB = PC = OP = r
  
Consider a − r =r
        
⇒ a−r ⋅a−r = rr ⇒ a2 − 2a ⋅ r + r2 = r2 ⇒ a2 = 2a ⋅ r
Vector & Three-Dimensional Geometry

2
  2  
Similarly, b= 2b ⋅ r and c= 2c ⋅ r
     
( )( ) (
Since b × c , c × a and a × b are non-co-planar, then)
      
( ) (
r= x b × c + y (c × a ) + z a × b )
     
⇒ a= ( )
⋅ r xa. b × c + y ⋅ 0 + z=⋅ 0 x abc 
 
  2
a⋅r a
⇒ =x  =
 
abc  2 abc 
   

84.
b2 c2
Similarly, y =

  and z = 
2 abc  2 abc 
    Point to Remember!!!
  
2 2 
 a b×c +b c×a + c a ×b
Hence r =

( )2 
( ) ( ) (i) 2
 non zero, non collinear vectors

2 abc  are linearly independent.
 
(ii) T
 hree non zero, non coplanar
Real definition of linearly independence : vectors are linearly
  
If V1 , V2 , …… ..Vn are vectors and λ 1 , λ2 , …… ..λn independent i.e.
   
a b c  ≠ 0 ⇔ a,b, c are linearly
are scalar and if the linear
 combination  
λ 1V1 + λ2 V2 + …… .. + λnVn =0 , necessarily independent.
implies λ 1 = λ2 = …… .λn = 0 , we say that (iii) 
Four or more vectors in 3D
  
V1 , V2 , …… ..Vn are said to constitutes a space are always linearly
dependent.
linearly independent set of vectors.

        
Q. Show that vectors i − 3 j + 2k, 2i − 4 j − k and 3i + 2 j − k are linearly independent.

           
Sol. Let a =i − 3 j + 2k,b =2i − 4 j − k and c = 3i + 2 j − k
1
−3 2
  
a b c = 2 −4 −1 ≠ 0
 
3 2 −1

        
Q. If a,b and c are non-coplanar vectors, then prove that (a·d)(b × c) + (b·d)(c × a)
    
+(c·d)(a × b) is independent of d , where d is a unit vector.

   
Sol. Given [abc] ≠ 0 as a,b, c are non-coplanar. Also there does not exist any linear relation
between them because if any such relation exists, then they would be coplanar.
       Vector & Three-Dimensional Geometry
Let A = x(b × c) + y(c × a) + z(a × b)
  
where
= x a·d,
= y b·d,
= z c·d
  
We have to find the value of modulus of A, i.e., | A |, which is independent of d .
  
Multiplying both sides scalarly by a,b and c and we know that scalar triple product is
zero when two vectors are equal.
  
A·a x[abc] + 0
=
Putting for x, we get

85.
    
(a·d)[abc] = A·a
Similarly, we have
    
(b·d)[abc] = A·b
    
(c·d][abc] = A·c
Adding the above relations, we get
              
[(a + b + c)·d][abc]
= A·(a + b + c) or (a + b + c)·[d[abc] − A] =0
     
Since a,b and c are non-coplanar, a + b + c ≠ 0 because otherwise any one is
expressible as a linear combination of other two.
  
Hence [abc]d = A
  
| A |=| [abc] | as d is a unit vector.

It is independent of d .
Reciprocal system of vectors:
     
1. If a,b, c and a ',b ', c' are 2 sets of non coplanar vectors such that
                 
a ⋅ a' = b ⋅ b' = c ⋅ c' = 1 and a ⋅ b' = a ⋅ c' = b ⋅ a' = b ⋅ c' = c ⋅ a' = c ⋅ b' = 0 ,
     
then a,b, c and a',b', c' are said to be constitute a reciprocal system of
vectors.
2. Reciprocal system of vectors exists only in case of dot product.
     
3. It is possible to define a ',b ', c ' in terms of a,b, c as
     
 b × c  c × a  a ×b 

= a′ =    ;b′ =   ; c′    ([a b c] ≠ 0)
[a b c] [a b c] [a b c]
        
      a × (b × c) + b × (c × a) + c × (a × b)
Note: (i) a × a '+ b × b '+ c × c ' = 0 i. e. 
[a b c]
       
( )
(ii) (a + b + c) ⋅ a′ + b′ + c′ = 3 (  
as a ⋅ b′ = a ⋅ c′ = 0 etc )
Vector & Three-Dimensional Geometry

    1    
(iii) If [a b c] =V then a′ b′ c′  = ⇒ [abc] a′ b′ c′  =
1
  v  
  
′ ′ ′ ′ ′ ′ a + b + c 
(iv) a × b + = b ×c +c ×a    , [a b c] ≠ 0
[a b c]

Q.
(i) Find the set of vectors reciprocal to the set of vectors
2i + 3 j − k,
 i − j − 2k , −i + 2 j + 2k

Sol. Let the given vectors be a, b, c

86.
2 3 −1
Now, abc = 1 −1 −2
−1 2 2

= 2(–2 + 4) – 3(2 – 2) – 1(2 – 1)


=4–1=3
i j k
 
b×c = 1 −1 −2 = 2i + k
−1 2 2

i j k
 
c × a =−1 2 2 =−8i + 3 j − 7k ,
2 3 −1

i j k
 
a × b =2 3 −1 =−7i + 3 j − 5k
1 −1 −2

b×c 2i + k
Hence, =a′ =
[abc] 3
c×a −8iˆ + 3 j − 7k
=b' =
[abc] 3

a × b −7i + 3 j − 5k
and =c' =
[abc] 3

(ii) Find a set of vector reciprocal to the vectors a, b and a × b.

Sol. Let the given vectors be denoted by a, b and c where c = a × b.


Vector & Three-Dimensional Geometry
         2
( )
∴ [abc] = (a × b)·c = (a × b) = . a × b =| a × b | …(i)
and let the
 reciprocal
  system of vector be a’, b’ and c’.

b×c b × (a × b)
∴ a= '   =  
[a b c] | a × b |2
    
c×a (a × b) × a
= b' = 
   
[a b c] | a × b |2
   
a ×b a ×b
=c' =  
[a b c] | a × b | 2
      
∴ a ', b' and c' are required reciprocal system of vectors for a, b and a × b .

87.
     
 b×c  c×a a ×b
(iii) If a '
= =   ,b' =
[a b c]
   , c'
[a b c]
   then show that
[a b c]
        
a × a ' + b × b' + c × c' =0 , where a, b and c are non-coplanar.

        
  a × (b × c)   (a·c)b − (a·b)c
Sol. Here, a × a ' =   
[a b c]
a ×a' = 
[abc]
…(i)
     
  (b·a)c − (b·c)a
Similarly, b × b' =  …(ii)
[abc]
     
  (c·b)a − (c·a)b
c × c' =  …(iii)
[abc]
                 
      (a·c)b − (a·b)c + (b·a)c − (b·c)a + (c.b)a − (c.a)b
= a × a ' + b × b' + c × c' =  0
[abc]
   
( a·b = b·a)
Isolating an known vectors
Satisfying a given relationship with some known vectors:
There is no general method for solving such equations, however dot or
 
cross with known or unknown vectors or dot with a × b , generally
isolates the unknown vector. Use of linear combination also proves to
be advantageous.

       
Q. Find vector  r  if r ⋅ a =
m and r × b =c, where a ⋅ b ≠ 0 .

 
Sol. r ⋅a =
  
m …(i)
and r ×b = c …(ii)
  
( )  
From (ii), a × r × b = a × c
       
( ) ( )
Vector & Three-Dimensional Geometry

Or a ⋅ b r − a ⋅ r b =a × c
          
Or ( ) ( )
a ⋅ b r = a × c + a ⋅ r b = a × c + mb

1   
  ( a × c + mb )

∴ =r
a ⋅b

      
Q. Find r such that tr + r × a =b, where a &  b are non collinear vectors.

   
Sol.
Given, tr + r × a =
   
b …(i)

Since a,b, a × b are non-coplanar vectors therefore r can be expressed as linear

88.
   
combination of a,b and a × b
  
 
Let  r = xa + yb + z a × b ...(ii) ( )

Putting the value of r in (i), we get
        
  
( ) (
t xa + yb + z a × b  + x a × a + y b × a + z a × b × a =
 
b ) ( ) ( )
       


 
 ( 

)
 
(
t xa + yb + z a × b  + y b × a + z  a ⋅ a b − a ⋅ b a  =

b ) ( ) ( )
     
(  
)
⇒  tx − z a ⋅ b  a + ty + z a ⋅ a − 1 b + tz − y a × b =
   
0 ( ) ( )( )
   
Equating the coefficients of a,b and a × b , we get
 
(
tx − z a ⋅ b = )
0 …(iii)

ty + za2 − 1 =0 … (iv)
0 …(v)
tz − y =
Solving (iii), (iv) and (v), we get
 
a ⋅b t 1
=x = ,  y = ,  z
2
t t +a ( 2 2
t +a 2
) t + a2
2

Putting the values of x, y, z in (ii), we have




=

r
1  (
 a ⋅ b a )
  
+ tb + a × b
t2 + a2  t 
 
Second method:
   
Given tr + r × a = b ...(i)
   
∴ r × a = b − tr …(ii)

Taking cross product of both sides of (i) with a , we get
        
( ) ( )
t r × a + r × a × a = b × a  [Putting the value of r × a ]
   
Vector & Three-Dimensional Geometry
( 
) (
  
⇒ t b − tr + r ⋅ a a − a2r =b × a )
     
( 
) ( )
⇒ tb − t 2 + a2 r + r ⋅ a a =b × a …(iii)

In (i), taking dot product with a , we get
      
( )   a ×b
r. r × a = 0 = b × a  ∴ r × a =
t
 
  a ×b   
From (ii), tb − t 2 + a2 r + ( t
a= )
b×a
    
 1 a ×b 

= r  a + tb + a × b
t 2 + a2  t 

89.
       
Q. Solve the following simultaneous equations x and y , x + y= a, x × y= b and
 
x⋅a = 1

  
Sol. Given
 
x+y =

a ...(i)
x×y = b ...(ii)
 
x⋅a = 1 ...(iii)

Putting the value of y from (i) in (ii), we get
   
x × (a − x ) = b
  
⇒  x × a = b
    
⇒  a × ( x × a ) = a × b
         
⇒  a2 x − ( a ⋅ x ) a =a × b ⇒  a2 x − a =
a ×b

1    1   


( )    
(
∴ x = 2 a + a × b   and  y = a − x = a − 2 a + a × b
a a
)
      
Q. (i) Solve the vector
 equation r × b = a × b, r.c = 0 provided that
c is not
perpendicular to b .

Sol.
We are given,
   
r ×b = a ×b
  
⇒ ( r − a ) × b = 0
  
Hence, ( r − a ) and b are parallel.
 
⇒ r −a = tb …(i)
 
and we know r.c = 0 
∴ Taking dot product of Equation (i) by c , we get
     
⇒ r.a − a.c = t b.c ( )
Vector & Three-Dimensional Geometry

   
⇒ 0 − a.c =
 
t b.c ( )
 a·c 
⇒ t = −     …(ii)
 b·c 

∴ From Eqs. (i) and (ii) solution of r is
  
   a·c 
r= a −     b
 b·c 

90.
       
Q. (ii) Solve for x , such that A.X = C and A × X =
B with C ≠ 0.
  
Sol.
We have, A × X = B

Taking vector product of both sides with A , we get
   
A ×B = A ×(A × X)
    
= ( A ⋅ X )A − ( A ⋅ A)X
       
= CA − | A | 2 X (using A
= ⋅ X C and A
= ⋅ A | A |2 )
   
⇒| A | 2 X = CA − A × B
  
 CA + B × A
or X= 
| A |2

     
Q. (iii) Solve the vecor equation r × a + kr =b , where a and b are two given
vector and k is any scalar.

   
Sol. Since, a, b and a × b are two non-coplanar vectors.
    
r = xa + yb + z(a × b) …(i)
(where, x, y and z scalars)
   
On putting r in r × a + kr ' = b , we get
         
{ xa + yb + z(a × b)} × a + k{ xa + yb + z(a × b)} = b
            
( )
⇒ y b × a + z{ ( a ⋅ a ) b − (a ⋅ b)a} + k{ xa + yb + z(a × b)} =
b
   
⇒ kx − z(a ⋅ b)= 0, ky + z(a ⋅ a)= 1
⇒ – y + zk = 0
On solving these equations, we get
1
z= 2 
k + | a |2
 
Vector & Three-Dimensional Geometry
a·b
x= 
(
k | a | 2 +k 2 )
k
and y = 
k + | a |2
2

On putting theses value in Eq. (i), we get the solution.


 
(a·b) k 1  
r=  2 + 2  2 (b) + 2  2 (a × b)
(2
k k +|a| k +|a| ) k +|a|
   
1  (a·b)a   

= r   + (k )b + (a × b) 
k 2 + | a |2  k 

91.
       
Q. (iv) Solve for vector A and B, where A + B= a, A × B= b, A ⋅ a= 1 .
  
Sol.
We have, A + B =
     
a
⇒ A ⋅ a + B ⋅ a = a ⋅ a
   2  
⇒ 1 + B ⋅ a = | a | (given A.a = 1 )
  
⇒ B ⋅ a= | a | 2 −1 …(i)
  
Also, A × B = b
    
⇒ a × ( A × B =a × b )
       
⇒ (a ⋅ B)A − (a ⋅ A)B =a × b
    
⇒ (| a | 2 −1 ) A − B = a × b [using Eq. (i) and a. A = 1 ] …(ii)
  
and A + B = a
From Eqs. (i) and (ii), we get
 
 (a × b) + a    (a × b) + a 
A=  and B= a −   2 
| a |2  | a | 
    2
⇒ B=
(
 (b × a) + a | a | −1 )

| a |2
    2
Thus A
= =

 (a × b) + a  (b × a)a | a | −1
and B
( )
 
| a |2 | a |2

        
Q. If=

r·a 0,
= r·b 1 and [rab] =1, a·b ≠ 0, (a·b)2 − | a |2 | b |2 =1 , then find r īn terms of
a and .

         
Sol. Writing r as linear combination of a,b and a × b , we have r = xa + yb + z(a × b)
For scalars x, y and z


 
= x | a |2 + ya·b
0 r·a

(taking dot product with a )
Vector & Three-Dimensional Geometry

=
   
= = xa·b + y | b |2
1 r·b (taking dot product with b )
 
| a |2  a·b 
Solving, we get y =  2  2   2 == – | a |2 and x = 2  2  2 a·b
| a | | b | –(a·b) (a·b) − | a | | b |
    
Also 1 =

[rab] =z | a × b |2 (taking dot product with a ×b)
1
⇒z=
| a × b |2
   
   2    


( )a ×b a ×b
thus r = (a·b)a − | a | b +   2 = a × (a × b) +   2
| a ×b | | a ×b |

92.
Previous Year’s Question

Incident ray is along the unit vector v̂ and the


reflected ray is along the unit vector ŵ . The normal
is along unit vector â outwards. Express ŵ in
terms of â and v̂ .
[JEE 2005 (Mains), 2 + 4 out of 60]

     
     p2 b + (b·a)a − p(b × a)
Q. If px + (x × a)= b; (p ≠ 0) prove that x =  2 .
2
(
p p +|a| )
   
Sol. (
px + x × a = )
b …(i)
       

( ) (
p x ⋅a + x ×a ⋅a = a ⋅b

)
(Take dot produt with a )
   
p(x · a) + 0 =
a ·b

 
  a ⋅b
x ⋅ a = …(ii)
p
       

( ) ( )
a × px + a × x × a = a × b (Take cross product with a both side)
        
( ) ( ) ( )
p a × x + a ⋅ a x − a.x a = a × b
      

( )  
p px − b + | a |2 x − a ⋅ x a =a × b ( )
[Using (I)]

    
 2  a ⋅b 
2
()
p x −p b + |a | x −
 p 
a= − b × a [Using (II)] ( )
 
 
Vector & Three-Dimensional Geometry
 a ⋅b   
(  
)
  p2 + | a |2 x= p b + 
 p 



()
a − b×a ( )
  
 
() (
p2 b + b ⋅ a a − p b × a ) ( )

 2
(
x p +|a| =
 2
) p
  
2
() (
 p b + b⋅a a −p b×a
x=
  
) ( )

(
p p2 + | a |2 )

93.
THREE DIMENSIONAL GEOMETRY
VECTORS AND SCALARS :
COORDINATES OF A POINT IN SPACE:
Consider a point P in space whose position is
given by x, y, z( ) where x, y, z are perpendicular

distance from yz plane, zx plane and xy plane


respectively.
If we assume ˆi, ˆj,kˆ unit vectors along OX, OY, OZ
respectively then the position vector of point P is
(
xˆi + yˆj + zkˆ or simply x, y, z . )
When a point lies on Co-ordinates
(
(i)  x − axis α, 0, 0 )
(ii)  y − axis ( 0, β, 0 )

(iii)  z − axis ( 0, 0, γ )

(iV) XY–plane ( α, β, 0 )

(V) XZ-plane ( α, 0, γ )

(VI) YZ–plane ( 0, β, γ )

Distance formulae:
Distance between the points x 1 , y 1 , z1 and( )
( x2 , y2 , z2 ) ( x1 − x2 ) + ( y 1 − y2 ) + (z1 − z2 )
2 2 2
=

Section formulae:
(1) Coordinates of a point P which divides line
Vector & Three-Dimensional Geometry

( ) ( )
joining A x 1 , y 1 , z1 and B x2 , y 2 , z2 in the ratio

m : n internally is given by
 mx2 + nx 1 my 2 + ny 1 mz2 + nz1 
 , , 
 m+n m+n m + n 
(2) Coordinates of a point P which divides line
( ) (
joining A x 1 , y 1 , z1 and B x2 , y 2 , z2 ) in the ratio

m : n externally is given by
 mx2 − nx 1 my 2 − ny 1 mz2 − nz1 
 , , .
 m−n m−n m − n 

94.
Direction cosines:
If α, β, γ are the angles which vector
 Point to Remember!!!
a = a 1ˆi + a2ˆj + a3kˆ makes with positive
direction of the x, y, z axes respectively then Coordinates of mid-point of line
α, β, γ are called direction angles and their ( ) ( )
joining A x 1 , y 1 , z1 and B x2 , y 2 , z2

cosines cosα, cosβ, cosγ are called the  x + x2 y 1 + y 2 z1 + z2 


is  1 , , 
direction cosines of the vector and are  2 2 2 

generally denoted l,  m , n respectively.
Thus l = cosα,m = cosβ,n = cosγ .

Note :
(i) 
If a line makes angle α, β, γ with positive
direction of x, y, z axes respectively then
direction cosines of line will be
cosα, cosβ, cosγ or −cosα, −cosβ, −cosγ
(ii) A unit vector along the line whose direction
cosines are cosα, cosβ, cosγ can be written
( ) ( ) (
as cosα ˆi + cosβ ˆj + cosγ kˆ . )

(iii) If a vector a = a 1ˆi + a2ˆj + a3kˆ makes angles
α, β, γ with positive direction of x, y, z axes
respectively then
 
a ⋅ ˆi a1 a ⋅ ˆj a2
cos
= α =  , cos
= β =  and
a ˆi a a ˆj a

a ⋅ kˆ a3
cos
= γ = 
a ˆj a

Vector & Three-Dimensional Geometry


a21 + a22 + a23
∴ cos2 α + cos=
2
β + cos2 γ  2 ⇒ cos2 α + cos=
2
β + cos2 γ 1
|a|
Also note that sin2 α + sin2β + sin2 γ = 2
( ) (
(iV) Direction cosines of x -axis are 1, 0, 0 or −1, 0, 0 . )
Direction cosines of y -axis are ( 0, 1, 0 ) or ( 0, −1, 0 ) .

Direction cosines of z-axis are ( 0, 0, 1) or ( 0, 0, −1) .

95.
Direction ratios :
If a, b, c are three numbers proportional to the direction cosines l, m,
n of a straight line, then a, b, c are called its direction ratios. They are
also called direction numbers or direction components.
l m n
Hence, we have = = = λ( say ) ⇒ l = aλ,m = bλ,n = cλ
a b c
( )
 l 2 + m2 + n2 = 1 ⇒ a2 + b2 + c2 λ2 = 1 ⇒ λ = ±
1
a2 + b2 + c2
a b c
∴ l =± ,m =±  and n =±
2 2 2 2 2 2
a +b +c a +b +c a + b2 + c2
2

Note :
(i) Direction ratios of a line is not unique but Point to Remember!!!
infinite in number but direction cosines will
be for a line will be only two. (l, m, n or –l,– A vector along the line with
m,–n) direction ratios a,b, c can be
(ii) Direction ratios of a line joining two points A
aiˆ + bjˆ + ckˆ .
and B are proportional to x2 − x 1 , y 2 − y 1 , z2 − z1 .
(iii) Projection of a Point on a Line:
Let P be a point and AB be a given line. Draw
perpendicular PQ from P on AB which meets
it at Q. This point Q is called projection of P
on the line AB.
(iV) Projection of a Line Segment Joining Two
Points on a Line:
 Projection of the line segment joining two
(
points P x 1 , y 1 , z1 ) (
and Q x2 , y 2 , z2 on )
another line whose direction cosines are l,
m, n is AB= l ( x2 − x 1 ) + m ( y2 − y 1 ) + n ( z2 − z1 ) .
Vector & Three-Dimensional Geometry

Proof:

( ) ( ) (
Vector PQ = x2 − x 1 ˆi + y 2 − y 1 j + z2 − z1 kˆ )
A unit vector along another line â =lˆi + mˆj + nkˆ
∴ Projection AB = Projection of

 PQ ⋅ aˆ
PQ on â =

( ) ( )
= l x2 − x 1 + m y 2 − y 1 + n z2 − z1 ( )

96.
Q. Find the D.C. of the vector 2i − 2j − k


Sol. Let a = 2î − 2ˆj − kˆ

| a | (2)2 + ( −2)2 + ( −=
= 1)2 3
 2 −2 −1
D.C. of a are , , (True)
3 3 3

Q. Find the locus of all points P for which OP represents a vector with direction
1
cosine cos a = (O is origin).
2

Sol. From the diagram, it is obvious that locus is a cone concentric with the positive
x-axis having vertex at the origin and the slant height equal to the magnitude of
the vector.

1 1
Q. Find the number of unit vectors with
= cos α
2
and
= cos β
2
.

Sol. cos2 α + cos2 β + cos2 γ = 1


1 1
or + + cos2 γ =1
4 4
1 1
or cos2 γ = 1 − =
2 2
1
∴ cos γ = ±
2
Two value of cos γ are possible. Hence, two vectors are possible, one
1 1 1
corresponding to direction cosines , , and other corresponding to
2 2 2

direction cosines
1 1
, −
1
. Vector & Three-Dimensional Geometry
2 2 2

Q. Find the number of vector with direction angles a = 30° and b = 30°.

Sol. cos2 α + cos2 β + cos2 γ = 1


cos2 30° + cos2 30° + cos2 γ =1
2 2
 3  3
  +  + cos2 γ =1
 2   2 
   

97.
3 3 −1
cos2 γ = 1 − 2   = 1 − =
4 2 2
Not possible
∴ Required number of vectors is zero.

Angle between two lines :


If direction ratios of two lines are a 1 ,b1 , c1
and a2 ,b2 , c2 then acute angle between two
lines is given by
a 1a2 + b1b2 + c1c2
cosθ =
a21 + b21 + c21 a22 + b22 + c22
Point to Remember!!!
Proof:
Vector along lines can be taken as (i) If lines are perpendiculars

a = a 1ˆi + b1ˆj + c1kˆ and b = a2ˆi + b2ˆj + c2kˆ . (i.e. vectors along them are
Acute angle between lines = acute angle also perpendicular) then
 
between vectors a and b . a 1a2 + b1b2 + c1c2 =
0 or
 
a ⋅b a 1a2 + b1  b2 + c1c2 l 1l 2 + m1m2 + n1n2 =
0
∴ cos
= θ =
a b a21 + b21 + c21 a22 + b22 + c22 (ii) If lines are parallel (i.e.
vectors along them are also
If direction cosines of lines are l 1 ,  m1 ,n1 and a1 b1 c1
parallel) then = =
l 2 ,  m2 ,n2 then acute angle between them is a2  b2 c2

given by cos=
θ l 1l 2 + m1m2 + n1n2 l1 m1 n1
or= = .
l2  m2 n2

Q. (
Find the coordinates of the point which divides the line joining points 2, 3, 4 )
Vector & Three-Dimensional Geometry

( )
and 3, −4, 7 in ratio 5 : 3 internally.

( )
2 3 +3 5 ()
21
Sol. (
Let the coordinates of the required point be x, y,= )
z then x =
3+5 8
;

=y
( )
3 3 −4 5
= =
()
−11
;z
4 3 +7 5
=
( )
47 ()
3+5 8 3+5 8
 21 −11 47 
∴ The required point is  , , .
8 8 8 

98.
1 1
Q. Find unit vector(s) with cosα =
2
and cosβ = , where α, β are angles made
2
by unit vector with positive direction x, y axes respectively.

Sol. Let unit vectors makes angle γ with positive z-axis.


∴ Unit vector will be ( cosα ) ˆi + ( cosβ ) ˆj + ( cosγ ) kˆ
1 1
 cos2 α + cos2β + cos2 γ = 1 ⇒ + + cos2 γ = 1
4 4
1 1
⇒ cos2 γ = ⇒ cosγ = ±
2 2
ˆi ˆj kˆ
∴ Unit vector will be: + ±
2 2 2

Q. Find the direction cosines of two lines which are connected by the relations
l − 5m + 3n =0 and 7l 2 + 5m2 − 3n2 =0

Sol. The given relations are l − 5m + 3n = 0 ⇒ l = 5m − 3n …(1)


and 7l + 5m2 − 3n2 =
2
0 …… . ( 2 )

Putting the value of l from ( 1) in ( 2 ) , we get

7(5m − 3n)2 + 5m2 − 3n2 =


0
m 1 2
( )(
or 2m − n 3m − 2n =0 ⇒ ) n
= or
2 3
m 1 l
When = i.e. n =2m ⇒ l =5m − 3n =−m or = −1
n 2 m
m 1 l l m n
thus = and = −1 giving = =
n 2 m −1 1 2

(l 2
+ m2 + n2 )=
Vector & Three-Dimensional Geometry
l m n 1
or = = =
−1 1 2
{(−1) 2 2
+1 +2 }
2 6

−1 1 2
So, direction cosines of one line are , , .
6 6 6
m 2
Again when =
n 3
l 1 l m n 1 1
⇒ =giving = = = =
m 2 1 2 3 2
1 +2 +32 2 14
1 2 3
∴ The direction cosines of the other line are , ,
14 14 14

99.
Q. Find the direction ratios and direction cosines of the line joining the points
( )
A 6, −7, −1 and B 2, −3, 1 . ( )
Sol. Direction ratios of AB (
are 4, −4, −2 = ) ( 2, −2, −1)
a2 + b2 + c2 =
9
 2 2 1
Direction cosines are  ± ,  ,  
 3 3 3 

Q. Find direction cosines of a line perpendicular to two lines whose direction


ratios are 1, 2, 3 and −2, 1, 4.

 
Sol. Vector along lines can be taken as a =ˆi + 2ˆj + 3kˆ, b =−2ˆi + ˆj + 4kˆ
 
(
∴ a × b= 5 ˆi − 2ˆj + kˆ )
 1 −2 1   −1 2 −1 
DC’s of line   =  , ,   or  , ,
 6 6 6  6 6 6 
   

Q. Find the projection of the line segment joining the points −1, 0, 3 and 2, 5, 1 ( ) ( )
on the line whose direction ratios are 6, 2, 3 .

Sol. The direction cosines l,m,n of the line are given by

l m n l2 + m2 + n2 1 1
= = = = =
6 2 3 2 2
6 +2 +3 2 49 7
6 2 3
=l =,m = ,n
7 7 7
∴ The required projection is given by
( ) (
l x2 − x 1 + m y 2 − y 1 + n z2 − z1 ) ( )
Vector & Three-Dimensional Geometry

6 2 3

7
( ( ))
2 − −1 +
7
5−0 +
7
(
1−3 ) ( )
6 2 3

7
× 3 + × 5 + × −2
7 7
( )
18 10 6 18 + 10 − 6 22
+= − =
7 7 7 7 7

100.
Q. A variable line has dc’s l, m, n and l + dl, m + dm, n + dn in two adjacent posi-
tions. If dq be the angle between the lines in these two positions then prove
that (dq)2 = (dl)2 + (dm)2 + (dn)2

Sol.
Direction cosines are l, m, n & (l + δl), (m + δm), (n + δn)
Hence l2 + m2 + n2 = 1 and (l + δl)2 + ( m + δm)2 + (n + δn)2 = 1
l2 + (δl)2 + 2 lδl + m2 + (δm)2 + 2mδm + n2 + (δn)2 + 2nδn = 1
(δl)2 + 2ldl + (δm)2 + 2mδm + (δn)2 + 2nδn = 1 – (l2 + m2 + n2) = 1 – 1 = 0
(δl)2 + (δm)2 + (δn)2 = – 2(lδl + mδm + nδn)
Now cos δθ = l(l + δl) + m(m + δm) + n(n + δn)
= l2 + m2 + n2 + lδl + mδm + nδn
 −1 
= 1 +   (dl )2 + (dm)2 + (dn)2 
 2  
1 dθ
[(dl)2 + (δm)2 + (δn)2] = 1 – cosδθ = 2 sin2
2 2
dθ dθ
Since δθ is very small, so sin ≈
2 2
1 2(dθ)2
We get (dl)2 + (dm)2 + (dn)2  =
2  4
⇒ (δθ)2 = (δl)2 + (δm)2 + (δn)2 Hence proved.

Q. Find the direction cosines of a line perpendicular to two lines whose dr’s are 1,
2, 3 and –2, 1, 4

Sol. dr’s of one line, 1, 2, 3


dr’s of other line, –2, 1, 4
vector along the line whose D.R. are 1, 2, 3 =ˆi + 2ˆj + 3kˆ
vector along the line whose D.R. are –2, 1, 4 =−2iˆ + ˆi + 4ˆj
vector along the line which is ⊥ar to both given lines is 5iˆ − 10ˆj + 5kˆ Vector & Three-Dimensional Geometry
So, D.R. are (5, –10, 5)
2
52 + ( −10 ) + 52 =5 6
5 −10 5
So, D.C. are , ,
5 6 5 6 5 6
 1 −2 1 
 , , 
 6 6 6

101.
Q. A line makes angle, α, β, γ and δ with the four diagonals of cube, prove that
4
cos2 α + cos2 β + cos2 γ + cos2 d =
3

Sol. Let the cube be shown in the figure, where four diayonals are OP, AL, BM and
CN and A(a, 0,0), B(0, a, 0), C(0,0, a), (0, a, a), M(a, 0, a), N(a, a, 0) and P(a, a, a),
hence direction cosines of OP are
a a a
, ,
a2 + a2 + a2 a2 + a2 + a2 a2 + a2 + a2
 1 1 1 
= , ,
 3 3 3 
 
a

 1 1 1 
TheDC′s of AL are  – , , 
 3 3 3 

 1 1 1 
TheDC′s of BM are  ,– , 
 3 3 3 

 1 1 1 
TheDC′s of CN are  , ,- 
 3 3 3 

Let the DC’s of required line be (l, m, n).
l +m+n −l + m + n
∴ cos α
= = , cos β
3 3
l −m+n l +m−n

= cos γ = and cos d
3 3
cos2 α + cos2 β + cos2 γ + cos2 d

=
1
3
{
(l + m + n)2 + ( −l + m + n)2 + (l − m + z)2 +(l + m − n)2 }
(
4 2
)4
Vector & Three-Dimensional Geometry

= l + m2 + n2 =
3 3

102.
Planes Definition

A plane is a surface such that


General equation of plane:
a line joining any two points on
A linear equation in three variables of the type
the surface lies completely on
ax + by + cz + d = 0 represents the general
it.
equation of a plane.
where a,b, c are not simultaneously zero.
a b c Point to Remember!!!
Dividing by d we get   x +   y +   z + 1 =0.
d
  d
   d
(i) Equation of xy plane is z = 0 .
Thus equation of plane involves only three
arbitrary constants. Hence in order to determine (ii) Equation of yz plane is x = 0 .
a unique plane 3 independent conditions are (iii) Equation of zx plane is y = 0 .
needed.
Division by Coordinate Planes :
The ratios in which the line segment PQ joining
(
P x 1 , y 1 , z1 ) (
and Q x2 , y 2 , z2 ) is divided by

coordinate planes are as follows.


x
(i) by yz -plane  : − 1 ratio
x2
y1
(ii) by zx − plane : − ratio
y2
z1
(iii) by xy − plane : − ratio
z2

Q. ( )
Find the ratio in which the line joining the points 3, 5, −7 and −2, 1, 8 is ( )

Vector & Three-Dimensional Geometry


divided by yz-plane.

Sol. Let the line joining the points ( 3, 5, −7 ) ( )


and −2, 1, 8 is divided by yz-plane in the

 −2λ + 3 λ + 5 8λ − 7 
ratio λ : 1 , then coordinates of the dividing point will be  , , 
 λ+1 λ+1 λ+1 
Now above points lies on the yz-plane, so its x-coordinate should be zero i.e.
−2λ + 3 3
= 0 ⇒ λ=
λ+1 2
3
Hence yz-plane divides line joining the given points in the ratio : 1 or 3 : 2.
2

103.
DIFFERENT FORMS OF THE EQUATIONS OF
PLANES :
1. 
A point in the plane and a vector normal to it
is given :
Let a point A (a) lies in the plane and a vector

normal to it is n = ai + bj + ck .

 ()
P r is a moving point whose locus is plane

then for every position of vector AP , vector

n will be perpendicular to it.
 
∴ AP ⋅ n = 0
      
( )
⇒ r − a ⋅n = 0 ⇒ r ⋅n = a ⋅n
 
⇒ r ⋅ n =d is general equation of plane in
vector form.
It is also known as equation of plane in dot
(or scalar) product form.
 
If r = xi + yj + zk and a = x0i + y0 j + z0 k the
equation of plane will be
( ) (
a x − x0 + b y − y0 + c z − z0 =
0 ) ( ) Point to Remember!!!
This is equation of plane containing point
If equation of a plane is ax + by +
(
x0 , y0 , z0 )
and perpendicular to vector
cz + d = 0 then a, b, c are direction
aiˆ + bjˆ + ckˆ ratio of normal to the plane.

Plane passing through three given points :



( ) ( )
 
Let three points A a ,B b and C c lies in ()

point P ( r )
Vector & Three-Dimensional Geometry

the plane and is moving point


whose locus is plane.
  
 AP, AB and AC are coplanar.
     
∴ [r − a b − a c − a] =
0

represents equation of plane passing through
three points.
=If A = (
x 1 , y 1 , z1 ,B x=
2 )
, y 2 , z2 , C ( )
x3 , y 3 , z3 ( )
and P(x, y, z) then equation of plane is

104.
x − x1 y − y 1 z − z1
x2 − x 1 y 2 − y 1 z2 − z1 =
0
x3 − x 1 y 3 − y 1 z3 − z1

Plane containing two intersecting lines :


  
Let the equations of two lines are r = a + λp
  
and r = a + µq .
 
Now, n= p × q is a vector perpendicular to
the plane.
   
(
Hence equation of plane is r − a ⋅ p × q =
0) ( )
         
⇒ r − a  p q = 0 ⇒ r p q = a p q]
   
Since vectors r − a,p, q are coplanar.
       
Therefore r − a = αp + βq ⇒ r = a + αp + βq
It represents equation of a plane containing

point a and parallel to two non-collinear
 
vectors p and q . This is also known as
parametric equation.


Q. Express the equation of a plane r = ˆi − 2ˆj + λ(2iˆ − ˆj + 3k)
ˆ + µ(3iˆ + 4ˆj − k)
ˆ in

(a) cartesian form.


(b) Scalar product form.

Sol. (a) Clearly plane is passing through the point (1, –2, 0) and parallel to vectors

2iˆ − ˆj + 3kˆ and 3iˆ + 4ˆj − kˆ

x−1 y+2 z−0 Vector & Three-Dimensional Geometry


∴   Equation of plane is 2 −1 3 =0
3 4 −1


⇒ (x – 1)(–11) – (y + 2)(–11) + z(11) = 0
⇒ x – 1 – y – 2 – z = 0 ⇒ x – y – z = 3 ⇒ x(1) + y(–1) + z(–1) = 3

(b) Therefore equation of plane is scalar product form in r ⋅ (iˆ − ˆj − k)
ˆ =3

105.
Equation of plane containing two parallel lines :
     
Let lines be r = a + λb and r = c + µb
vector normal to plane is
   
(
n = a − c ×b )
∴   equation of plane is
    
( ) ( )
r − a ⋅ a − c ×b =0
Alternatively :
    
Vectors r − a, c − a and b are coplanar
    
∴ equation of plane is r − a c − a b =0
 

Q. Find the equation of the plane passing through the point (2, −1, 3) which is the
foot of the perpendicular drawn from the origin to the plane.

Sol. The direction ratios of the normal to the


plane are 2, −1, 3
The equation of required plane is
2(x − 2) − 1(y + 1) + 3(z − 3) = 0
⇒ 2x − y + 3z − 14 = 0

Q. ( )( )
Find the equation of the plane through 2, 3, −4 , 1, −1, 3 and parallel to x-axis.

Sol. The equation of the plane passing through ( 2, 3, −4 ) is


Vector & Three-Dimensional Geometry

a ( x − 2) + b ( y − 3) + c (z + 4 ) =
0 … (1)

Since ( 1, −1, 3) lies on it, we have a + 4b − 7c = 0 … (2)


Since required plane is parallel to x -axis i.e. perpendicular to YZ plane i.e.
b c
I ⋅ a + 0 ⋅ b + 0 ⋅ c = 0 ⇒ a = 0 ⇒ 4b − 7c = 0 ⇒ =
7 4
∴ Equation of required plane is 7y + 4z = 5.

106.
Q. Two planes are given by equations x + 2y − 3z =
0 and 2x + y + z + 3 =0. Find
(i) DC ‘s of their normals and the acute angle between them.
(ii) DC ‘s of their line of intersection.
(iii) Equation of the plane perpendicular to both of them through the point
( )
2, 2, 1

 
Sol. nl = ˆi + 2jˆ – 3k,
ˆ n = 2iˆ + ˆj + kˆ
 
2

n1 . n2 1
(i)  cos
= θ = 
n| n2 2 21

1  1 2 3 
⇒ θ =cos−1
& D.C of their normals are  ± ,± , 
2 21  14 14 14 
 2 1 1 
&  ± ,± ,± 
 6 6 6
ˆi ˆj kˆ
 
(ii) nl × n2 = 1 2 -3 = 5iˆ - 7jˆ - 3kˆ
2 1 1

 5 7 3 
DC′s of line of intersection of the plane  ± , , 
 83 83 83 

  
(iii) (r - (2iˆ + 2jˆ + k))
ˆ × (n ×n ) = 0 1 2


⇒ 5 x−2 −7 y−2 −3 z−1 =
0 ( ) ( ) ( )

⇒  5x − 7y − 3z + 7 =
0

Vector & Three-Dimensional Geometry


Normal form of the plane :
A unit vector n̂ normal to the plane from
origin is known and perpendicular distance
of the plane from the origin is d.

Projection of r on n̂ = d Point to Remember!!!

 = d … (1)
r •n
d > 0, as d is distance of the plane
from origin. Cartesian form of the
plane is lx + my + nz = d
where l, m, n are dcs of normal to
plane.

107.

Q. Let equation of plane be r ⋅ (6iˆ − 3ˆj − 2k)
ˆ +1=0 then find perpendicular
distance of plane from origin and also find direction cosines of this
perpendicular.

 ˆ = –1 ⇒ r ·  –6 ˆi + 3 ˆj + 2 kˆ  = 1
r · (6iˆ – 3jˆ – 2k)
Sol. Plane is 
 7 7 7  7

1  −6 3 2 
∴ perpendicular distance from origin = and dcs of perpendicular =  , , 
7  7 7 7

Q. Find the vector equation of plane which is at a distance of 8 units from the
origin and which is normal to the vector 2iˆ + ˆj + 2kˆ


Sol. Here, d = 8 and n = 2iˆ + ˆj + 2kˆ

n 2iˆ + ˆj + 2kˆ 2iˆ + ˆj + 2kˆ
∴ n̂ =  = =
|n| 22 + 12 + 22 3
Hence, the required equation of plane is,

ˆ=d
r ·n
  2iˆ + ˆj + 2kˆ 
⇒ r . =8
 3 
 

or r · (2iˆ + ˆj + 2k)
ˆ = 24

Intercept form the plane :


Equation of plane in the intercept form is
x y z
+ + = 1
a b c
where a = x-intercept,
b = y-intercept,
Vector & Three-Dimensional Geometry

c = z-intercept

Proof :
E
 quation of plane passing through three
( ) ( ) (
points A a, 0, 0 ,B 0,  b, 0 and C 0, 0, c will )
x−a y−0 z−0
be −a b 0 =0
−a 0 c

( ) ( )
⇒ x − a bc − y −ac − 0 + z 0 + ab =
0 ( )

108.
⇒ xbc + yac + zab =
abc
x y z
⇒ + + =1
a b c
Note :
1   1 ˆ |= 1 | bci + acj + abk |
Area of ∆ABC = | AB × BC |= | ( bj - ai)
 × (ck - bj)
2 2 2
2 2 2
1  ab   bc   ca 
= a2b2 + b2c2 + c2a=
2
  +  + 
2  2   2   2 
∴ Area of ∆ABC = ( area of ∆OAB)2 +( area of ∆OBC)2 +( area of ∆OCA)2

Examples of Different forms of Plane

Q. Find the equation of the plane through the point (2, –3, 1) and parallel to the
plane 3x – 4y + 2z = 5.

Sol. Equation of plane parallel to the plane 3x – 4y + 2z = 5 is 3x – 4y + 2z + λ = 0


It passes through the point (2, –3, 1)
then 3 × 2 + 4 × 3 + 2 × 1 + λ = 0
⇒ λ = – 20
then plane: 3x – 4y + 2z – 20 = 0

 
Q. The intercept made by the plane r · n = q on the x-axis is

q i·n  q
(A)  (B) 
(C) (i·n)q (D) 
i · n q |n|

Sol. (A)
 

Vector & Three-Dimensional Geometry
r ·n = q
x y z
If equation of plane is + + =1
a b c
then a, b, c are respectively X intercept, Y intercept, Z intercept.

q = coordinates intercept . n
q
Hence, X intercept = 
î·n

Q. Find the equation of the plane which is parallel to the plane x + 5y – 4z + 5 = 0


and the sum of whose intercepts on the co-ordinate axes is 19 units.

109.
Sol. Equation of plane parallel to the plane x + 5y – 4z + 5 = 0 is x + 5y – 4z + λ = 0
this can be written as x + 5y – 4z = – λ
– x – 5y + 4z = λ
x y z
+ + =1
 λ   λ  λ
     
 −1   −5   4 
x intercept : – λ
λ
y intercept: −
5
λ
z intercept:
4
sum of intercept s = 19
λ λ
⇒ −λ − + = 19
5 4
−20λ − 4λ + 5λ −19λ
⇒ = 19 ⇒ = 19 ⇒ λ= −20
20 20
then plane: x + 5y – 4z – 20 = 0

Q.
If from the point P (f, g, h) perpendiculars PL, PM be drawn to yz and zx planes
then the equation to the plane OLM is
x y z x y z x y z x y z
(A) + − = 0 (B) + + = 0 (C) − + = 0 (D) − + + = 0
f g h f g h f g h f g h

Sol. (A)


P(f, g, h)
yz plane x = 0 i.e. L = (0, g, h)
zx plane y = 0 i.e. M = (f, 0, h)
Vector & Three-Dimensional Geometry

O = (0, 0, 0)
Therefore, the equation of required plane is given by,
x−0 y−0 z−0
0−0 g −0 h−0 = 0
f −0 0−0 h−0

x y z
0 g h =0
f 0 h
⇒ xgh + yfh − zfg = 0
x y z
⇒ + – =0
f g h
110.
Q. The feet of normal from origin on a plane is a, b and g. Find the equation of the
plane.

Sol. n = αî + βˆj + rkˆ


  
( )
R − m n = 0

(R − m)·(αî + βˆj + rk)


ˆ =0

[(x – α )iˆ + (y – β)jˆ + (z – γ )k]


ˆ · (αî + βˆj + γk)
ˆ =0

α(x – α ) + β(y – β) + γ(z – γ ) = 0

    
Q. Consider three vectors p = i + j + k , q = 2i + 4j − k and r = i + j + 3k . If p ,  q and

r denotes the position vector of three non-collinear points then the equation
of the plane containing these points is
(A) 2x – 3y + 1 = 0 (B) x – 3y + 2z = 0
(C) 3x – y + z – 3 = 0 (D) 3x – y – 2 = 0

Sol. (D)
    
OP = p = ˆi + ˆj + k,
ˆ OQ = q ˆ OR = r = ˆi + ˆj + 3kˆ
= 2iˆ + 4ˆj − k,

x−1 y−1 z−1
Equation of plane = 1 3 −2 =
0
0 0 2
= 2(3(x – 1) – 1(y – 1)) = 0
⇒ 3x – 3 – y + 1 = 0
⇒ 3x – y – 2 = 0

Q. The cartesian equation of the plane r = (1 + λ − µ )i + (2 − λ )j + (3 − 2λ + 2µ )k , is


(A) 2x + y = 5 (B) 2x – y = 5 (C) 2x + z = 5 (D) 2x – z = 5 Vector & Three-Dimensional Geometry

Sol. (C)

We have,

r= (1 + λ − µ )i + (2 − λ ) j + (3 − 2λ + 2µ )k

⇒ r= (i + 2 j + 3k ) + λ(i − j − 2k ) + µ( −i + 2k )

Which is a plane passing through a =i + 2 j + 3k and parallel to the vectors
 
b = i − j − 2k and c =−i + 2k .

111.
  
Therefore, it is normal to the vector n =b × c =−2i − k
Hence, its vector equation is
  

(r − a ) · n = 0
   
⇒ r · n = a·n

⇒ r·( −2i − k ) =−2 − 3

⇒ r·(2i + k ) = 5
So, the cartesian equation of the plane is
  + k)
(xi + yj + zk)·(2i  = 5 ⇒ 2x + z = 5

PERPENDICULAR DISTANCE OF A POINT FROM A


PLANE :
 
Let equation of plane is r ⋅ n = d then
perpendicular distance from the point
  
( )
A r1 on the plane = projection of RA on n .
      
( r1 − r0 ) ⋅ n r1 ⋅ n − r0 ⋅ n
⇒P
= =
 
n n
 
r1 ⋅ n − d
⇒P = 
n

If equation of plane is ax + by + cz + d =
0
then perpendicular distance from point
ax 1 + by 1 + cz1 + d Point to Remember!!!
( )
x 1 , y 1 , z1 is given by
a2 + b2 + c2 (i) T
 he equation of a plane parallel
Note : to the plane ax + by + cz + d is
(i) 
Planes a 1x + b1 y + c1z + d1 =
0 and ax + by + cz + k =,
0 where k is
a2 x + b2 y + c2z + d2 =
0 are an arbitrary constant and is
Vector & Three-Dimensional Geometry

determined by the given


(a) parallel but not identical if
condition.
a1 b1 c1 d
= = ≠ 1 (ii) Distance between two parallel
a2 b2 c2 d2
planes ax + by + cz + d1 =
0
(b) perpendicular if a 1a2 + b1b2 + c1c2 =
0
and ax + by + cz + d2 =
0 is
a1 b1 c1 d1
(c) identical if = = = d1 − d2
a2 b2 c2 d2 equal to
a2 + b2 + c2

112.
(ii)  3 planes ar x + br y + cr z = dr  r = 1, 2, 3
(a) Can intersect at a point ≡ system of equations in 3 variables
having unique solution.
(b) Can intersect coaxially ≡ system of equations in 3 variables having
infinite solutions.
(c) May not have a common point ≡ system of equations in 3 variables
having no solution.

Q. ( )
A variable plane passes through a fixed point α, β, γ and meets the axes in

A,B,C. Show that the locus of the point of intersection of the planes through
A,B and C parallel to the co-ordinate planes is αx −1 + βy −1 + γz−1 = 1.

x y z
Sol. Let the equation of the variable plane be + + =
a b c
1 ... (1)

where a,b, c are parameters


The plane (1) passes through the point α, β, γ . ( )
α β γ
∴ + + =1 …(2)
a b c
The plane (1) meets the co-ordinate axes in the points A,B and C whose co-
( )( ) ( )
ordinates are respectively given by a, 0, 0 , 0,b, 0 and 0, 0, c . The equations of

the planes through A,B and C are x = a, y = b, z = c respectively … (3)


The locus of the point of intersection of these planes is obtained by
eliminating the parameters a, b, c between the equation (2), (3). Putting the
α β γ
values of a,b,c from (3) in (2), the required locus is given by + + = 1  or
x y z
 αx −1 + βy −1 + γz−1 = 1

Vector & Three-Dimensional Geometry


ANGLE BETWEEN TWO PLANES
1. Vector form :
The angle between the two planes is defined as the angle between
their normals. Let θ be the angle between planes;
 
    n ⋅n
d2 is given by cosθ =  1 2
d1 and r ⋅ n2 =
r ⋅ n1 =
n1  n2
2. Cartesian form :
The angle θ between the planes a 1x + b1 y + c1z + d1 =
0 and
a 1a2 + b1b2 + c1c2
0 is given by cosθ =
a2 x + b2 y + c2z + d2 =
a21 + b21 + c21 a22 + b22 + c22

113.
Angle between a line and a plane : Point to Remember!!!
The angle between a line and a plane is the
Two planes are perpendicular iff
complement of the angle between the line
a1a2 + b1b2 + c1c2 = 0 and Parallel if
and the normal to the plane  
n1 × n2 =0
If α, β, γ be the direction ratios of the line
and ax + by + cz + d =
0 be the equation of
plane and θ be the angle between the line
and the plane.
aα + bβ + cγ
cos(90 –q) =
a2 + b2 + c2 α2 + β2 + γ 2
aα + bβ + cγ
∴ sinq =
a + b2 + c2 α2 + β2 + γ 2
2

Vector form :
  
If θ is the angle between the line; r = a + λb
 
and plane r ⋅ n = d
 
b ⋅n
sinθ =  
b n

Q. Find the angle between the planes 2x – y + z = 11 and x + y + 2z = 3.

a 1 a 2 + b 1 b2 + c 1 c 2
Sol. cosθ =
(a 2
1 )(
+ b21 + c21 a 22 + b22 + c22 )
⇒  cosθ=
( )
2 ⋅ 1 + −1 ⋅ 1 + 1 ⋅ 2
=
1
⇒ θ=
π
22 + ( −1)2 + 12 12 + 12 + 22 2 3
Vector & Three-Dimensional Geometry

Q. Find the equation of plane passing through the intersection of planes


2x − 4y + 3z + 5 =0, x +y −z =6 and parallel to straight line having direction
(
cosines 1, −1, −1 . )

Sol. Equation of required plane be ( 2x − 4y + 3z + 5) + λ ( x + y − z − 6) =0


i.e. ( 2 + λ ) x + ( −4 + λ ) y + z ( 3 − λ ) + ( 5 − 6λ ) =0
This plane is parallel to a straight line. So, al + bm + cn =
0

114.
1 ( 2 + λ ) + ( −1 )( −4 + λ ) + ( −1 )( 3 − λ ) = 0 i.e. λ = −3
∴ Equation of required plane is −x − 7y + 6z + 23 =0.
i.e.  x + 7y − 6z − 23 =
0.

x+1 y−1 z−2


Q. Find the angle between the line =
3
=
2 4
and the plane

2x + y − 3z + 4 =0


Sol. The given line is parallel to the vector b = 3iˆ + 2jˆ + 4kˆ and the given plane is

normal to the vector n = 2iˆ + ˆj – 3kˆ
 
b·n (3iˆ + 2jˆ + 4k)
ˆ · (2iˆ + ˆj – 3k)
ˆ
sinθ =   =
| b || n | 32 + 22 + 4 2 22 + 1 2 + 32

6 + 2 − 12 −4  −4 
= =  ∴  θ sin−1 
= 
29 14 406  406 

Q. Find the equation of plane passing through (2, 2, 0) and (0, 3, 7) and parallel to
y-axis.


OP = 2 î + 2ˆj + 0kˆ
Sol. 
OQ = 0 î + 3ˆj + 7 kˆ

PQ =−2ˆj + ˆj + 7kˆ

n ^ y-axis (j)
 
n ^ PQ
 
n= ˆj × PQ
ˆi ˆj k̂ Vector & Three-Dimensional Geometry

n
= 0 1 0 = 7 î + 2kˆ
−2 1 7

R = x î + yjˆ + zkˆ

PR =( x − 2) î + ( y − 2)ˆj + zkˆ
 
Plane: PR · n = 0
(x – 2)7 + z × 2 = 0
7x – 14 + 2z = 0
7x + 2z − 14 =
0
115.
Q. If the plane 2x – 3y + 6z – 11 = 0 makes on angle sin–1(k) with x-axis, then k is
equal to
(A) 3 / 2 (B) 2/7 (C) 2 / 3 (D) 1

Sol. (B)

P : 2x – 3y + 6z – 11 = 0

n = 2 î − 3ˆj + 6kˆ (normal vector)

L : a = ˆi represents vector along x-axis. If a is angle between line and plane then

n·ˆi 2
cos(90 = − α) =

| n | | î | 7
2 2
sin α= ⇒ α= sin−1  
7 7 
Comparing with given angle in question k = =2/7

Q. A variable plane is at a constant distance p from the origin and meets the
coordinate axes in points A, B and C respectively. Through these points, planes
are drawn parallel to the coordinates planes. Find the locus of their point of
intersection.

Sol. Let us say that the line perpendicular to the plane is making α, β, γ with x, y and
z axis respectively
p
So OB = = psec β = y1
cos β
OA = psecα = n1
OC = psecγ = z1
We know that cos2α + cos2β + cos2γ = 1
p2 p2 p2
+ + 1
=
Vector & Three-Dimensional Geometry

x 21 y 21 z21

1 1 1 1
⇒ 2
+ 2
+ 2
=
x y z p2

Q. Find the equation of the plane parallel to 2x – 6y + 3z = 0 and at a distance of


2 from the point (1, 2, –3).

Sol. Equation of plane parallel to the plane 2x – 6y +3z = 0 is 2x – 6y + 3z + λ = 0

Length of perpendicular from (1, 2, –3) to 2x – 6y + 3z + λ = 0 is 2 units


2 × 1 − 6 × 2 + 3( −3) + λ
=2
4 + 36 + 9

116.
λ − 19
=2 ⇒ λ – 19 = ± 14
7
λ = + 14 + 19 or λ = 19 – 14
λ = 33 or λ = 5
2x – 6y + 3z + 33 = 0
and 2x – 6y + 3z + 5 = 0

Q. A plane which always remains at a constant distance p from the origin cuts
the co-ordinate axes at A, B, C. Find the locus of
(a) Centroid of the plane face ABC
(b) Centre of the tetrahedron OABC

x y z
Ans. (a) Suppose equation of plane is + + =
a b c
1 A(a, 0, 0), B(0, b, 0), C(0, 0, c)

Length of ^ from origin to this plane is p

−1 1 1 1 1
So
= p ⇒ =
2
+ 2 + 2 …(i)
1 1 1 a b c p2
2
+ 2
+
a b c2
a b c
and centroid is G :  , ,  = (x, y, z)
3 3 3
⇒ a = 3x, b = 3y, c = 3z
replace term in (i)
1 1 1 1 1 1 1 9
(i) 2
+ 2
+ 2
= 2
⇒ 2
+ 2
+ 2
=
x y z p x y z p2

(ii) centre of the tetrahedron


a b c
C :  , ,  = (x, y, z)
4 4 4
Vector & Three-Dimensional Geometry
⇒ a = 4x, b = 4y, c = 4z
replace term in (i),
1 1 161
2
+ 2
+ = 2

x y z p2

Two intersecting lines determine a unique plane:


  
r = a + λb 
Let the equations of the two lines are     where a is position
r= a + µc 
vector of common point.

117.
Hence the equation of the plane is,
       
0 or [r − a b c] =
(r − a) ⋅ (b × c) = 0 or
  
[rbc] = [abc]

Condition for coplanarity of two lines :


     
Let lines are r = a + λb and r = c + µd
 
(b and d are non collinear)
   
( )
(r − a) ⋅ b × d =0 is the equation of the plane
 
Condition is [abd] = [cbd] .

Plane containing two parallel lines :


     
Let lines are r = a + λc and r = b + µc
   
∴ (r − a) ⋅ {(a − b) × c} =0
    
or [rac] + [rcb] + [abc] = 0
    
or r − a a − b c  = 0
 

Q.
Find whether the two lines intersect or not. If they do, find the equation of the
plane containing them. If they don’t find the shortest distance between them.
Determine whether the following pair of lines intersect or not.
 ˆ r = 2iˆ − ˆj + µ(iˆ + ˆj − k)
(i) r = ˆi − ˆj + λ(2iˆ + k); ˆ
 
(ii) r = ˆi + ˆj − kˆ + λ(3iˆ − ˆj); r = 4iˆ − kˆ + µ(2iˆ + 3k)
ˆ
 
r =i + 2j + 3k + λ(2j + 3j + 2k)
(iii)  & r= 2i + 4j + 5k + µ(4i + 6j + 4k)

   


Sol. (i) Here a 1 = ˆi − ˆj, a2 = 2iˆ − ˆj,b1 = 2iˆ + kˆ and b2 = ˆi + ˆj − kˆ
Vector & Three-Dimensional Geometry

2 − 1 −1 + 1 0
   
Now a 2 − a 1 b1 b2  = 2 0 1
 
1 1 −1
1 0 0
= 2 0 1
1 1 −1
=–1≠0
Thus, the two given lines do not intersect.

118.
   
(ii) Here a 1 = ˆi + ˆj − k,
ˆ a = 4iˆ − k,b
2
ˆ
1
= 3iˆ − ˆj and b2 = 2iˆ + 3kˆ
4 − 1 0 − 1 −1 + 1 3 −1 0
   
⇒ a 2 − a 1 b1 b=
2
 3 −1 0 = 3 −1 0 =0
2 0 3 2 0 3
Thus, the two given lines intersect. Let us obtain the point of intersection of

the two given lines. For some values of λ and µ, the two values of r -must
coincide.
Thus, ˆi + ˆj − kˆ + λ(3iˆ − ˆj) = 4iˆ − kˆ + µ(2iˆ + 3k)
ˆ

⇒ (3 + 2µ − 3λ )iˆ + (λ − 1)ˆj + 3µkˆ =


0

⇒ 3 + 2µ − 3λ = 0, λ − 1 = 0, 3µ = 0

Therefore, the point of intersection is = r 4iˆ − kˆ (by putting µ = 0 in the second
equation
 
(iii) r =ˆi + 2ˆj + 3kˆ + λ(2iˆ + 3ˆj + 2k)
ˆ & r= 2iˆ + 4ˆj + 5kˆ + µ(4ˆi + 6ˆj + 4kˆ )

These two lines are parallel and parallel to the vector c = 2iˆ + 3ˆj + 2kˆ
 
also a =ˆi + 2ˆj + 3kˆ & b = 2iˆ + 4ˆj + 5kˆ
Distance between these two lines is
î ˆj kˆ
2 3 2
   1 2 2 | 2iˆ − 2ˆj + kˆ |
| c × (b − a) | 3
=  = =
| c| 17 17 17
Equation of plane containing these two lines.
    
[ r − a b − a c] =0
x−1 y−2 z−3
1 2 2 = 0

Vector & Three-Dimensional Geometry


2 3 2
⇒ (–2) (x – 1) – (y – 2) (–2) + (z – 3) (–1) = 0
⇒ –2x + 2y – z + 1 = 0
⇒ 2x – 2y + z – 1 = 0

Line of intersection / Common Line between two intersecting planes :


P 1 ≡ a 1x + b1 y + c1z + d1 =0
P 2 ≡ a2 x + b2 y + c2z + d2 =0
then line is
a1x + b1y + c1z + d1 = a2x + b2y + c2z + d2 = 0

119.
Q.
Reduce the equation of line x – y + 2z = 5 and 3x + y + z = 6 in symmetrical
form.
Or
Find the line of intersection of plane x – y + 2z = 5 and 3x + y + z = 6

Sol. Given x – y + 2z = 5, 3x + y + z = 6
Let z = λ
Then, x – y = 5 – 2λ
and 3x + y = 6 – λ.
Solving these two equations, 4x = 11 – 3λ
and 4y = 4x – 20 + 8λ = – 9 + 5λ
4x − 11 4y + 9 z − 0
The equation of the line is = =
−3 5 1

Previous Year’s Question

If the lines x = ay + b, z = cy + d and x = a’z + b’, y = c’z + d’ are perpendicular, then


[JEE Main 2019]
(A) cc’ + a + a’ = 0 (B) aa’ + c + c’ = 0 (C) ab’ + bc’ + 1 = 0 (D) bb’ + cc’ + 1 = 0

Previous Year’s Question

x−1 y z−1 x −1 y z−1


Let L1 and L2 be the following straight lines. L 1 :
= = and L2 : = = .
1 −1 3 −3 −1 1
x−α y−1 z−γ
Suppose the straight line L : = = lies in the plane containing L1 and L2,
l m −2
and passes through the point of intersection of L1 and L2. If the line L bisects the acute
angle between the lines L1 and L2, then which of the following statements is/are TRUE?
Vector & Three-Dimensional Geometry

[JEE Advanced 2020]


(A) a - g = 3 (B) l + m = 2 (C) a - g = 1 (D) l + m = 0

120.
EQUATION OF PLANES BISECTING THE ANGLES
BETWEEN TWO GIVEN PLANES :
Cartesian Form :
The equation of the planes bisecting the angles Know the facts
between the planes a 1x + b1 y + c1z + d1 =
0 and
If angle between bisector plane
a2 x + b2 y + c2z + d2 =
0 are
and one of the plane is less
(a1x + b1y + c1z + d1 ) = ± (a2x + b2 y + c2z + d2 ) than 45° then it is acute angle
a21 + b21 + c21 a22 + b22 + c22 bisector otherwise it is obtuse
angle bisector.

Vector Form :
The equation of the planes bisecting the angles
   
between the planes r ⋅ n1 = d1 and r ⋅ n2 = d2 are
       
r ⋅ n1 − d1
=
r ⋅ n2 − d2
or
r ⋅ n1 − d1
= ±
(
r ⋅ n2 − d2 )
   
n1 n2 n1 n2

( ) d
n1
d
or r × n 1 ± n 2 =  1 ±  2
n2

FAMILY OF PLANES :
The equation of a plane passing through the lines
of intersection of a 1x + b1 y + c1z + d1 =
0 and
a2 x + b2 y + c2z + d2 =
0 is

(a1x + b1y + c1z + d1 ) + λ (a2x + b2 y + c2z + d2 ) =0 ,


where λ is a constant.

Vectorially :
Vector & Three-Dimensional Geometry
Equation of a plane passing through the line of
   
intersection of planes r ⋅ n1 = d1 and r ⋅ n2 =
d2 is
   
( ) (
given by r ⋅ n1 − d1 + λ r ⋅ n2 − d2 =0 )

121.
Q. Find the equation of plane containing the line of intersection of the plane
x+ y+z−6 = 0 and 2x + 3y + 4z + 5 =0 and passing through (1, 1, 1)

Sol. The equation of the plane through the line of intersection of the given planes is,
( x + y + z − 6) + λ ( 2x + 3y + 4z + 5) =0 … (1)
If it is passes through ( 1, 1, 1)

3
( ) ( )
⇒   1+ 1+ 1−6 + λ 2+ 3+ 4 +5 = 0 ⇒ λ =
14
Putting λ =3 / 14 in (i) we get

( x + y + z − 6) + 143 ( 2x + 3y + 4z + 5) =0
⇒  20x + 23y + 26z − 69 =
0

Q. Find the equation of the plane passing through (3, 4, –1), which is parallel to
the plane r·(2i − 3j + 5k)
 +7 =
0

Sol. The equation of plane which is parallel to


r·(2i − 3j + 5k)
 +7 =
0 is

r·(2i − 3j + 5k)


 + λ =0 ...(i)

or 2x – 3y + 5z + λ = 0
Further (i) will pass through (3, 4, – 1)
If (2) (3) + (–3) (4) + 5(–1) + λ = 0
or –11 + λ = 0 ⇒ λ = 11
Thus, equation of the required plane is r · (2i − 3 j + 5k ) + 11 =
0

Q. The plane x – y – z = 4 is rotated through 90° about its line of intersection


Vector & Three-Dimensional Geometry

with the plane


x + y + 2z = 4. Find its equation in the new position.

Sol. Given planes are x – y – z = 4 …(i)


& x + y + 2z = 4 …(ii)
Equation of any plane passing through the line of Intersection of these planes,
(x – y – z − 4)+ λ(x + y + 2z – 4) = 0
x(1 + λ) + y(–1 + λ) + z(–1 + 2λ) + (–4 – 4λ) = 0 …(A)
The direction cosine of a normal to plane (A) are
1+λ ( λ − 1) −1 + 2λ
, ,
6λ − 4 λ + 3 6λ − 4 λ + 3 6λ 2 − 4 λ + 3
2 2

122.
Since the angle between plane (A) and (i) is 90°, so we have
(1 + λ ) ( −1 ) ( λ − 1) ( −1 ) ( −1 + 2λ )
+ − 0
=
2 2
3 6λ − 4 λ + 3 3 6λ − 4 λ + 3 3 6λ 2 − 4 λ + 3
0 = 1 + λ – λ + 1 + 1 – 2λ ⇒ λ = + 3/2
So equation of plane is 5x + y + 4z = 20

Q. Find the condition of lines x = ay + b, z = cy + d, x = a’y + b’, z = c’y + d’ are per-


pendicular.

Sol. The equations of straight lines can be rewritten as


x −b y −0 z −d
x = ay + b, z = cy + d ⇒ = =
a 1 c
x − b ' y − 0 z − d'
and x = a ' y + b ', z = c ' y + d' ⇒ = =
a' 1 c'
The above lines are perpendicular if aa' + 1.1 + c.c' = 0.

Q. Find the equation of the plane containing the line of intersection of the plane
x + y + z – 6 = 0 and 2x + 3y + 4z + 5 = 0 and passing through the points (1, 1, 1)


Sol. The equation of a plane through the line of intersection of the given plane is
(x + y + z – 6) + λ(2x + 3y + 4z + 5) = 0 …(i)
If line (1) passes through (1, 1, 1), we have – 3 + 14λ = 0
3
⇒ λ = .
14
3
Putting λ = in line (i), we obtain the equation of the required plane as
14
3
(x + y + z − 6) + (2x + 3y + 4z + 5) =0
14
⇒ 20x + 23y + 26z – 69 = 0 Vector & Three-Dimensional Geometry

Q. The plane ax + by = 0 is rotated through an angle α about its line of


intersection with the plane z = 0. Show that the equation to the plane is the
new position is ax + by ± z a2 + b2 tan α =0

Sol: Given planes are ax + by = 0 …(i)


and z = 0 …(ii)
Therefore, the equation of any plane passing through the line of intersection of
planes
(i) and (ii) may be taken as

123.
ax + by + kz = 0 ...(iii)
The direction cosines of a normal to the plane (iii) are
a b k
, and
2 2 2 2 2 2
a +b +k a +b +k a + b2 + k2
2

The direction cosines of a normal to the plane (i) are


a b
and 0
a2 + b2 a2 + b2
Since the angle between the planes (i) and (iii) is α,
a·a + b·b + k·0 a2 + b2

= cos α =
a2 + b2 + k2 a2 + b2 a2 + b2 + k2

(
⇒ k2 cos2 α = a2 1 − cos2 α + b2 1 − cos2 α ) ( )
2
⇒k =
(
a2 + b2 sin2 α
⇒k =
)
± a2 + b2 tan α
cos2 α
Putting this in (iii), we get the equation of the plane as ax + by ± z a 2 + b2 tan α =0

STRAIGHT LINES
Symmetric Form :
1. Equation of a straight line passing through
(x1, y1, z1) and having drs as a, b, c is
x − x1 y − y1 z − z1
= = = λ
a  b c
Proof:
A vector parallel to line will be aiˆ + bjˆ + ck
A vector along the line can be written as
 Point to Remember!!!
( ) (
AP = x − x 1 i + y − y 1 j + z − z1 k ) ( )
 y Any point on this line can be
Vector & Three-Dimensional Geometry

   vector AP is parallel to aiˆ + bjˆ + ck taken as


x − x1
=
y − y1
=
z − z1
= λ
(x 1 + λl, y 1 + λm, z1 + λn .)
a  b c y If dcs of line be l,  m,n then
2. Equation of straight line passing through two
its equation will
( )
points x 1 , y 1 , z1 and x2 , y 2 , z2 will be ( ) x − x1 y − y 1 z − z1
= = = λ and
x − x1 y − y1 z − z1 l  m n
= =
x2 − x 1 y 2 − y 1 z2 − z1 any point on this line can be
taken as
( )
x 1 + λa, y 1 + λb, z1 + λc

124.
Note:
x y z
(a)  Equation of x-axis is = = (or) y = z = 0
1 0 0
x y z
(b) Equation of y-axis is = = (or) x = z= 0
0 1 0
x y z
(c) Equation of z-axis = = (or) x = y = 0
0 0 1
Here zero in denominator represents that line is perpendicular to
that axis.
x−2 y+1 x−2 y+1 z−2
Ex. Line = and z = 2 is written as = =
3 −2 3 −2 0
 This line is perpendicular to z-axis or parallel to xy plane at a
distance of 2 units.

Unsymmetrical form of straight line :


The equations a 1x + b1 y + c1z + d1 =
0 and a2 x + b2 y + c2z + d2 =
0
together represents a line in unsymmetrical form. This represent
equation of line of intersection of planes a 1x + b1 y + c1z + d1 =
0 and
a2 x + b2 y + c2z + d2 =
0

Procedure to convert Unsymmetrical Form of straight line to Symmetrical


Form :
Let the direction ratios of the line of intersection (AB) of two planes
a 1x + b1 y + c1z + d1 =
0  …(1)
and a2 x + b2 y + c2z + d2 =
0 …(2)
are a, b, c
Direction ratios of normal to plane (1) are a 1 ,  b1 , c1 and
Direction ratios of normal to plane (2) are a2 ,  b2 , c2
Line AB lies in both the planes (1) and (2) Vector & Three-Dimensional Geometry
hence normals to (1) and (2) are perpendicular to AB.
Hence  aa 1 + bb1 + cc1 =
0  and  aa2 + bb2 + cc2 =0
these two will give the proportional values of a, b, c.
(
Let the line AB cuts the xy plane at x 1 , y 1 , 0)
Hence a 1x 1 + b1 y 1 =
−d1 and a2 x 1 + b2 y 1 =
−d2   This will give a point on
the line AB
x − x1 y − y1 z − 0
∴ equation of AB is = = .
a  b c

125.
Q. Find the angle between the line x − 2y + z = 0 = x + 2y − 2z and
x + 2y + z = 0 = 3x + 9y + 5z

Sol. Let a1 ,b1 , c1 be the direction ratios of the line x − 2y + z =0 and x + 2y − 2z =


0.
Since it lies in both the planes, therefore, it is ^ to the normals to the two
planes.
∴ a1 − 2b1 + c1 = 0 ⇒ a1 + 2b1 − 2c1 = 0
Solving these two equations by cross-multiplication, we have
a1 b1 c1 a 1 b1 c 1
= =   or   = =
4 −2 I+ 2 2+ 2 2 3 4
Let a2 ,b2 , c2 be the direction ratios of the line x + 2y + z = 0 = 3x + 9y + 5z.
Then the discussed above
a2 + 2b2 + c2 = 0 3a2 + 9b2 + 5c2 = 0
a2 b2 c2 a2 b2 c2
⇒ = =   or   = =
10 − 9 3 − 5 9 − 6 1 −2 3
Let θ be the angle between the given lines. Then
al a 2 + bl b2 + cl c2
cosq =
al2 + bl2 + c21 a 22 + b22 + c22

=
( 1)( 2) + ( −2)( 3) + ( 3)( 4 )
22 + 32 + 42 12 + ( −2)2 + (3)2
2 − 6 + 12 8  8 
= = ⇒ θ cos−1 
= 
29 14 406  406 

Q. Find the coordinates of the point where the line joining the points 2, −3, 1 and ( )
( 3, −4, −5) cuts the plane 2x + y + z =7
Vector & Three-Dimensional Geometry

Sol. The direction ratios of the line are ( )


3 − 2, −4 − −3 , −5 − 1 i.e. 1, −1, −6
x −2 y +3 z−1
Hence equation of the line joining the given points is = = = r (say)
1 −1 −6
(
Coordinates of any point on this line are r + 2, −r − 3, −6r + 1 )
If this point lies on the given plane 2x + y + z =7 , then
( ) ( ) ( )
2 r + 2 + −r − 3 + −6r + 1 =7 ⇒ r =−1

Coordinates of the point are (−1 + 2, −(−1) − 3, −6(−1) + 1) i.e. (1, − 2, 7).

126.
Q. Find unsymmetrical form the equations of the line
3x + 2y − z − 4 = 0 = 4x + y − 2z + 3.

Sol. The equation of the line in unsymmetrical form are


3x + 2y − z − 4 = 0 , 4x + y − 2z + 3 = 0 …(1)
Let l,m,n be the direction cosines of the line. Since the line is common to both the
planes, it is perpendicular to the normals to both the planes. Hence,
3l + 2m − n= 0, 4l + m − 2n= 0
l m n
Solving these we get, = =
−4 + 1 −4 + 6 3 − 8
l m n
i.e. = = and ( −3)2 + 22 + ( −5)2= 38
−3 2 −5
So, direction cosines of the lines are
−3 2 −5
, ,
38 38 38
Now to find the coordinates of a point on a line. Let
us find out the point where it meets the plane
()
z = 0. Putting z = 0 in the equation given by 1 ,
we have 3x + 2y − =
4 0, 4x + y + =
3 0 . Solving
these, we get x =
−2, y =
5
(
So, one point of the line is −2, 5, 0 )
x+2 y−5 z−0
∴ Equation of the line in symmetrical form is = = i.e.
−3 2 −5
x+2 y−5 z
= = .
−3 2 −5

Q. Find the equation of the plane which contains the two parallel lines
x+1 y−2 z
= =
x−3 y+4 z−1
and = = . Vector & Three-Dimensional Geometry
3 2 1 3 2 1

Sol. The equations of the two parallel lines are


x+1 y−2 z−0
= = …(1)
3 2 1
x−3 y+4 z−1
and = = …(2)
3 2 1
the equation of any plane through the line (1) is
a(x + 1) + b(y - 2) + cz = 0 …(3)

127.
where 3a + 2b + c = 0 …(4)
the line (2) will also lie on the plane (3) if the point (3, -4, 1) lies on the plane (3),
and for this we have a(3 + 1) + b(–4 –2) + c = 0 or 4a – 6b + c = 0 …(5)
a b c
Solving (4) and (5), we get = =
8 1 −26
Putting the values of a, b, c in (3), the required equation of the plane is
8x + y – 26z + 6 = 0.

Q. Find the shortest distance between the lines


x−3 y−8 z−3 x+3 y +7 z−6
= = ,= = . Also find the equation of line of
3 −1 1 −3 2 4
shortest distance.

x−3 y−8 z−3


Sol. Given lines are =
3
= = rI
−1 1
(say) …(1)

x +3 y +7 z–6
= = = r2 (say) …(2)
–3 2 4
any point on line (1) is P(3r1 + 3, 8 – r1, r1 + 3) and on line (2) is
Q (–3 – 3r2, 2r2 – 7, 4r2 + 6).
If PQ is line of shortest distance, then direction ratios of
PQ = (3r1 + 3) – (–3 – 3r2), (8 – r1) – (2r2 – 7), (r1 + 3) – (4r2 + 6)
i.e. 3r1 + 3r2 + 6, – r1 – 2r2 + 15, r1 – 4r2 – 3
As PQ is perpendicular to lines (1) and (2)
∴ 3(3r1 + 3r2 + 6) – 1 (–r1 – 2r2 + 15) + 1(r1 – 4r2 − 3) = 0
⇒ 11r1 + 7r2 = 0 …(3)
and –3(3r1 + 3r2 + 6) + 2(–r1 – 2r2 + 15) + 4(r1 – 4r2 − 3) = 0
i.e. 7r1 + 29r2 = 0 …(4)
On solving equations (3) and (4), we get r1 = r2 = 0.
So, point P(3, 8, 3) and Q(–3, –7, 6)
∴ Length of shortest distance PQ= {(−3 − 3)2
}
+ ( −7 − 8)2 + (6 − 3)2 = 3 30 .
Vector & Three-Dimensional Geometry

Direction ratios of shortest distance line is 2, 5, –1.


x−3 y−8 z−3
∴ Equation of shortest distance line = = .
2 5 −1

Q. Find the distance of the point (1, –2, 3) from the plane x – y + z = 5 measured
x y z
parallel to the line = = .
2 3 −6

Sol. Here we are not to find perpendicular distance of the point from the plane but
distance measured along with the given line. The method is as follow:
The equation of the line through the point (1, -2, 3) and parallel to given line is

128.
x−1 y+2 z−3
= = = r (say)
2 3 −6
The coordinate of any point on it is (2r + 1, 3r – 2, –6r + 3).
If this point lies in the given plane then
1
2r + 1 − (3r − 2) + ( −6r + 3) = 5 ⇒ −7r = −1 or r =
7
 9 −11 15 
∴ Point of intersection is  , , 
7 7 7 
∴ The required distance = the distance between the points (1, −2, 3) and
 9 −11 15 
 , , 
7 7 7 
2 2 2
 9  11   15  49
=  1 −  +  −2 +  +  3 − = = 1 Unit.
 7  7  7  7

Q. Find the image of the point (1, 3, 4) in the plane 2x – y + z + 3 = 0.

Sol. As it is clear from the figure that PQ will be perpendicular to the plane and foot
of this perpendicular is mid point of PQ i.e. N.
So, direction ratios of line PQ is 2, –1, 1
x−1 y−3 z−4
⇒ Equation of line= PQ = = = r (say)
2 −1 1
⇒ Any point on line PQ is (2r + 1, – r + 3, r + 4)
If this point lies on the plane, then
2(2r + 1) – (– r + 3) + (r + 4) + 3 =0
⇒r=–1
∴ Coordinate of foot of perpendicular N = (–1, 4, 3).
As N is middle point of PQ.
1 + x1 3 + y1 4 + z1
∴ –1 = ,4 = ,3 = ⇒ x 1 = –3, y 1 = 5, z1 = 2
Vector & Three-Dimensional Geometry
2 2 2
∴ Image of point P(1, 3, 4) is the point Q(–3, 5, 2).

Q. Determine whether each statement is true or false


(a) Two lines parallel to a third line are parallel.
(b) Two lines perpendicular to a third line are parallel.
(c) Two planes parallel to a third plane are parallel.
(d) Two planes perpendicular to a third plane are parallel.
(e) Two lines parallel to a plane are parallel.
(f) Two lines perpendicular to a plane are parallel.
(g) Two planes parallel to a line are parallel.

129.
(h) Two planes perpendicular to a line are parallel.
(i) Two planes either intersect or are parallel.
(j) Two lines either intersect or are parallel.
(k) A plane and a line either intersect or are parallel.

Ans. (a) True, (b) False, (c) True, (d) False, (e) False, (f) True,
(g) False, (h) True, (i) True, (j) False, (k) True

Q. Find the equation of a plane passing through the point A(3,–2, 1) and
perpendicular to the vector 4ˆi + 7 ˆj − 4kˆ . If PM be perpendicular from the point
P(1, 2, –1) to this plane, find its length.
 
Sol. Let O be the origin, then a = OA = 3iˆ − 2ˆj + kˆ

Let n = 4ˆi + 7 ˆj − 4k
ˆ
Let Q(x, y, z) be any point on the plane
 
Then, r = OQ = x ˆi + yjˆ + zkˆ
 
Now r − a =( x − 3)iˆ + ( y + 2)ˆj + (z − 1)kˆ
Now, equation of the required plane
 
passing point A(a) and perpendicular to n is
  
(r − a)·n =
0
∴ 4(x – 3) + 7(y + 2) – 4(z – 1) = 0
or 4x +7y – 4z + 6 = 0. This is the required equation of the plane.
Now PM ^ AM

∴ Unit vector parallel to PM

 n 4ˆi + 7 ˆj − 4kˆ
n =
= 
| n| 9
 
Vector & Three-Dimensional Geometry

Now PM = length of projection of PA on PM


  ˆ ˆ ˆ
ˆ  4 i + 7j – 4k  = 28
ˆ |= (2iˆ – 4jˆ + 2k)·
= | PA·n

 9 
 9

Second method:
 
Let α = OP = ˆi + 2ˆj − kˆ
  
Alternatively, PM = ( α − a)·n
n
   
Here, α − a = AP = −2iˆ + 4ˆj − 2k
ˆ and n = 4ˆi + 7 ˆj − 4kˆ
−8 + 28 + 8 28

= PM =
9 9
130.
Q. Find the shortest distance and the vector equation of the line of shortest

distance between the lines given by r= 3iˆ + 8ˆj + 3kˆ + λ(3iˆ − ˆj + k)
ˆ and

r = −3iˆ − 7ˆj + 6kˆ + µ( −3iˆ + 2ˆj + 4k)
ˆ


Sol. Given lines are r= 3iˆ + 8ˆj + 3kˆ + λ(3iˆ − ˆj + k)
ˆ …(i)
and r =−3i − 7 j + 6k + µ( −3i + 2 j + 4k ) …(ii)
Equation of lines (i) and (ii) in cartesian from are,
x−3 y−8 z−3
AB : = = = λ …(iii)
3 −1 1 L
x+3 y+7 z−6
and CD : = = = µ …(iv)
−3 2 4
Let L ≡ (3λ + 3, –λ + 8, λ + 3)
and M ≡ (–3m – 3, 2m – 7, 4m + 6)
Direction ratios of LM are
3l + 3m + 6, –l – 2m + 15, l – 4µ – 3
Since LM ^ AB
∴ 3(3λ + 3µ + 6) – 1(– λ – 2µ + 15) + 1 (λ – 4µ – 3) = 0
or 11λ + 7µ = 0 …(v)
Again LM ^ CD
∴ −3(3λ + 3µ + 6) + 2( −λ − 2µ + 15) + 4(λ − 4µ − 3) =0
or –7λ – 29µ = 0 …(vi)
Solving (v) and (vi), we get λ = 0, µ = 0
∴ L ≡ (3, 8, 3), M ≡ (–3, –7, 6)
Hence shortest distance LM = (3 + 3)2 + (8 + 7)2 + (3 − 6)2 = 270 = 3 30 units
Vector equation of LM is

r = 3i + 8j + 3k + t(6i + 15j − 3k)

x−3 y−8 z−3


Note: Cartesian equation of LM is = =

Vector & Three-Dimensional Geometry


6 15 −3
Miscellaneous Examples on Line and Plane and Shortest Distance between Skew Lines

Q. Find the vector equation of the following planes in cartesian form:



r = ˆi − ˆj + λ(iˆ + ˆj + k)
ˆ + µ(iˆ − 2ˆj + 3k)
ˆ


Sol. The equation of the plane r = ˆi − ˆj + λ(iˆ + ˆj + k)
ˆ + µ(iˆ − 2ˆj + 3k)
ˆ

Let r = xiˆ + yjˆ + zkˆ
Hence, the equation is (xiˆ + yjˆ + zk)
ˆ − (iˆ − ˆj) = λ(iˆ + ˆj + k)
ˆ + µ(iˆ − 2ˆj + 3k)
ˆ

131.
Thus vectors (xiˆ + yjˆ + zk)
ˆ − (iˆ − ˆj), ˆi + ˆj + k,
ˆ ˆi − 2ˆj + 3kˆ are coplanar.
Therefore, the equation of the plane is
x − 1 y − ( −1) z − 0
1 1 =1 0 or 5x − 2y − 3z=
−7 0
1 −2 3

x−1 y+2 z
Q. Find the points where line =
2
=
−1 1
intersects xy, yz and zx planes.

Sol. line meets xy-plane where z = 0


x−1 y+2 0
Hence from the given equation of line, = =
2 −1 1
⇒ x = 1 and y = – 2.
⇒ Line meets xy-plane at (1, –2, 0).
Line meets yz-plane where x = 0
0−1 y+2 z
Hence from the given equation of line, = =
2 −1 1
−1 3
⇒ z = and y =

2 2
 3 −1 
⇒ Line meets yz -plane at  0, − , 
 2 2
Line meets zx-plane where y = 0
x−1 0+2 z
Hence from the given equation of line = =
2 −1 1
⇒ z = –2, x = – 3
⇒ Line meets zx-plane at (–3, 0, –2)

x −2 y −2 z−1
Q. Find the point where the line = = cuts the plane x + y + z = 0.
Vector & Three-Dimensional Geometry

3 0 2

x−2 y−2 z−1


Sol. Given line,
3
=
0
=
2
= (λ )

Any point on this line (3λ + 2, 2, 2λ + 1)


Given plane, x + y + z = 0, point lies on this plane,
3λ + 2 + 2 + 2λ + 1 = 0
λ = −1
So point is (–1, 2, –1).

132.
Q. Find equation of line passing through (0, 0, 0) and ^ to fixed line
x−1 y+2 z
= = and || to fixed plane x – y + z + 2 = 0.
2 −3 5

Sol. Equation of line passing through (0, 0, 0) and having direction rations
< a, b, c > is
x y z
= = …(i)
a b c
x−1 y−2 z
Line (i) is perpendicular to = =
2 −3 5
So we get, 2a – 3b + 5c = 0 …(A)
and parallel to plane x – y + z + 2 = 0
So we get a – b + c = 0 …(B)
a −b c
by (A) & (B) = =
−3 5 2 5 2 −3
−1 1 1 1 1 −1
a −b c a b c
⇒ = = ⇒ = =
2 −3 1 2 3 1
x y z
Equation of line (i), = =
2 3 1

Q. Find the length of the perpendicular drawn from point (2, 3, 4) to line
4 − x y 1−z
= =
2 6 3

Sol. Let P be the foot of the perpendicular from A(2, 3, 4) to the given line l whose
4 − x y 1−z x−4 y z−1
equation is = = or = = = k (say). Therefore, …(i)
2 6 3 −2 6 −3
x = 4 – 2k, y = 6k, z = 1 – 3k
Vector & Three-Dimensional Geometry
As P lies on (i), coordinates of P are
(4 – 2k, 6k, 1 – 3k) from some value of k.
The direction ratios of AP are
(4 – 2k – 2, 6k –3, 1 – 3k – 4)
or (2 – 2k, 6k – 3, –3 – 3k).
Also, the direction ratios of l are –2, 6 and –3.
Since AP ^ l,
⇒ –2(2 – 2k) + 6(6k – 3) – 3(–3 – 3k) = 0
⇒ −4 + 4k + 36k − 18 + 9 +=9k 0 or 49k = − 13 0 or
= k 13 / 49
We have AP2 = (4 – 2k – 2)2 + (6k – 3)2 + (1 – 3k – 4)2
= (2 – 2k)2 + (6k – 3)2 + (–3 – 3k)2

133.
= 4 − 8k + 4k2 + 36k2 − 36k + 9 + 9 + 18k + 9k2
= 22 – 26k + 49k2
2
 13   13 
22 − 26 
=  + 49  
 49   49 
22 × 49 − 26 × 13 + 132 909

= =
49 49
3
AP = 101
7

Previous Year’s Question

A line l passing through the origin is perpendicular to the lines  [IIT-Advance 2013]
l : (3 + t)i + ( −1 + 2t)j + (4 + 2t)k,
1
 −∞ < t < ∞

 −∞ < s < ∞
l 2 : (3 + 2 s)i + (3 + 2 s)j + (2 + s)k,

Then, the coordinate(s) of the point(s) on l2 at a distance of 17 from the point of


intersection of l and l1 is(are) :
7 7 5 7 7 8
(A)  , ,  (B) (–1, –1, 0) (C) (1, 1, 1) (D)  , , 
3 3 3 9 9 9

Q. Find equation of line passing through (1, 1, 2) and | | to planes


x + 2y + 2z + 3 = 0 and x + 2y – z + 4 = 0.

Sol.
Equation of line passing through (1, 1, 2) and having direction ratios < a, b, c > is
x−1 y−1 z−2
= = − (i)
Vector & Three-Dimensional Geometry

a b c
This line is parallel to the planes x + 2y + 2z + 3 = 0 & x + 2y – z + 4 = 0
So we get, a + 2b + 2c = 0
& a + 2b – c = 0
a −b c
We get, = =
2 2 1 2 1 2
2 −1 1 −1 1 2
a b c
⇒ = =
−6 3 0
x−1 y−1 z−2
Hence equation of line, = =
−6 3 0

134.
Q. Reduce the equation of line x – y + 2z = 5 and 3x + y + z = 6 in symmetrical
form. or
Find the line of intersection of planes x – y + 2z = 5 and 3x + y + z = 6.

Sol. Given x – y + 2z = 5, 3x + y + z = 6.
Let z = λ.
Then x – y = 5 – 2λ and 3x + y = 6 – λ.
Solving these two equations, 4x = 11 – 3λ and 4y = 4x – 20 + 8λ = – 9 + 5λ.
4x − 11 4y + 9 z − 0
The equation of the line is = =
−3 5 1

Q. Prove that the lines 3x + 2y + z – 5 = 0 = x + y – 2z – 3 and 2x – y – z = 0 = 7x +


10y – 8z are perpendicular to each other.

Sol. 3x + 2y + z – 5 = 0 = x + y – 2z – 3
Direction ratio of line passing through the intersection of these two planes,
ˆi ˆj k̂
n1 × n2 =3 2 1 = ˆi( −5) − ˆj( −7) + k(1)
ˆ =−5î + 7 ˆj + kˆ
1 1 −2

2x – y – z = 0 = 7x + 10y – 8z
ˆi ˆj k̂
n3 × n4= 2 −1 −1 = ˆi(18) − ˆj( −16 + 7) + k(27)
ˆ = 181ˆ + 9ˆj + 27kˆ
7 10 −8

Product of these: – 5 × 18 + 7 × 9 + 1 × 27 = 0
hence perpendicular to each other

Q. The distance of the point (1, –2, 3) from the plane x – y + z = 5 measured
x y z−1
parallel to the line = = is Vector & Three-Dimensional Geometry
2 3 −6
(A) 1 (B) 2 (C) 4 (D) None of these

Sol. (A)

The equation of the line passing through P(1, – 2, 3) and parallel to the given
line is
x−1 y+2 z−3
= =
2 3 −6
Suppose it meets the plane x – y + z = 5 at the point Q given by

135.
x−1 y+2 z−3
= =
2 3 −6
= λ i.e. (2λ + 1, 3λ – 2, – 6λ +3)
This lies on x – y + z = 5. Therefore,
2λ + 1 – 3λ + 2 – 6λ + 3 = 5
1
⇒ – 7λ = – 1 ⇒ λ =
7
 9 11 15 
So, the coordinate of Q are  , − , 
7 7 7 

4 9 36
Hence, required distance = PQ = + + = 1.
49 49 49

x−1 y−3 z−2


Q. Find distance of the point A(3, 8, 2) from the line =
2
=
4 3
measured parallel to the P : 3x + 2y –2z +15 = 0.

Sol. Any point ‘Q’ on the given line: (2λ + 1, 4λ + 3, 3λ + 2)


Direction rations of ‘PQ’

(2λ – 2, 4λ – 5, 3λ)
PQ is parallel to the given plane, hence
Vector & Three-Dimensional Geometry

3(2λ – 2) + 2(4λ – 5) – 2(3λ) = 0


⇒ 6λ – 6 + 8λ – 10 – 6λ = 0
⇒λ=2
Hence PQ 4 + 9 + 36 = 7

Q. Find the vector equation of a line passing through (2, –1,1 ) and parallel to the
x−3 y+1 z−2
line whose equation is = = .
2 7 −3

136.
x−3 y+1 z−2
Sol. Since the required line is parallel to = =
2 7 −3
, it follows that the

required line passing through A(2iˆ − ˆj + k)


ˆ has the direction of 2iˆ + 7ˆj − 3kˆ .

Hence, the vector equation of the required line is r= 2iˆ − ˆj + kˆ + λ(2iˆ + 7ˆj − 3k)
ˆ

where λ is a parameter.

Q. Prove that the three lines from O with direction cosines l1, m1, n1; l2, m2, n2 and
l3, m3, n3 are coplanar, if l1(m2n3 – n2m3) + m1(n2l3 – l2n3) + n1(l2m3 – l3m2) = 0.

Sol.
Here, three given lines are coplanar, if they have common perpendicular.
Let DC’s of common perpendicular be l, m and n.
ll1 + mm1 + nn1 = 0 …(i)
ll2 + mm2 + nn2 = 0 …(ii)
and ll3 + mm3 + nn3 = 0 …(iii)
Solving Eqs. (ii) and (iii) by cross multiplica-
tion method, we get
l m n
⇒ = = = k
m2n3 − n2m3 n2l 3 − n3l 2 l 2m3 − l 3m2

= (
l k m2n3 − n2m3 = ) (
,m k n2l 3 − n3l 2= ) (
,n k l 2m3 − l 3m2 )
Substituting in Eq. (i), we get

= (
l k m2n3 − n2m3 = ) (
,m k n2l 3 − n3l 2=) (
,n k l 2m3 − l 3m2 )
( ) ( ) (
⇒ l 1 m2m3 − n2m2 + m1 n2l 3 − n3l 2 + n1 l 2m3 − l 3m2 )

Q. Find the shortest distance between the lines


x−1 y−2 z−3 x−2 y−4 z−5
= = and = = . Also obtain the equation of the
2 3 4 3 4 5
lines of the shortest distance. Vector & Three-Dimensional Geometry

x−1 y−2 z−3


Sol.
(i) The two given lines are = = = r1 (say)
2 3 4
…(i)

x−2 y−4 z−5


and = = = r2 (say) …(ii)
3 4 5
Any point on (i) is given by P(2r1 + 1, 3r1 + 2, 4r1 + 3 )
And any point on (ii) is given by Q(3r2 + 2, 4r2 + 4, 5r2 + 5)
Direction ratios of PQ are given by 3r2 – 2r1 + 1, 4r2 – 3r1 + 2 and 5r2 – 4r1 + 2
Since PQ is perpendicular to (i), we get

137.
2(3r2 – 2r1 + 1) + 3(4r2 – 3r1 + 2) + 4(5r2 – 4r1 + 2) = 0
Or 38r2 – 29r1 + 16 = 0
Also PQ is perpendicular to (ii), we get
( ) ( ) (
3 3r2 − 2r1 + 1 + 4 4r2 − 3r1 + 2 + 5 5r2 − 4r1 + 2 =0 )
or 50r2 – 38r1 + 21 = 0
Solving (iii) and (iv), we obtain r2 = – (1/6), r1 = (1/3).
5 13   3 10 25 
Therefore, coordinates of P and Q are  , 3,  and  , , ,
3 3 2 3 6 
respectively.
2 2 2 2 2 2
3 5  10   25 13   1  1  1 1
Thus, PQ2 =  −  +  − 3 +  −  = −  +   + −  =
 2 3   3   6 3   6  3
   6  6
1
⇒ PQ =
6

The equation of the line of the shortest distance is given by
x − (5 / 3) y−3 z − (13 / 3)
= =
(3 / 2) − (5 / 3) (10 / 3) − 3 (25 / 6) − (13 / 3)
x − (5 / 3) y−3 z − (13 / 3)
= =
−(1 / 6) (1 / 3) −(1 / 6)
x − (5 / 3) y − 3 z − (13 / 3)
= =
1 −2 1
Alternative method for finding the shortest distance:
Line (i) is passing through the point (x1, y1, z1) ≡ (1, 2, 3) and is parallel
to vector
a 1ˆi + b1ˆj + c1kˆ ≡ 2iˆ + 3ˆj + 4kˆ
Line (ii) is passing through the point (x2, y2, z2) ≡ (2, 4, 5) and is parallel to
the vector
a2ˆi + b2ˆj + c2kˆ ≡ 3iˆ + 4ˆj + 5kˆ
Vector & Three-Dimensional Geometry


Hence the shortest distance between the lines using the formula
x 2 − x 1 y 2 − y 1 z2 − z1 2−1 4−2 5−3
a1 b1 c1 2 3 4
a2 b2 c2 3 4 5 1
ˆ is =
i ˆj kˆ ˆi ˆj kˆ 6
a 1 b1 c1 2 3 4
a 2 b2 c 2 3 4 5

138.
Q. Find equation of line through (1, 1, 1) and intersect the lines
x−1 y−2 z−3 x+2 y−3 z+1
= = , = = .
2 3 4 1 2 4

x−1 y−1 z−1


Sol. Any line passing through the point (1, 1, 1) is =
a
=
b c
…(i)

x−1 y−2 z−3


This line intersects the line = = .
2 3 4
1− 1 2− 1 3− 1
If a : b : c ≠ 2 : 3 : 4 and a b c = 0
2 3 4
⇒ a – 2b + c = 0
x − ( −2) y − 3 z − ( −1)
Again, line (i) intersects line = = …(ii)
1 2 4
−2 − 1 3 − 1 −1 − 1
If a : b : c ≠ 1 : 2 : 4 and a b c = 0
1 2 4
⇒ 6a + 5b – 4c = 0
a b c
From (ii) and (iii) by cross multiplication, we have = =
8 − 5 6 + 4 5 + 12
a b c
⇒ = =
3 10 17
x−1 y−1 z−1
So, then required line is = =
3 10 17

Q. Prove that the three planes 2x + y – 4z – 17 = 0, 3x + 2y – 2z – 25 = 0 and


2x – 4y + 3z + 25 = 0 intersect at a point and find the coordinate of the points.

Vector & Three-Dimensional Geometry


Sol.
Given planes,
2x + y – 4z – 17 = 0
3x + 2y – 2z – 25 = 0
2x – 4y + 3z + 25 = 0
2 1 −4
D =3 2 −2 =−4 − 13 + 64 =47
2 −4 3

17 1 −4
D1 = 25 2 −2 =−34 + 325 =141
−25 −4 3

139.
2 17 −4
D2 = 3 25 −2 = 50 − 221 + 500 = 329 & similarly D3 = – 47
2 −25 3
D1 D2 D3
=x = ,y = ,z
D D D
141 329 47
x= ,y = ,z = −
47 47 47
x = 3, y = 7, z = – 1
hence point is (3, 7, –1).

Line of Greatest Slope


AB is the line of intersection of G-plane and
H-plane (horizontal plane). Line of greatest slope
on a given plane, drawn through a given point
on the plane, is the line through the point ‘P’
perpendicular to the line of intersection of the
given plane with any horizontal plane.

Q. Assuming the plane (4x – 3y + 7z) = 0 to be horizontal plane, find the equation
of line of greatest slope in the plane 2x + y – 5z = 0 and passing through the
point (2, 1, 1).

Sol.
The required plane must be perpendicular to the line of intersection of the
planes, i.e.
2x + y – 5z = 0 and 4x – 3y + 7z = 0
Let the direction ratio’s of the line of intersection of the given planes are a, b, c
Therefore, 2a + b – 5c = 0
4a – 3b + 7c = 0
Vector & Three-Dimensional Geometry

a b c
After solving, = =
4 17 5
Now direction ratios of required line be proportional to l, m, n and the line pass-
es through the point (2, 1, 1)
x−2 y−1 z−1
equation of line, = = …(i)
l m n
whereas, 2l + m – 5n = 0
4l + 17m + 5n = 0
l m n
After solving, = = …(ii)
3 −1 1

140.
Hence required line,
x−2 y−1 z−1
= =
3 −1 1
Sphere
A sphere is the locus of a point which moves
in space in such a way that its distance from
a fixed point always remains constant. The
fixed point is called the centre of the sphere
and the fixed distance is called the radius of
sphere. Shown as in adjoining figure.

Equation of Sphere whose Centre c and Radius


is a
Let O be the origin of reference and C be
the centre of sphere whose position vector
is c. Let P be any point on the surface of the
sphere whose position vector is r.

Thus, OP = r and OC = c
∴ CP = OP – OC = r – c
 
Now, | r − c | = a [radius of sphere]
  2
⇒ | r − c | = a2
   
⇒ | r − c | . | r − c | = a2
 
⇒ r2 − 2r. c + c2 = a2

⇒ r2 − 2r.c + (c2 – a2) = 0
which is the required equation of sphere.

Cartesian Equation of a Sphere

Vector & Three-Dimensional Geometry


The equation of sphere with centre (a, b, c)
and radius R is
(x–a)2 + (y–b)2 + (z–c)2 = R2

Proof :
Let C be the centre of the sphere.
Then, coordinates of C are (a, b, c). Let
P(x, y, z) be any point on the sphere,
then
CP = R
⇒ CP2 = R2

141.
⇒ (x–a)2 + (y –b)2 + (z–c)2 = R2
Since, P(x, y, z) is an arbitrary point on the sphere, therefore
required equation of the sphere is
(x-a)2 + (y-b)2 + (z-c) 2 = R2

Remarks
1. The above equation is called the central form of a sphere. If the centre
is at the origin, then equation of sphere is, x2 + y2 + z2 = R2
(known as the standard form of the sphere)
2. Above equation can also be written as
x2 + y2 + z2 –2ax –2by–2cz + (a2 + b2+c2 – R2) = 0
which has the following characteristics of the equation of sphere
(i) It is a second degree equation in x, y and z.
(ii) The coefficient of x2, y2 and z2 are all equal.
(iii) The term containing the product of x y, y z and z x are absent.

Q. Find the equation of sphere whose centre is (5, 2, 3) and radius is 2 in


cartesian form.

Sol. The required equation of the sphere is


(x - 5) 2 + (y - 2) 2 + (z - 3) 2 = 22
⇒ x2 + y2 + z2 - 10x - 4y - 6z + 34 = 0

General Equation of Sphere


The equation x2 + y2 + z2 + 2ux + 2vy + 2wz + d = 0
represents a sphere with centre (–u, –v, –w) i.e.
 1 1 1 
 − coefficient of x, − coefficient of y, − coefficient of z 
 2 2 2 
Vector & Three-Dimensional Geometry

and radius = u2 + v 2 + w2 − d
Point to Remember!!!

The equation x2 + y2 + z2 + 2ux +


2vy + 2wz + d = 0 represents a real
sphere. If u2 + v2 + w2 –d > 0. If u2
+ v2 + w2 –d = 0, then it represents
a point sphere. The sphere is
imaginary, if u2 + v2 + w2 –d < 0.

142.
Q. Find the centre and radius of the sphere 2x2 + 2y2 + 2z2 – 2x –4y + 2z + 3 = 0

Sol. The given equation 3


x2 + y 2 + z2 − x − 2y + z + =0
2
where centre is
 1 1 1 
 − coefficient of x, − coefficient of y, − coefficient of z 
 2 2 2 
1 1
∴ Centre =  , 1,- 
 2 2 
2 2
 1 2  1 3
and Radius =   + (1) +  –  –
2  2 2

1 1 3
= + 1+ − = 0
4 4 2
1 1
∴ Given sphere represents a point sphere  , 1,-  .
2 2

Q. Find the sphere through (1, 0, 0), (0, 1, 0), (0, 0, 1) having radius as small as
possible.

Sol. Let the equation of the required sphere be


x2 + y 2 + z2 + 2ux + 2vy + 2wz + d = …(i)
0
As the sphere passes through (1, 0, 0), (0, 1, 0) and (0, 0, 1), we get
1 + 2u + d = 0, 1 + 2v + d = 0 and 1 + 2w + d = 0
1
⇒ u = v= w= − (d + 1)
2
If R is the radius of the sphere, then R2 = u2 + v2 + w2 – d Vector & Three-Dimensional Geometry
3
⇒ R2 = (d + 1)2 − d
4
3 2 4 
= d + 2d + 1 − d
4 3 
3 2 2 
= d + d + 1
4 3 

3  1 
2
1
=  d +  + 1 −
4  3 9
 

143.
3  8
2
1
= d +  + 
4  3 9
 
The last equation shows that R2 (and thus R) will be the least if an only if
1
d= −
3
1 1 1
Therefore, u = v ==w − 1 −  = −
2 3 3
Hence, the equation of the required sphere is

2 1
( )
x2 + y 2 + z2 – (x + y + z) – = 0 or 3 x2 + y 2 + z2 – 2(x + y + z) – 1 = 0 .
3 3

Diameter Form of the Equation of a Sphere


If the position vectors of the extremities of a
 
diameter of a sphere are a and b , then its
equation is
 
) ( )
 
(r –a . r –b = 0
  
2  
( )
⇒           r – r . a + b + a . b = 0 

Proof :
Let a and b be the position vectors of the
extremities A and B of a diameter A B of
sphere. Let r be the position vector of any
point P on the sphere. Then,
   
AP = r − a and BP = r − b
Since, the diameter of a sphere subtends a
right at any point on the sphere, therefore
π
⇒ ∠APB =
2
 
⇒ AP · BP = 0
Vector & Three-Dimensional Geometry

   
⇒ (r − a).(r − b) -= 0
     
r.r − r.b − r.a + a .b = 0
     
| r | 2 − (a + b) . r + a . b =
0
This is the required equation of sphere.

Vector Form
If the position vectors of the extremities of
a diameter of a sphere are a and b, then its
equation is

144.
     
| r − a |2 + | r − b |2 = | a − b |2

Proof :
Let a and b be the position vectors of the extremities A and B of a
diameter of a sphere. Let r be the position vector of any point P on the
sphere, then,
 
AP = r − a
 
and BP = r − b
Since, ΔAPB is a right angled triangle.
∴ AP2 + BP2 = AB2
⇒ |AP|2 + |BP|2 = |AB|2
     
⇒ | r − a | + | r − b |2 = | a − b |2
This is the required equation of the sphere.

Cartesian Form
If (x1, y1, z1) and (x2, y2, z2) are the coordinates of the extremities of a
diameter of a sphere, then its equation is, (x–x1) (x–x2) + (y–y1) (y–y2) +
(z–z1) (z–z2) = 0

Q. Find the equation of the sphere described on the joint of points A and B having
Position vectors 2i + 6j − 7k and − 2i + 4j − 3k , respectively, as the diameter.
Find the centre and the radius of the sphere.

Sol. If point P with position vector r = xi + yj + zk is any point on the sphere, then
 
AP · BP = 0 .
(x – 2) (x + 2) + (y – 6) (y – 4) + (z + 3)(z + 7) = 0
⇒ (x2 – 4) + (y2 – 10y + 24) + (z2 + 10z + 21) = 0
⇒ x2 + y2 + z2 – 10y + 10z + 41 = 0
The centre of this sphere is (0, 5, –5) and its radius is 52 + ( −5)2 − 41= 9= 3 Vector & Three-Dimensional Geometry

Section of a Sphere by a Plane


Consider a sphere intersected by a plane. The set of points common
to both sphere and plane is called a plane section of a sphere. It can
be easily seen the plane section of sphere is a circle.
Let C be the centre of the sphere and M be the foot of the perpendicular
from C on the plane. Then, M is the centre of the circle and radius of
circle is given by PM.
i.e. =
PM CP2 − CM2

145.
The centre M of the circle is the point
of intersection of the plane and line
CM, which passes through C and is
perpendicular to given plane.

 
15 and | r − ( j + 2k ) | =
Q. Find the centre of the circle given by r ⋅ (i + 2 j + 2k ) = 4

j + 2k and normal to the given plane


Sol. The equation of a line through the centre
is
r= (j + 2k)
 + λ(i + 2j + 2k)
 …(i)

This meets the plane at a point for which we must have


(jˆ + 2k ) + λ(i + 2jˆ + 2k )] · (ˆi + 2 j + 2k ) = 15
⇒ 6 + 9λ = 15
⇒ λ = 1
On putting λ = 1 in Eq. (i), we obtain the position vectors of the centre as
î + 3j + 4k , Hence, the coordinates of the centre of the circle are (1, 3, 4)


Q. Find the radius of circular section in which the sphere r = 5 is cut by plane

r·(i + j + k)
 =3 3

Sol. Let A be the foot of the perpendicular from the centre O to the plane
r·(i + j + k)
 −3 3 =0
0·(i + j + k)
 −3 3 3 3
Then, |=
OA | = = 3
 
|i+ j+k | 3
(perpendicular distance of a point from the plane)
If P is any point on the circle, then P lies on the plane as
Vector & Three-Dimensional Geometry

well as on the sphere.


Therefore, OP = radius of the sphere = 5
Now, AP2 = OP2 – OA2 = 52 – 32 = 16
⇒ AP = 4

146.
147.
148.

You might also like